Jump to content

Wikipedia:Reference desk/Science: Difference between revisions

From Wikipedia, the free encyclopedia
Content deleted Content added
Line 1,061: Line 1,061:


What kinds of animals are predators of starfish? The [[starfish]] article mentions predators in at least two places, but does not elaborate. -- [[User:Dominus|Dominus]] ([[User talk:Dominus|talk]]) 03:58, 28 August 2008 (UTC)
What kinds of animals are predators of starfish? The [[starfish]] article mentions predators in at least two places, but does not elaborate. -- [[User:Dominus|Dominus]] ([[User talk:Dominus|talk]]) 03:58, 28 August 2008 (UTC)

== Do today's dyes weaken fabrics still? ==

My mom just told me that back in the days, yellow (I think!) clothing ripped most easily because yellow dye weakened the fabric. Is that true? Is it still true? What about other colors? Thanks. [[Special:Contributions/67.243.6.204|67.243.6.204]] ([[User talk:67.243.6.204|talk]]) 04:31, 28 August 2008 (UTC)

Revision as of 04:31, 28 August 2008

Welcome to the science section
of the Wikipedia reference desk.
Select a section:
Want a faster answer?

Main page: Help searching Wikipedia

   

How can I get my question answered?

  • Select the section of the desk that best fits the general topic of your question (see the navigation column to the right).
  • Post your question to only one section, providing a short header that gives the topic of your question.
  • Type '~~~~' (that is, four tilde characters) at the end – this signs and dates your contribution so we know who wrote what and when.
  • Don't post personal contact information – it will be removed. Any answers will be provided here.
  • Please be as specific as possible, and include all relevant context – the usefulness of answers may depend on the context.
  • Note:
    • We don't answer (and may remove) questions that require medical diagnosis or legal advice.
    • We don't answer requests for opinions, predictions or debate.
    • We don't do your homework for you, though we'll help you past the stuck point.
    • We don't conduct original research or provide a free source of ideas, but we'll help you find information you need.



How do I answer a question?

Main page: Wikipedia:Reference desk/Guidelines

  • The best answers address the question directly, and back up facts with wikilinks and links to sources. Do not edit others' comments and do not give any medical or legal advice.
See also:


August 21

Herbal infusion

Can I make a herbal infusion from the cannabis plant leaves.?

Cannabis (drug)#Drink. --JWSchmidt (talk) 03:08, 21 August 2008 (UTC)[reply]
Do you want to know a) if it is legal, b) if it is technically possible, c) if it is a good idea to do so, or d) other? Wanderer57 (talk) 03:25, 21 August 2008 (UTC)[reply]
Note the key point from the above article. Because THC is lipophilic, it's not a particularly effective way if you want to get stoned... That's why people tend to make cannabis cookies, cakes and food items containing a fair amount of fat. You could try perhaps some sort of cannabis Cendol or other dessert or drink with a liquid base containing a fair amount of fat if you really prefer it in drink form (never tried it myself but it seems plausible). Nil Einne (talk) 11:42, 21 August 2008 (UTC)[reply]
However, if you use one of those recipes for Tibetan butter tea that can be found on the net, your lipophilic troubles are eliminated. Butter tea needs getting used to, though. --Ayacop (talk) 16:07, 21 August 2008 (UTC)[reply]
Science! It like totally does stuff, you know dude? Oh man! SamuelRiv (talk) 00:51, 22 August 2008 (UTC)[reply]
Bhang Lassi. DuncanHill (talk) 08:56, 22 August 2008 (UTC)[reply]

eye damage from hand dryers

Hi, I read just now on a hand dryer in a public toilet, that if you blow the hot air in your face, there is a risk of eye damage, from something called "fusion." What is the story here? 134.115.68.21 (talk) 08:36, 21 August 2008 (UTC)[reply]

I don't know the specifics, but it seems blindingly (pun unintended) obvious that if your eyes get too hot, there's a strong risk of damage and given the low level of pain receptors in the eye, you may not even realise Nil Einne (talk) 11:46, 21 August 2008 (UTC)[reply]
Unless the dryer was made by the Fleischmann & Pons Corporation, I'd say they're referring to the danger of a plastic contact lens fusing to the eye because of the heat; ouch! --Sean 16:54, 21 August 2008 (UTC)[reply]
It might not even be the heat. Could it be that the drier would normally dry the liquid from the surface of your eye - but you'd blink and tear up a bit - and no harm is done. But one of those soft contact lenses that's permeable to water would dry out without you noticing and form a water-tight seal against the eye - with no way for liquid to get back inside and rehydrate it? That's a guess though. SteveBaker (talk) 17:42, 21 August 2008 (UTC)[reply]

Astronomy Question: Need a Repeating Celestial Event

I'm looking for an astronomical/celestial event for a story I'm writing. Thus far, I haven't been able to find anything that exactly fits what I'm looking for. My conditions are this: the event has to repeat every 200 to 300 years. It has to be visible from the same location on Earth. Also, I'd rather not go with a comet if I can help it. Can anyone give me some suggestions, or a pointer in the right direction? Thanks. --Brasswatchman (talk) 08:45, 21 August 2008 (UTC)[reply]

The Transit of Venus occur in a pattern that repeats every 243 years, with pairs of transits eight years apart separated by long gaps of 121.5 years and 105.5 years. Of course the sun has to be up for one to see it, but as half of the transits currently occur in june, there are populated places with midnight sun that could have viewed it regardless. In any case, the article has a table of the timings. EverGreg (talk) 10:16, 21 August 2008 (UTC)[reply]
And if you did choose the transit of Venus (which I think is an excellent choice, personally), you'd have a rich set of historical anecdotes to draw on and even some interesting scientific controversies (the "black drop effect" has been a major issue regarding the transit of venus, and there is ample historical material out there about international attempts to resolve it in the 19th century, etc.). In 1874 it was referred to as the "astronomical event of the century". --98.217.8.46 (talk) 12:18, 21 August 2008 (UTC)[reply]
As with a solar eclipse, observing a transit of Venus is fraught with risk of serious eye damage. Whether this might affect your story, I don't know. Wanderer57 (talk) 12:24, 21 August 2008 (UTC)[reply]
The trouble with events that widely separated is that the previous occurrance would have been back in the 1700's when telescopes were still pretty primitive devices. So you're really needing a more or less "naked eye" event. Comets really are your best chance. If this is fictional - then a meteor shower might do what you need. Events like the Perseids happen annually as the Earth passes through a stream of debris that was originally a comet that broke up a very long time ago. That debris stretched out so far that pretty much the entire orbit of the original comet has debris within it. But if the comet had been one that repeated every 300 years and if it had only broken up relatively recently (on the scale of cosmic events) then there might well be a bunch of debris that only intersects the Earths orbit every 200 to 300 years. I don't know of any such events in reality...but fictionally, I don't see a problem with inventing one. Anyway - a meteor shower (especially from a relatively recently broken up comet) could be as spectacular as you need it to be. Anything from chunks of rock and ice hurtling out of the sky smashing buildings and splattering our Hero's one true love over half a county...down to a bunch of gentle shooting stars that commemorate that tragic event and bring our Hero's great, great, great, great grandson together with the (sadly splattered) heroines' great, great, great, great granddaughter together. Anyway - it would occur on the exact same night of the year - although not generally at the exact same time of the night.
Oh - and if you need the last occurance to have happened before 1753 - beware of calendar reform! Actually, this would make a great plot point! Suppose the original sighting of the event happened on September 1st 1752, with a prediction that it would happen again precisely 250 years later on Sept 1st 2002. Our modern day heroes might initially be unaware of the fact that 11 days were 'dropped' from the calendar in 1752 (so the day after September 2nd 1752 was September 14th 1752!!). They would gather with the bad guys holding guns to their heads (or whatever) at the appointed place on September 1st 2002 - and be horrified to find that the specified event doesn't happen as predicted. We're confused, they are confused, the bad guys go away - could the prediction really not be true? Then, just a week and a half later when hero and heroine are strolling in some romantic place believing that the prediction was a lot of nonsense - he proposes to her and at that exact moment the sky bursts forth with meteor trails (because it's September 12th) - which is actually 250 years later to the day exactly on schedule. A wandering Wikipedian strolls past and comments "Hey didn't you read Calendar reform?".
This stuff practically writes itself!
Good luck with your book! SteveBaker (talk) 13:21, 21 August 2008 (UTC)[reply]
And here I thought Steve was going to mention the change of New Year's Day, which in England took place about the same time, so that one year after March 24, 1750, it was March 24, 1752. --Anonymous, 21:39 UTC, August 21, 2008 (N.S.).
The trail of debris left by a comet will cause a meteor shower every year, but you can get more dramatic ones in certain years due to particular passes of the comet leaving clumps in specific places. It might be plausible to have a extra dramatic shower once every 250 years, or whatever. I think a transit of Venus is a better choice, though - it's almost tailor made for the OP's requirements. --Tango (talk) 17:55, 21 August 2008 (UTC)[reply]
Yes - certainly the Perseids have good years and bad years - suggesting a certain clumpiness. But it kinda surprises me that the debris spread so far through the original orbit of the comet. It seems that the debris was created about a thousand years ago and came from the 109P/Swift-Tuttle comet. For the purposes of a story, the comet could have broken up much more recently - leaving the debris in a relatively compact bunch that would orbit with roughly the same period as the comet - which could easily be 200 to 300 years. That would also produce an event with a fairly sharp onset and cutoff if most of the debris were still fairly close together. SteveBaker (talk) 00:17, 22 August 2008 (UTC)[reply]
Does it have to be set on Earth? How about a planet orbiting a flare star? (eg. a star like Groombridge 1618) Astronaut (talk) 18:30, 21 August 2008 (UTC)[reply]
Brasswatchman: Can we consider this question resolved, or would you prefer to keep it open? --Bowlhover (talk) 09:24, 23 August 2008 (UTC)[reply]

"Green Bags"

There's an ad on TV for "Debbie Meyer's Green Bags", which promise to almost stop fruits and veggies from rotting. The claim is that they remove ethylene gas produced by food which would otherwise cause spoilage. They also say the bags can be reused up to 10 times. I'd like to know if these claims are legit:

1) Do all fruits and veggies produce ethylene gas ?

2) Does this gas promote spoilage ? If so, to what degree ? The commercial makes it look like there is no spoilage once this gas is removed.

3) Do the bags remove 100% of the gas, or only a tiny portion ?

4) Could they really be reused 10 times without losing their effectiveness ?

5) Are there other side effects, like stopping fruit from ripening ? I prefer yellow bananas over brown, but brown is still better than green.

6) I'd also be interested in seeing a cost/benefit analysis. That is, does the cost of the bags outweigh the financial benefit of avoiding lost produce ?

7) Is there a more efficient way of removing ethylene than disposable bags ? StuRat (talk) 12:37, 21 August 2008 (UTC)[reply]

Update: I found these reviews, which are mixed to negative: [1]. However, I'd still like to know more about the science that's going on with the bags, so still would like answers to the Q's above. StuRat (talk) 12:50, 21 August 2008 (UTC)[reply]

Ethylene as a plant hormone may be of interest, and this site has an interesting sensitivity chart. Moving into the questions:
  1. Most, certainly. "All" is a difficult blanket statement.
  2. Promote, absolutely. Sole factor, generally no. Per the sensitivity chart, reaction to ethylene varies.
  3. Who knows?
  4. It probably depends on how long you're keeping fruit in the bag on each of the 10 times. I'm betting on an over-optimistic marketing pitch, but that's just me.
  5. Yes, rotting and ripening are both effects of ethylene.
  6. Depends on you personally. How much produce are you losing without the bags? How much produce would you lose if you just bought produce on an on-demand (or nearly so) basis? It's unlikely you save anything if you're buying and storing produce responsibly to begin with.
  7. Good ventilation is the easiest means of reducing ethylene quantities. Segregating ethylene producers from ethylene-sensitive produce is another useful approach. There are other methods, but they seem to be targeted at the industrial level. — Lomn 12:56, 21 August 2008 (UTC)[reply]
As for question #4, if they do in fact work, I wonder if telling you they can only be used for 10 times is just a ploy to get you to buy more. Just a thought; there may be a chemical in the lining of the bag that reduces ethylene that runs out after so many uses, and of course you don't want to reuse a bag for food too many times, but still, I wonder.--El aprendelenguas (talk) 20:10, 21 August 2008 (UTC)[reply]
I've found this more science-based set of reviews. The reviews are mixed, but it is clear that the bag is more effective with some products than others. It's also clear that leaving produce in a cool, open place is a better way to prevent spoilage.
To add some original research of my own, ethylene has about the same density as air and, for that reason, escapes very easily with minimal air circulation. Trapping the ethylene in a plastic bag is not the best way to promote circulation. --Bowlhover (talk) 10:24, 24 August 2008 (UTC)[reply]

The Large Hadron Collider and the Sun

Is the sole purpose of the large hadron collider to try to create black holes by accelerating hadrons to high speed and then crashing them into each other, or are there other experiments as well?

Is it likely that there are hadrons of comparable energy inside the sun? To put the question another way, is it likely that micro black holes are being created within the sun? If so, would this be a common event or a rare one? Thanks, Wanderer57 (talk) 16:04, 21 August 2008 (UTC)[reply]

Producing black holes is not a primary goal or purpose of the LHC at all. The only reason you hear about the LHC and black holes is that some cranks filed a lawsuit, because they thought it would cause the end of the world. See Large_Hadron_Collider#Research for its major goals. If you had to pick one goal, it would probably be the discovery of the Higgs boson. -- Coneslayer (talk) 16:13, 21 August 2008 (UTC)[reply]
It's interesting to note that we really do have machines which are reasonably likely to end the world at some point, but these folks didn't see fit to try to sue them out of existence. It's like fretting about spontaneous human combustion when there's a snake in the room. --Sean 16:46, 21 August 2008 (UTC)[reply]
Well, it sort of is apples and oranges from a legal point of view. --98.217.8.46 (talk) 19:52, 21 August 2008 (UTC)[reply]
The energy in each collision of individual nuclei in the LHC will be around 10-4 joules. A rough calculation shows that this is equivalent to a temperature of around 1019 K. The temperature at the centre of the Sun is only around 107 K. So LHC collisions will be much much more energetic that reactions within the Sun. Gandalf61 (talk) 16:19, 21 August 2008 (UTC)[reply]
For comparison, see Orders_of_magnitude_(temperature). --Ayacop (talk) 16:31, 21 August 2008 (UTC)[reply]
If your question was really "do such high energy particles/collisions occur naturally and have we seen any disastrous effects elsewhere?" the answer is yes for the particles and no for the disastrous effects, see Ultra-high-energy cosmic ray. Well if someone would be hit directly that would not be healthy, but neither the end of the world nor the end of our planet. 93.132.168.99 (talk) 16:54, 21 August 2008 (UTC)[reply]
Well, not quite. The idea that a cosmic ray could produce whatever weird/dangerous particles the LHC might produce is valid - but if it were that easy, we wouldn't need the LHC! The problem is that any particles appearing from a cosmic ray collision would continue to move at spectacular speed and probably go right through the earth and out the other side without anyone noticing. Even if you were hit by a mini-black hole coming about from a cosmic ray event, it would drill a hole through your body that would be considerably smaller than the diameter of an atom...you wouldn't notice. But if the LHC were to produce such an object, it could be stationary - or moving very slowly...so (the theory says) it could be a real problem. I think that's nonsense - but that was the thesis put forth in the law suite.
Anyway - the purpose of LHC is to "discover stuff" - and you don't always know what you're going to discover until after you've discovered it. It's hoped that there may be further evidence for dark matter - perhaps some of the predictions of string theory may be tested...and there is a lot of confidence that the Higgs Boson may be discovered. The Higgs is predicted to be a very heavy particle and because of E=mc2 it takes more energy to make particles with more mass. Since the energy comes from slamming particles together, the energy comes from the kinetic energy of the particle that's doing the hitting. The giant circular tunnel accelerates particles to higher speeds than we've ever been able to do before.
SteveBaker (talk) 17:18, 21 August 2008 (UTC)[reply]
(ec with SteveBaker, somewhat redundant now) The purpose of the LHC is to see what there is to see in that energy regime. Theoretical particle physics has failed spectacularly at making predictions in the last 30 years, so nobody really knows what to expect. Almost everyone thinks it will see the Higgs boson, since the Standard Model limits its mass to a range that's easily accessible to the LHC. It's not really the main goal of the LHC to find the Higgs boson, it's just the one thing most physicists agree it will succeed in doing. Some people expect to also see black holes, some expect to see superpartners, some expect to see other exotic particles (about which I know nothing). Some people think the Higgs won't be found. Nobody wants the Higgs to be found; it would be much more interesting if it wasn't. The worst-case scenario for the LHC is finding a fundamental scalar Higgs particle and nothing else, giving theorists almost no clues about physics beyond the Standard Model. That could very well mean the end of particle physics.
I don't know much about conditions inside the Sun, but collisions at the energy the LHC will produce happen all the time. The only reason they need the LHC is to make them happen at a predictable place and time, so that they can stick a huge honkin' detector there and analyze the result. The proposed ILC would produce lower energy collisions than the LHC but under even better controlled conditions. -- BenRG (talk) 17:22, 21 August 2008 (UTC)[reply]

Apparently Natural Causes?

Could we please have medical input to the discussion here:

Talk:Jack the Ripper#Apparently Natural Causes!!!

Thanks, Wanderer57 (talk) 17:05, 21 August 2008 (UTC)[reply]

  1. The standards of medicine in the 1880's were kinda poor so it's perfectly possible that nobody really checked for precise cause of death.
  2. She died many months before the ripper murders became notorious - so there was no particular reason to pay careful attention to investigate and record cause of death - a lazy doctor could just have scribbled "Natural causes" in order to avoid a lot of hassle.
  3. She died sometime AFTER she'd been discharged from hospital - over a month after she'd been stabbed - it seems unlikely that she was seriously injured at the time she died.
  4. If she'd gotten some kind of general infection, it's perfectly possible that they would not have connected her death to injuries from the wounds - and perhaps a death from a serious infection would have been impossible to diagnose at the time.
  5. It's unusual for someone who is only 38 years old to die of "natural causes" in this day and age - but even as late as 1900, life expectancies for urban poor were hovering around 45 years. So it's quite possible for the report to be 100% true.
I don't doubt that was the report of the time - but it's hard to know whether the report was accurate or not.
SteveBaker (talk) 17:34, 21 August 2008 (UTC)[reply]
Who she was and what she did for a living could also have prejudiced a doctor into not putting a lot of effort into her diagnoses. It's well known that cases of "undesirables" dying are often given far less attention by authorities, who are not looking to make more work for themselves if nobody cares or notices if they brush things under the rug. Just a speculation. --98.217.8.46 (talk) 19:42, 21 August 2008 (UTC)[reply]

Liquids going from perspiration to waste in humans

This may be a rather simple answer - at the point it reaches the kidneys, perhaps - but the womens' Olympic marathon brought to mind a query I'd had from a number of years ago. I'd consumed two bottles of water walking in 95 degree F heat for a couple hours,a nd hardly had to use the bathroom at all; it had all been sweated out.

My question is, at what point does water consumed go from being utilized as sweat to being discarded as waste? My guess from both articles would be that anytime before it reaches eht kidneys, it's game for being sweat, but afterward, it's automatically urine, but I'm not sure; it almost seems like if you've really gotta go, and yet you're running a lot, it can still wind up used as sweat even if it's in the bladder. Is the answer true for other liquids, too? Or, only with respect to the amount of water in them? (Weird, the thing to sign wasn't illuminated before. Now it is.) Somebody or his brother (talk) 19:03, 21 August 2008 (UTC)[reply]

The kidney is the irreversible/committing step where water goes to urine. Once it diffuses out of the blood and is not re-absorbed in the collecting ducts, there's no further chance for it to go anywhere else except ureter→bladder→urethra→outside. DMacks (talk) 19:09, 21 August 2008 (UTC)[reply]
Water also goes out through the digestive system but it can be recovered from the colon when needed. Franamax (talk) 22:37, 21 August 2008 (UTC)[reply]

what is important of material sciences?

—Preceding unsigned comment added by 116.71.61.200 (talk) 18:56, 21 August 2008 (UTC)[reply] 
Not really a coherent question here. But did you read the material science article? Think about how important it is that someone made the stuff your keyboard and monitor are made out of. DMacks (talk) 18:59, 21 August 2008 (UTC)[reply]
Studying it gives me something to do outside of Wikipedia. the wub "?!" 23:17, 21 August 2008 (UTC)[reply]

Telephone call to 911

Let's say that a person is incapacitated, cannot speak, and has absolutely no idea where they are (as a result of an accident or a crime or for whatever reason). The only thing the person can manage to do is to dial "911" on a telephone. Scenario A: If that person dials 911 from a landline phone, my understanding is that the 911 dispatchers can tell immediately what the location of the landline phone is. And, if the incapacitated person does not speak (after having made the 911 call), they will still dispatch help to the correct address. Is all of that correct? Scenario B: Now, what exactly happens if the 911 call was made from a cell phone? Does anyone know how exactly this works? I assume -- at some point -- and through technology -- they can somehow locate the cell phone and the person. Now, will this take a few minutes, hours, days, weeks, what? In other words ... in an emergency, how long would that incapacitated person expect to wait before help arrived, when summoning 911 from a cell phone? Also, are there procedures (similar to the landline phone) where they dispatch help no matter what, even if the incapacitated person on the cell phone does not / cannot speak? Thanks. (Joseph A. Spadaro (talk) 19:49, 21 August 2008 (UTC))[reply]

You should probably start with Enhanced 911. -- Coneslayer (talk) 19:52, 21 August 2008 (UTC)[reply]
The FCC's page on wireless 911 may also be of interest. — Lomn 19:53, 21 August 2008 (UTC)[reply]
I am fairly sure that 1. is correct at least in some locations. I have friends who dialed 911 and then hung up after a second or two and the cops did indeed show up their door. (It's a long story. It involved LSD. It worked out fine in the end. Just in case you're wondering.) As for a cell phone, hypothetically you can locate which cell tower the call came from (which puts you within a mile or so), and maybe even triangulate it based on information from multiple towers, I suppose, but emergency dispatchers do not currently have that capability as far as I know. If you don't tell them where you are, they don't know. Eventually I am sure this will change. --98.217.8.46 (talk) 20:05, 21 August 2008 (UTC)[reply]
As long as the phone is still turned on, you can track it more accurately. I'm not sure of the details, and I'm not sure if it's routinely used, but it can be done. --Tango (talk) 20:14, 21 August 2008 (UTC)[reply]
One thing the dispatchers get is the phone number, so that they can all back to see if the call was an accident, or a prank, or genuine. There is also a special number for tty machines if the user has such a facility. Graeme Bartlett (talk) 21:35, 21 August 2008 (UTC)[reply]
In many jurisdictions, if you don't talk to the 911 operator and simply hang up, the first thing they do is try to call you back at the same number. Also, if you have the misfortunate of living in my jurisdiction, you can call 911 and listen to a ridciulous "All operators are busy, please stay on the line" message for 10 minutes. This happened to me when trying to report a car crash. I can imagine that such delays have probably resulted in a least a few deaths from people that might otherwise have been saved. Dragons flight (talk) 01:00, 22 August 2008 (UTC)[reply]
The dispatch depends on the Emergency Medical Dispatcher protocols used by the authority having jurisdiction. In my county, this type of call goes out as an "unknown rescue;" fire, EMS and police respond together. Also see 9-1-1. --Shaggorama (talk) 03:28, 22 August 2008 (UTC)[reply]

Thanks. So, if the incapacitated person is on a cell phone and can't speak or offer any location clues ... how long would they have to wait for help to arrive? Minutes, hours, days, weeks? Thanks. (Joseph A. Spadaro (talk) 11:07, 22 August 2008 (UTC))[reply]

Depends where they are. Are they in a car accident on a major highway? Probably get found pretty quick. Are they hidden in somebody's basement? Might never get found. --98.217.8.46 (talk) 13:52, 22 August 2008 (UTC)[reply]
This hypothetical incapacitated person is just in a house, which he does not know the address of. (Joseph A. Spadaro (talk) 15:25, 22 August 2008 (UTC))[reply]
I dispatch for an ambulance company on Long Island and, along with the calls that we receive from MEDCOM, we also get a print out. The paper, along with the complaint of the caller, has the phone number call came from, the name of who the phone is registered to, and approximate longitude and latitude of where the call came from, if it came from a cell phone. If someone calls 911 and immediately hangs up, the 911 operator calls the number back with a message saying that in order to receive help, they need to call 911 again and give an address. -- MacAddct  1984 (talk • contribs) 18:50, 22 August 2008 (UTC)[reply]

Thanks for the input. Much appreciated. (Joseph A. Spadaro (talk) 03:57, 23 August 2008 (UTC))[reply]

Is that right? i heard reports that in the UK, if you dial 999 from a mobile you don't necessarily get put through to the right district (unlike if you rang from a landline). I.e. if you say you've been left for dead on George Street it's quite possible that an ambulance will show up in Edinburgh rather than London. Presumably if you didn't say anything the problem would be ten times worse. (i've also heard that this problem is solved if you dial 112 (the pan-european number) which doesn't quite seem right -have i been lied to?) 82.22.4.63 (talk) 20:25, 28 August 2008 (UTC)[reply]

Solutions and volume changes

In a typical solution of a crystal solid in a liquid e.g. salt in water, is there a change in the volume of the solvent as the crystal dissolves? - is any change the same as or proportional to the volume of solute? —Preceding unsigned comment added by Webb202 (talkcontribs) 20:34, 21 August 2008 (UTC)[reply]

I'm pretty sure there is a change in volume, but I can't confirm if that change is equal to the amount of solute added. Coolotter88 (talk) 20:47, 21 August 2008 (UTC)[reply]
This is certainly interesting. Would the break down of the salt crystals to ions cause the salt to occupy less volume? I think the answer is yes but it would be very insignificant change.--155.144.40.31 (talk) 00:58, 22 August 2008 (UTC)[reply]
Some materials have a very large volume difference when dissolving ("volume of solute A + volume of solvent B >> volume of solution of A in B"). It has to do with the dissolved particles (ions or solvated molecules) fitting into the existing spaces in the solvent rather than completely displacing solvent molecules. Doesn't even matter if the solute is a solid or not (alcohol+water is an easy experiment to do at home). DMacks (talk) 02:42, 22 August 2008 (UTC)[reply]
The concept you are looking for is called partial molar volume. --Heron (talk) 15:43, 22 August 2008 (UTC)[reply]
Ah yes. My engineering friends talk about volume of mixing, but we didn't have a page on that term. DMacks (talk) 16:50, 22 August 2008 (UTC)[reply]

Thanks for the info -Webb202 —Preceding unsigned comment added by Webb202 (talkcontribs) 13:24, 24 August 2008 (UTC)[reply]

Optical zoom

Does optical zoom refer to distances or area? If I have a *4 zoom and take a picture of a square "zoomed out" and it is 100*100 pixels, when "zoomed in" by *4 will it be 400*400 or 200*200? -- SGBailey (talk) 21:19, 21 August 2008 (UTC)[reply]

It's linear, so 400*400. It's the ratio of the focal lengths at the longest and shortest zoom positions. -- Coneslayer (talk) 22:40, 21 August 2008 (UTC)[reply]
It's linear if you're focused at infinity, but (if I've worked it out correctly) the width of the visible area of the focal plane in general is dw(1/f − 1/d), where d is the distance from the lens to the focal plane, w is the width of the detector (CCD), and f is the focal length. That would imply that the magnification is somewhat more than 4× if you're focused in close. However experimentation with my point-and-shoot digicam has failed to back this up, so maybe I made a mistake. -- BenRG (talk) 00:20, 22 August 2008 (UTC)[reply]
Many photographic lenses have an effective focal length that changes as you focus closer, especially if they have floating elements for zooming or to reduce aberrations at close-focus distances; see, for example, the charts here. This property could confound your measurements. -- Coneslayer (talk) 14:42, 22 August 2008 (UTC)[reply]
Ta -- sgb

For Chemistry Experts

What would result from the following?; If Methane (Ch4) is present in water (H2O) and Hydrogen Peroxide (H2O2) is introduced.Rukiddin (talk) 23:20, 21 August 2008 (UTC)[reply]

A solution of two gases in water. Need some activation energy to make something fun happen. In terms of organic mechanisms at least, you'd need to get the radicals going (H2O2 is often a difficult-to-start radical initiator unless there's something very reactive available to do something) and methyl radical is one of the least stable (hardest to form) ones:( DMacks (talk) 23:27, 21 August 2008 (UTC)[reply]


August 22

When someone says that they can see the intelligence in an animal's eyes...

...what exactly is the phenomenon that they're describing? I found myself pondering this whilst a Grey Heron and I were silently observing each other at close quarters earlier today. --Kurt Shaped Box (talk) 00:50, 22 August 2008 (UTC)[reply]

Anthropomorphism might fit. The more something appears to be volitional (an animal, a storm, the sea), the likelier humans are to believe the thing observes and reacts as humans do. I am not expert on herons, grey or otherwise, but since they feed off things like fish and frogs, the lack of motion and steady gaze likely come from hunting instinct. There's also not a great deal of brain in a creature weighing 2 kilograms. — OtherDave (talk) 01:00, 22 August 2008 (UTC)[reply]
I'm not an expert on herons myself (in fact, this is the first one I've ever seen at a distance closer than 100ft), so I have no idea where they rank in terms of intelligence and I accept that I may just be anthropomorphizing, but during seconds that my eyes met those of the heron, I got a distinct feeling that the bird was 'weighing me up' and 'observing', as opposed to just looking at me. Man, it's really hard to describe. I guess it's a bit like when a pet owner says that they can tell when their dog/cat/bird/whatever is curious or preoccupied by something by the way it looks at them... --Kurt Shaped Box (talk) 01:22, 22 August 2008 (UTC)[reply]
Birds can be surprisingly intelligent. I have similar experiences when making eye contact with our Cockatiel. It could just be a psychological thing that we're thinking way too hard about. --Russoc4 (talk) 01:52, 22 August 2008 (UTC)[reply]
On the other hand, magpies were recently added to the very short list of animals that are self-aware, an honor they share with great apes, Asian elephants, and the bottlenose dolphins. [2]. Dragons flight (talk) 07:45, 22 August 2008 (UTC)[reply]
There's a huge amount that's just how much sclera one can see. There's been a lot written about that of late—that humans judge eyes with a lot of white visible to be closer to their own intelligence, and the question of why humans evolved to have so much visible (unlike most animals). Additionally I think the ability of the animal to focus on one discrete object or make eye contact helps. Things that are looking all over the place look wild. When my dog makes eye contact with me and holds it, it's hard to argue that there's some kind of connection. --98.217.8.46 (talk) 02:38, 22 August 2008 (UTC)[reply]
Could the sclera thing also explain why animals with pale irises are often considered to be more vicious or angry-looking than animals with dark irises? In many animals, the iris pretty much covers the same area of eye as the 'whites' of ours eyes do. Consider these two Herring Gulls: --Kurt Shaped Box (talk) 23:03, 22 August 2008 (UTC)[reply]
Not particularly friendly
Friendlier-looking than the above
I think the reason why an animal that looks into your eyes seems intelligent is as follows: This implies that they know you can see them with your eyes, which requires a certain degree of intelligence to work out. StuRat (talk) 02:59, 22 August 2008 (UTC)[reply]
In other words, you get the feeling that the animal has a theory of mind, which is significant, because baby humans don't catch on to this for a while. ike9898 (talk) 16:41, 22 August 2008 (UTC)[reply]
There are a few references at Empathy#Empathy_and_animals which would imply that there is a possible difference between ability to identify with others of the same species, and other species. --Ayacop (talk) 08:06, 22 August 2008 (UTC)[reply]
Don't forget that humans were prey (as well as predator) for most of our evolutionary history, so it's possible that when we notice another creature watching us, we pay more attention. Doesn't even have to be the archetypal saber-toothed tiger. — OtherDave (talk) 15:49, 22 August 2008 (UTC)[reply]
Isn't George W. Bush our current expert on this question? Or at least, he was until a couple of weeks ago when Putin apparently got an occular transplant and invalidated Bush's previous conclusions.
Atlant (talk) 13:50, 23 August 2008 (UTC)[reply]

Drunk Driving deaths by country

I'm looking for a refernce source that will list drunk driving and or alcohol-related deaths by country. So far it looks like I could probably find those statistics if I searched country by country, but I was hoping some website existed where it's already been aggregated. --Shaggorama (talk) 02:26, 22 August 2008 (UTC)[reply]

Unfortunately I can't answer your question, but I'm prepared to offer some comments and advice which will make it more difficult for you to trust any results you find. You'll need to be very careful with the definitions that you use; I suspect it will be difficult for you to find a genuine apples-to-apples comparison. Do you define drunk-driving deaths as those where an intoxication-related criminal charge was laid? Do you count the case as an 'alcohol-related death' if it involved a sober driver and a drunk pedestrian? Do you look at blood alcohol content—remember that the legal limit varies from country to country and may also depend on type of license. (A look through our Driving under the influence shows limits ranging from zero to 0.08%.)
Some anti-drunk-driving organizations are also a bit dishonest with their statistics, reporting fatalities as 'alcohol-related' as long as at least one individual involved in a collision had any measurable blood alcohol content, regardless of the circumstances of the accident or who was actually at fault. TenOfAllTrades(talk) 02:54, 22 August 2008 (UTC)[reply]
I'm with both of you. I'm not inclined to really trust the numbers I find, but I'm not in a position to produce them myself so for my purposes I'll take what I can get. One of the reasons I'm looking for one single resource is I'm hoping it would largely unify the definitions used in producing the stats. But I'm probably being naive, seeing as I am looking for statistics that, as TOAT pointed out, are often produced by biased propagandizing organizations. --Shaggorama (talk) 03:18, 22 August 2008 (UTC)[reply]
Sorry about the confusion—there's actually just one of me. :D I forgot to indent my second paragraph (now fixed). Even if you get your numbers from 'honest' sources, you're apt to run up against the 'definitions' problem (0.03 is over the limit in Poland but not in Portugal, so might be reported differently by police and governments in the two countries) and the 'detection' problem (not all jurisdictions necessarily test all drivers for intoxicants). Best of luck to you, however! TenOfAllTrades(talk) 15:06, 22 August 2008 (UTC)[reply]

Some stuff to try: a 2007 report (PDF file) from the World Health Organization that includes a chart (page 3) comparing stats in about 30 countries. There might (not sure) be something also in this WHO report (links to several pdfs) on traffic accidents. There could also be stats in this Global Road Safety Partnership report (PDF), though for developing countries only. Finally, it seems the International Road Federation has stats, but not for free. Hope something here helps,WikiJedits (talk) 19:36, 25 August 2008 (UTC)[reply]

not a question, but... worth it (LHC)

"The Large Hadron Collider is set to fire up on September 10. Not sure why, but don't want to slog through tedious explanations of the Higgs boson and the Standard Model? Have a look at this informative rap narrative, delivered by persons in lab coats and hard hats."

http://blogs.spectrum.ieee.org/tech_talk/2008/08/switzerlands_nerdcore_scene.html

You'll laugh your head off. --Halcatalyst (talk) 02:34, 22 August 2008 (UTC)[reply]

I think they left out the part about how it'll blow up the world, right? ;-) --98.217.8.46 (talk) 02:52, 22 August 2008 (UTC)[reply]
The good thing about blowing up the world is that you can't end up looking bad because of it. If they'd rapped about the end of the world and it didn't happen, we'd all laugh at them. This way, whatever happens, they can't be proven wrong. --Tango (talk) 02:54, 22 August 2008 (UTC)[reply]
This is the way the world ends
This is the way the world ends
This is the way the world ends
Not with a bang but a whimper.
The Hollow Men --Halcatalyst (talk) 03:50, 22 August 2008 (UTC)[reply]
Heh. I just read Flashforward (novel), a SF novel dealing with the LHC. --—— Gadget850 (Ed) talk - 13:43, 22 August 2008 (UTC)[reply]
I like the idea but the rapping is awful. They could stand to take some lessons from Weird Al. -- BenRG (talk) 14:25, 22 August 2008 (UTC)[reply]
Maybe that's why the performers preferred to remain anonymous, heh. --Halcatalyst (talk) 16:14, 22 August 2008 (UTC)[reply]

Frostbite

If frostbite is caused by the body trying to maintain the core body temperature, is it possible to get frostbite if only one part of your body is exposed to extreme cold, for example if you stick your hand in liquid nitrogen? —Preceding unsigned comment added by 218.5.84.167 (talk) 05:55, 22 August 2008 (UTC)[reply]

Did you read frostbite. It usually affects toes or extremities of the body. Graeme Bartlett (talk) 07:09, 22 August 2008 (UTC)[reply]
Extremities generally lose heat faster, but you can get frostbite anywhere if that part of your body gets cold enough. And yes, holding your hand in liquid nitrogen would do it. (Incidentally, it's not instaneous. Brief contact with LN2 will create a significant signal of pain and normally one would yank their hand back before it froze. The same way that touching hot things will generally cause you to recoil before any major damage is done.) There are cases where someone's leg stepped through a frozen lake and they quickly got frostbite on their leg while the rest of them was okay. That's probably most analogous to what you are asking. Dragons flight (talk) 07:38, 22 August 2008 (UTC)[reply]
I agree with Dragons flight. To the person posing the question, it's a bit misleading to say that frostbite is caused by the body trying to maintain core temperature. In a localized situation, like the hand in nitrogen or feet frostbitten from immersion in the water, the frostbite's caused by (a) reduced blood flow followed by (b) freezing of tissue (as the frostbite article points out). In a whole-body situation (see Jack London's To Build a Fire) yes, the reduced blood flow occurs in all the extremities. If you were cross-country skiing in bitter cold, with all of your body well protected except your face and ears, your core temperature would probably be just fine. Stay exposed long enough, though, and you'll get frostbite. — OtherDave (talk) 16:11, 22 August 2008 (UTC)[reply]

Physics with Calculus queston, regarding differential mass elements

For chapter 9, my book describes the center of mass of a solid body on an x-axis as the integral from x1 to x2 of the differential mass elements all multiplied by 1/Mass

My question is it calls the differential mass elements, by the symbol dm which also occurs as the last symbol of the integral.

Are differential mass elements the mass of my object between 2 infinitesimal close points on the x-axis? In other words if I have a rod from x=0 to x=5, then divide this distance into infinite divisions, and the mass of each sub-distance totally comprises the mass of my object? Therefore summing all the differential mass elements will give you the "area under the curve".

The other way I can see it is that the term differential mass element means the change in mass between adjacent subdivisions of the 5 meter strip of the x-axis. I have helped improve the navigability for finding out information on differentials, but I'm not certain, by reading the wikipedia articles what a differential mass element is. It could either be f(x+dx) - f(x) or it could be dx*f(x+dx) - dx*f(x) is what I've narrowed it down to on my own. Thanks Sentriclecub (talk) 10:11, 22 August 2008 (UTC)[reply]

An engineer would say (mathematicians look away now) that the differential mass element dm is the mass of an infintesimally thin slice of the body between x and x+dx (a mathematician would talk about the "mass distribution function", but would get to the same result). So, for example, for a thin triangle with mass per unit area ρ between the x axis, the line y=x and the line x=a,
because the area of the thin slice is xdx. To find the distance of the centre of mass from the y axis you divide the first moment of mass about the y axis by the total mass of the body, so you have:
Gandalf61 (talk) 10:56, 22 August 2008 (UTC)[reply]
Thank you alot, Gandalf, for your help. I understand a lot better using the language of infinitesimally thin slices than my book's language. (I learned calculus from a Stewart book, so its inconsistent with my physics book which is more pragmatic at how it throws around symbols). Thanks again. Sentriclecub (talk) 11:05, 22 August 2008 (UTC)[reply]
No problem. I have a mathematics background, but my son is a student engineer, so I am quite used to doing this type of "tranlsation" ! Gandalf61 (talk) 11:13, 22 August 2008 (UTC)[reply]
In my experience, mathematicians would say it in the same way, they just wouldn't dare write it down! As long as you understand the real explanation well enough to know that you can trust the hand waving argument, it's so much easier just to wave your hands around. --Tango (talk) 17:35, 22 August 2008 (UTC)[reply]

Tornados at night?

Do tornados occur at night as frequently as they do during day?Leif edling (talk) 10:49, 22 August 2008 (UTC)[reply]

No, but not for an entirely clear set of reasons. The biggest reason is that a warm front collides with a cold front, forming cells is the initiation of what can lead to a tornado. Pressure systems you could say are propelled by differences in air pressure between colliding "blocks" of air. The sunlight heats these blocks of air, and increases the likelihood of discrepancies between contiguous blocks of air. When the sun is out, these "moving blocks of air" are more likely to collide, and at higher kinetic energies, thus leading to increased likelihoods of the prerequisite conditions. As an analogy, pretend white and black mice are playing in a tank, moving at random. Now randomly sweep a lit cigarette around in the tank. This stirs up the mice and makes them even more likely to wind up one-atop-another. Sun feeds the weather system energy, and it makes things more volatile and chance climate events more probable. Sentriclecub (talk) 11:01, 22 August 2008 (UTC)[reply]
I've been living in the northern part of Texas - slap bang inside "Tornado Alley" - for the last 15 years and I've gotta say that the only tornadoes we've heard about have happened in springtime - right around dusk. Between (say) 5pm and 10pm. I can't recall a single one happening outside that range. In terms of a warm front colliding with a cold front and a 'wall cloud' forming (the predecessor to a tornado)- it kinda makes sense that air that's been heated by the ferocious sunshine around these parts for the entire day might collide with colder air blown in from areas where the sun has already set and the air is cooling down. That would increase the prevelance around dusk. In the height of summer, nighttime temperatures are pretty similar to daytime temperatures because the ground has heated up and is acting as a reservoir of heat - so I guess that explains why tornadoes are not so common in the height of summer around here.
I've actually driven right underneath a tornado that was just forming and which touched down less than a mile away. It was AMAZING. All of the stories you hear and TV/Movies you see can't prepare you for the ferocity of the real thing. My car was a super-lightweight MINI Cooper - and it was a convertible. I was thinking that this was possibly the first time I really wish I'd bought a Hummer!
SteveBaker (talk) 18:56, 22 August 2008 (UTC)[reply]

Apple

Why does an apple turn brown few minutes after when it cut? —Preceding unsigned comment added by Kelvin caesar (talkcontribs) 12:12, 22 August 2008 (UTC)[reply]

According to our article,

Sliced apples turn brown with exposure to air due to the conversion of natural phenolic substances into melanin upon exposure to oxygen. Different cultivars differ in their propensity to brown after slicing. Sliced fruit can be treated with acidulated water to prevent this effect.

Algebraist 12:20, 22 August 2008 (UTC)[reply]
And if you meant Apple I, it was already brown before they cut it. Nimur (talk) 15:15, 24 August 2008 (UTC)[reply]

Is it right that accidentally snorting msg can cause bleeding from nose and eyes?

This question has been removed. Per the reference desk guidelines, the reference desk is not an appropriate place to request medical, legal or other professional advice, including any kind of medical diagnosis or prognosis, or treatment recommendations. For such advice, please see a qualified professional. --EronTalk 13:34, 22 August 2008 (UTC)[reply]

Meta-discussion has been moved to the talk page. Please confine comments about the operation of the Reference Desk to Wikipedia talk:Reference desk#Accidental snorting question. Please refer to the discussion there before adding to this section. Your assistance is appreciated. TenOfAllTrades(talk) 01:39, 23 August 2008 (UTC) [reply]


drug reaction

This question has been removed. Per the reference desk guidelines, the reference desk is not an appropriate place to request medical, legal or other professional advice, including any kind of medical diagnosis, prognosis, or treatment recommendations. For such advice, please see a qualified professional. If you don't believe this is such a request, please explain what you meant to ask, either here or on the Reference Desk's talk page.
This question has been removed. Per the reference desk guidelines, the reference desk is not an appropriate place to request medical, legal or other professional advice, including any kind of medical diagnosis or prognosis, or treatment recommendations. For such advice, please see a qualified professional. If you don't believe this is such a request, please explain what you meant to ask, either here or on the Reference Desk's talk page. --~~~~
-- Coneslayer (talk) 13:54, 22 August 2008 (UTC)[reply]

Olympics

What olympic sports can I still do when heavily pregnant?

Has anyone done this?? —Preceding unsigned comment added by 86.128.218.142 (talk) 16:41, 22 August 2008 (UTC)[reply]

I don't see any reason why you couldn't participate in some of the shooting events (unless the IOC has some rule against it). I'm pretty sure nobody has ever done it before. (Somewhere out there, there's an advertising executive drooling over the idea.) Clarityfiend (talk) 17:17, 22 August 2008 (UTC)[reply]
Being heavily pregnant pretty much is guaranteed to deteriorate your performance at any physical sport. I imagine even something like shooting, which requires a variety of physical aspects to be really great (concentration, control over breathing, etc.) would even take a small hit from the changed physiology and additional burdens of pregnancy. You don't need much to push you out of the runnings alltogether. --98.217.8.46 (talk) 18:18, 22 August 2008 (UTC)[reply]
If the question is really "What can I still do" - and not "What can I possibly be competitive at" - then there are a lot more options. If you're just asking what events you can complete safely - then the possibilities are wide open. You can (for example) still swim - so you could do short distance swimming events. Some things (like maybe the 'luge' event in the winter olympics) are possible - but totally not safe for the baby. But in terms of being competitive - I agree that you're probably limited to shooting and archery, which require less physical effort than most olympic sports - but as '98 says - it's doubtful that you could attain the breathing control needed to win such an event. In the past there have been other events like Ballooning, Boules, Gliding and Motorsports that would maybe have been within reach. In the winter games, you could probably participate in Curling too. SteveBaker (talk) 18:38, 22 August 2008 (UTC)[reply]
The advice here has been from a sports performance perspective; you should see your doctor (we cannot give medical advice here) for medical advice if you actually do intend to engage in any competitive sports while pregnant rather than this being a theoretical question. --Random832 (contribs) 18:57, 22 August 2008 (UTC)[reply]
A search on "olympics pregnant" turned up this article: [3] -- BenRG (talk) 19:14, 22 August 2008 (UTC)[reply]
You could be the cox on a rowing team without too much problem. The rest of the team may not thank you for the extra weight though. Dostioffski (talk) 00:24, 23 August 2008 (UTC)[reply]
It's vital that you avoid beach volleyball though - trust me - just don't go there! SteveBaker (talk) 03:49, 23 August 2008 (UTC)[reply]
Why not? Nil Einne (talk) 05:18, 23 August 2008 (UTC)[reply]
maieusiophobia/maieuticophobia (What?!? No article on that?! Not even a wiktionary entry? I'm shocked! Deeply shocked.) SteveBaker (talk) 17:52, 23 August 2008 (UTC)[reply]
I just had a thought. Why not try out for the 4x100 team relay semifinal? There's a fair chance everyone else will drop the baton so you may still make it to the final even if you're a bit slow. And once your team is in the final, you don't actually have to run to win Nil Einne (talk) 17:21, 23 August 2008 (UTC)[reply]
Continuing the humour, I think sailing and equestrian might be suitable as well, though there might be a bit too much up and down movement. In softball, you can just be the pitcher to avoid running. Table tennis also does not require too much running around. --Kvasir (talk) 22:24, 26 August 2008 (UTC)[reply]

Glass figure changing color

I know someone who had a glass trinket that was originally purple, and then one day looked bright blue, and then the next day turned purple again. The person in question was adamant about the fact that the color changes were real and not just a matter of lighting or confusion. I haven't seen the thing myself. (And I don't actually know the person. It's a friend of a friend, the latter of which came to me.) What do you think? --98.217.8.46 (talk) 18:16, 22 August 2008 (UTC)[reply]

People are very poor at detecting lighting changes. Our eyes continually adapt to try to maintain visual consistency no matter how bright or dim the lighting is - and to correct for color variation (eg at dawn and dusk). So I strongly suspect that it really was lighting changes - but that the person was unaware of that change. Failing that - I don't think we have enough information to go on. SteveBaker (talk) 18:42, 22 August 2008 (UTC)[reply]
Moreover, human color vision is especially poor at the blue–purple end of the spectrum. You may be interested in the Color vision article. Strad (talk) 19:17, 22 August 2008 (UTC)[reply]
Lighting changes could be the cause, but not for the reasons you suggest. If a glass has more than one optical absorption band in the visible region then natural or synthetic lighting could cetainly induce different pigmentations of the glass, as per my explanation below. Jdrewitt (talk) 20:05, 22 August 2008 (UTC)[reply]
There are a number of salts that change color based on their hydration state (one example is cobalt chloride). Changes in humidity could conceivably affect the color of such a item. Another potential explanation is photochromism, where changes in lighting level affect the actual (not just apparent) light absorption profile of an object. However, the most likely explanation is probably still perceptual differences, especially if the color change is not reproducible. -- 128.104.112.147 (talk) 19:19, 22 August 2008 (UTC)[reply]
It might be blurple, especially if the day in question was January 1, 2000. -- BenRG (talk) 19:27, 22 August 2008 (UTC)[reply]
See Alexandrite#Alexandrite. By the way, is the article Ratnagarbhas#Medicinal_qualities_of_gems_administered_in_Bhasma_.28ash.29_or_powder_form I stumbled upon giving medical advice? 93.132.170.130 (talk) 19:45, 22 August 2008 (UTC)[reply]
It may be some variant of a "mood ring". There are "Mood Necklaces" and many of the ones I can find on the internet seem to be somewhere between blue and purple when they were photographed. Since the color of "mood" jewelry is almost entirely based ambient temperature, perhaps the day it was bright blue was a really hot day. Or a really cold one. APL (talk) 19:47, 22 August 2008 (UTC)[reply]

Colour changes can certainly occur in glasses. Glasses doped with the rare earth ion Holmium for example appear bright red under flourescent lights and pale yellow under natural light. I made some rare earth aluminosilicate glasses (for scientific purposes) and was amazed how pronounced this effect actually is. It is similar to the effect in the Alexandrite mineral and is the result of different optical absorption bands in visible light. People who make coloured glass for ornamental / art purposes are cetainly aware of the different compounds to use to produce different pigments (see Glass colors and it is not inconceivable that whoever made your glass trinket incorporated some colour changing ion into the glass composition. Jdrewitt (talk) 20:00, 22 August 2008 (UTC)[reply]

Here is an image of the HoAlSiO glass that I made, the glass is yellow under natural light and red under flourescent light!
HoAlSiO Glass
Jdrewitt (talk) 20:21, 22 August 2008 (UTC)[reply]
Follow up: So I forwarded these possibilities on to the person, and they realized that indeed, they had installed fluorescent lights or something like that in the area where it was. In the daytime, it was purple, at night, with the lights, it was blue. Crazy stuff. --98.217.8.46 (talk) 15:38, 23 August 2008 (UTC)[reply]
Yep - so much for "The person in question was adamant...not just a matter of lighting". It pays to be skeptical! SteveBaker (talk) 17:43, 23 August 2008 (UTC)[reply]
I think that's a bit harsh, I think the OP meant it was not their perception which was at fault which I would tend to agree with. Jdrewitt (talk) 19:26, 23 August 2008 (UTC)[reply]
Well, I didn't take any opinion on whether it was lighting or not. I don't know this person, I was just reporting what they said. Truth be told, I think the fact that narrow wavelength light sources can radically affect what certain materials look like would strike a lot of people as rather surprising. From a scientific point of view it makes perfect sense—once I heard that about the light bulb, it's obvious what's going on with it, to anyone who has taken any sort of physics or astronomy course that dealt with spectrums—but the notion that two different light bulbs could radically change the color of something is less intuitive. Even explaining to someone why it's the case takes a little trip down "abstract concept of what colors are" lane. --98.217.8.46 (talk) 20:10, 23 August 2008 (UTC)[reply]
Sorry - I didn't mean to imply that the OP was unreliable - but when someone says something offhand to a friend and the friend asks you and you ask us - there have been three word-of-mouth tellings of the information - and it's no surprise that odd bits of crucial information become missing or distorted. SteveBaker (talk) 20:52, 23 August 2008 (UTC)[reply]
Agreed 100%. I tried to indicate my agnosticism of their account accordingly—was just fishing for possibilities. --98.217.8.46 (talk) 14:18, 26 August 2008 (UTC)[reply]

On the Topic of Color Changes

Some color prints have been hanging in our local coffee shop for years, exposed to a lot of sky light and artificial light. Not direct sun.

Originally brightly colored, they have changed. The lightest areas have stayed light, the blacks are still black, everything else is in various shades of blue.

How can this phenomenon be explained in terms of chromophores, which were mentioned recently in this page?

Thanks, Wanderer57 (talk) 20:57, 22 August 2008 (UTC)[reply]

On reflection, various intensities of blue is more accurate than various shades. Wanderer57 (talk) 21:45, 22 August 2008 (UTC)[reply]
Well, many color printing processes use CMYK (Cyan, Magenta, Yellow, Black) inks - if the magenta and yellow ink faded in the sunlight at a faster rate than the cyan (blue) and black - then you'd get the exact effect you're seeing. SteveBaker (talk) 03:44, 23 August 2008 (UTC)[reply]
That's precisely it. The black pigment (usually being based on carbon particles) is very durable. The cyan dye or pigment is either more durable or less subject to attack by the incoming spectrum of light that it absorbs. But magenta and yellow are easily destriyed, probably by the incoming ultraviolet light.
Atlant (talk) 13:35, 23 August 2008 (UTC)[reply]
Yep. My thinking exactly. A possible explanation for why cyan (Blue) ink survives the longest is because (by definition) blue objects reflect light at the blue end of the spectrum. (ie at the higher frequencies) Magenta and yellow inks absorb blue light (that's why they don't look blue). Since ultraviolet frequencies in the sunlight are the cause of fading and UV light is at the high frequency end of the spectrum, blue dyes that reflect high frequency light are likely to be more fade-resistant than those that absorb it. Hence the cyan ink survives longer than the magenta and yellow. Black ink doesn't fade at all because it's based on carbon which absorbs all frequencies but doesn't react because of it. SteveBaker (talk) 17:41, 23 August 2008 (UTC)[reply]

Victorian women

I don't know if this question is better placed here, as medicine, or at the Humanities desk, as history. I'm starting here to offer editors a change of pace from black holes.

I'm quoting from a discussion elsewhere.

"Victorian women, through lack of exercise and general laziness, did have quite large posteriors."

Thinking particularly of lower class women in poorer parts of London, what do you think of this generalization?

Thanks, Wanderer57 (talk) 21:43, 22 August 2008 (UTC)[reply]

That clearly only applies to the upper classes, if at all. My understanding is that much of the "large posteriors" were petticoats. --Tango (talk) 23:12, 22 August 2008 (UTC)[reply]
It was the height of fashion to have a large bottom - many women wore a "Bustle"...which is a kind of wire contraption worn under the skirt to enhance the size of the hind end. It adds a whole new meaning to "Does this make my bottom look big?" SteveBaker (talk) 03:35, 23 August 2008 (UTC)[reply]
Yes, that's ass-backwards from the usual meaning. StuRat (talk) 02:00, 24 August 2008 (UTC)[reply]


August 23

The perfect murder

Is there any way to dispose of a human body, so that it cannot possibly be found? If the murderer hides or buries the body, evidence remains in the form of bones. If the murderer throws the body in the ocean, I assume that animals eat the remains ... but that still leaves the bones behind ... correct? Or no? If the murderer burns the body, evidence remains in the form of bones, teeth, ashes, etc. Is there any completely fail-safe way to dispose of a human body ... such that no evidence at all is left behind? Thank you. (Joseph A. Spadaro (talk) 04:02, 23 August 2008 (UTC))[reply]

If you do it right, burning ought to work. The bits left behind will be small enough that you can scatter them at sea or something and be confident no-one will find them. Of course, I would strongly advise against committing murder, for the obvious reasons. --Tango (talk) 04:30, 23 August 2008 (UTC)[reply]
Isn't there a policy against giving legal advice? 93.132.139.138 (talk) 07:38, 23 August 2008 (UTC)[reply]
Baaah, cremation is for amateurs. If you really want to get rid of a body, then how about a volcano? At 1650 C or so even bone will melt, plus you have the advantage that even if there were any remains they would be encased in rock. Dragons flight (talk) 05:03, 23 August 2008 (UTC)[reply]
Why don't you just launch a spaceship and take the body to the sun? Or even better, a black hole? Nil Einne (talk) 05:14, 23 August 2008 (UTC)[reply]
Burning can reduce the bones to ash but it requires high temperatures and the ability to burn a body without anybody noticing. In the past people have been convicted for murder because of bad burning techniques—in one case I remember (from a forensic anthropology class I took years ago), a single vertebrae and a single tooth were all that didn't get fully burnt, but the combination of the two was enough to convict (tooth proved identity, vertebrae proved murder).
If you had a large supply of certain acids you could probably use those. Certain chemicals cause bodies and bones to decompose (e.g. lye) but they take time, space, hazardous supplies. I'd probably go with a bathtub full of lye, personally—a lot easier than dealing with a fire.
Note that getting rid of a body does not necessarily get you off the hook for murder. In some jurisdictions you do not need a body to prove murder. My father (a defense lawyer) represented a client who was charged with murder on circumstantial and forensic evidence (e.g. blood at house, etc.) in California where there was no body for the victim found. (In the end, they found the body—in Mexico—and it was shipped back to the US. Except the Mexican authorities removed and lost the brain. Moral of the story: don't get an autopsy in Mexico.) --98.217.8.46 (talk) 06:05, 23 August 2008 (UTC)[reply]
This question has been asked before, so have a look in the archives. There is no shortage of bodies that were never found (see our list of people who have disappeared), many of them at sea, so maybe take your victim on a cruise.--Shantavira|feed me 07:08, 23 August 2008 (UTC)[reply]
The problem with dumping bodies at sea is that the sea can be very unpredictable. Sometimes they'll be gone forever. Sometimes they wash up on a tourist beach the next day. If it were me—no, not the ocean. Maybe deep, deep ocean, far from shore is a bit better but even then I'd be wary (who knows if there's no some tiny, expensive island within a few miles that I don't know about, and currents run towards it?). --98.217.8.46 (talk) 13:49, 23 August 2008 (UTC)[reply]
With sufficient weights attached the currents become irrelevant, the body will go almost straight down. And even bones decompose completely in the ocean in short order. However, the weights themselves become evidence, as in the case of Chilean dictator Pinochet, who liked to dump bodies in the ocean by helicopter, attached to sections of railroad rails to weigh them down. The investigators found the ties, and one had a shirt button encrusted on it, which, along with the testimony of those who dumped the bodies, is pretty conclusive evidence. StuRat (talk) 00:56, 24 August 2008 (UTC)[reply]
If the shoes are left attached, after sufficient decay the buoyancy of a running shoe can allow it to float up to the surface, with the foot in it. These turn up sometimes on shore. There's been a few feet showing up in the Gulf Islands in Canada, all left feet. Apparently currents carry left and right shoes differently, so they end up in different places. --Joelmills (talk) 01:33, 24 August 2008 (UTC)[reply]
There is some considerable debate about the left-shoe/right-shoe thing. The original evidence came from professional beach-combers who found that thousands of shoes that got washed overboard in a shipping container ended up with all of the left shoes on one beach and all of the right ones thousands of miles away. However, it's since been discovered that many shoe manufacturers ship left and right shoes in separate containers in order to dissuade would-be thieves from hijacking truckloads of shoes and selling them on the black market. That effectively busts the original theory that (empty) running shoes are somehow sorted by the ocean currents due to their shape. That in turn makes it vastly less likely that only left feet of corpses show up in Canada...the numbers in which they get washed up must be tiny - and a statistical flook or an urban legend based on the beachcomber story for empty shoes is a more likely explanation. SteveBaker (talk) 04:37, 24 August 2008 (UTC)[reply]
Woodchipper exiting into pigpen after removing the teeth and hair (dissolve those in acid)? --Kurt Shaped Box (talk) 12:50, 23 August 2008 (UTC)[reply]
Yeah, unless Margie comes along and notices you doing it. Personally, after I read a book named Death to Dust; What happens to Dead Bodies? (ISBN 1-883620-07-4), I became convinced that no matter how hard one might try, you'll always leave some sort of forensic evidence behind. Well, unless you can transport the vic into the heart of the sun or something.
Atlant (talk) 13:31, 23 August 2008 (UTC)[reply]
In that case, I suppose that it's equally as important when committing a murder that the suspicion should never be allowed fall upon you in the first place. If the police never come to your door, then they'll never have the opportunity to find even the smallest piece of forensic evidence indicating your prior body-disposal activities. So, the crime would have to be meticulously preplanned and the victim a perfect stranger with no connection to yourself whatsoever. Which would kinda defeat the object of the majority of premeditated murders (i.e. vengeance against someone who has seriously upset you). --Kurt Shaped Box (talk) 13:46, 23 August 2008 (UTC)[reply]
(ec) Well, but again, forensic evidence is only one part of "a perfect crime". And it should be noted that there are thousands of unsolved murders per year. We only spend a lot of time talking about the times when our "brilliant police forces" manage to solve them using some clever scientific means. There are nearly as many cold cases as there are solved ones. --98.217.8.46 (talk) 13:47, 23 August 2008 (UTC)[reply]

Follow up

Thanks for the input above. Let me ask the question this way. Say that you have a tiny dead human body (infant / baby / small child) and there are no clothes (naked). If you fed that body to, say, an alligator (or bear or whatever) ... would there be any evidence whatsoever? Assume that from the point of murder to the point of feeding the alligator (bear, whatever) ... the murder was perfect and undetectable. Once the murderer leaves the point of feeding the alligator or bear, is there any incriminating evidence whatsoever ... or can the murderer be assured that detection is impossible at this point? Thanks. (Joseph A. Spadaro (talk) 14:53, 23 August 2008 (UTC))[reply]

It's possible that the animal could 'pass through' some teeth or identifiable bone fragments. I believe that's how they caught the guy who was feeding dead prostitutes to the pigs a few years back... --Kurt Shaped Box (talk) 14:56, 23 August 2008 (UTC)[reply]
As I discussed above in the burning case, you only need a very small amount of very hard bone to have damning evidence of a murder and identify who it was. Teeth and vertebrae are numerous, small, and dense and I would not be surprised if a number of them survived in the stool of the animals. A tooth can give you identification, a vertebrae can prove murder. It's not how I would do things, is all I'm saying, and you're also relying a little too much on something you can't control—the animal and their individual physiology and health. What if your alligator runs away and is promptly killed by someone for its hide, who then happens to discover a stomach full of corpse? Too many possible ways for something like that to go wrong... IMO. But I'm speaking from the point of view of whether it was my future that was on the line in such a thing—I'd want to be pretty dang sure of things, I wouldn't be taking chances with luck. And note that most people are going to notice a missing baby—that's going to be as problematic as forensic evidence, because the number of people who would want to kill said baby are few. --98.217.8.46 (talk) 15:11, 23 August 2008 (UTC)[reply]
You probably don't need to completely erase all evidence of the body - you only need to hide it with a sufficiently good probability of it not being found and identified in your lifetime. You could (for example) take a small aircraft over the amazon rainforest and push the body out of the door somewhere at random. So long as no evidence remains of where you flew, the authorities would have to search a billion acres of forest that's so dense that you could walk within a few feet of a body and not find it. Decay would take a while - but investigators combing the forest for human remains would find lots of human remains from people who live there - and the effort to do DNA studies on all of them would be all but impossible. The issue is then one of having a plausible reason to be out there - and whether you can fly far enough from radar and other aircraft who might record your position. I don't think that's too difficult.
Other strategies that might help (but could be risky) would be to arrange that nobody noticed the victim was no longer present. "My wife is terribly sick - the doctor recommends bed rest"...then one or two days later..."She's been hospitalized for specialist treatment at (some hospital 200 miles away so nobody will try to visit)"...then a couple of weeks later..."She's much better - but she's been under a lot of stress. I sent her to Switzerland to relax and recover in the mountain air"...then a few months later..."OMG! She's met another man and is being unfaithful to me"...then six months later..."We've decided not to go through a messy divorce - we're separating"...then a few years later..."I heard she died in a car crash - I don't have many details...I'm devastated". You could fake emails to her close friends. If you did it right - nobody would ever think to call the cops and the issue of searching for a body would never come up.
SteveBaker (talk) 17:32, 23 August 2008 (UTC)[reply]
Right, but if it were truly I, and not just some character in a story I was writing, I wouldn't want to trust chance on that sort of thing. What happens if the dead person's sister really needs to talk to said person because of some family emergency? Oh, you try to put it off, and redirect, and pretty soon it becomes clear that you're the one standing in the way of contact, and people get worried, and etc. It's not that easy to remove a person out of their network of people and society if they have any ties to it—people notice that sort of thing pretty quickly, we're more interconnected with one another than we often think. And something like the Amazon seems foolproof, except when some hikers that you can't see in the plane end up seeing you, or you end up being on some sort of surveillance camera at the airport that you can't see, and etc. Again, a lot of jurisdictions can convict you of murder on purely circumstantial evidence, and some without an actual body.
If I were to speculate, I would say the "perfect murder" is one where 1. it is clear to all that there was a murder (e.g. body is found quick), but 2. the forensic evidence is not incriminating to the killer, and 3. there are multiple plausible possible shooters. Hence a drive-by gang murder is much more likely to go unsolved than a suburban white guy killing his wife. Drive by a gang member's house in a stolen car, shoot them with a generic (stolen) gun, drive off, throw the guns in the ocean, set the car on fire, get the hell out of there. The body is in plain sight—and you don't possibly incriminate yourself by being caught with remains or anything—but the question of who did it could be totally up in the air (especially if you live in an area where witnesses are unlikely to collaborate with the police out of fear of recriminations). But if you're some uptight white guy and you kill your wife and kids—you're going to get caught almost surely, because you're probably the only suspect from the minute anyone notices they're missing. The best way to get rid of your wife and kid (if that's what you want to do, you sick bastard) is to make it look like they died in an accident, in my opinion. Nobody's going to believe that some sicko decided to target your wife and kids randomly unless there is hard evidence for that, because that sort of stuff is rare, whereas family-on-family violence is remarkably likely. But a carbon monoxide leak, a gas leak, a hit-and-run car accident, etc.—these sorts of things actually do occur, and if you were clever and careful about that, well, you could get away with it, maybe. So I guess what I'm trying to suggest again is that the idea of a "perfect murder" being preoccupied with disposing of the body is short-sighted. Most real unsolved, "perfect" murders involve bodies that are right there in the open. Only in bad detective novels do they involve bizarre arrangements of hiding a body and then covering it up. --98.217.8.46 (talk) 20:01, 23 August 2008 (UTC)[reply]
You people should collaborate in writing murder mysteries, or perhaps CSI episodes. Wanderer57 (talk) 17:43, 23 August 2008 (UTC)[reply]
The problem with writing murder mysteries is not the mystery or the murder - but the writing. Writing a novel is difficult for a million reasons totally unrelated to finding a plot. SteveBaker (talk) 20:50, 23 August 2008 (UTC)[reply]
Fact is, none of this is really all that novel. If you watch The Wire you'll see my point of view written plain as day. If you read a forensic anthropology book you'll see tons of discussions about dissolving bodies and feeding them to animals and things like that. As I mentioned before, I took an undergrad class in forensic anthropology and like 90% of what we discussed was how "perfect murders" that try to get rid of bodies usually go awry. In my opinion/experience most "perfect murders" are pretty boring. One teen shoots another teen and nobody talks to the cops about it and they never find the gun or maybe they have a good idea of who it might have been but they don't have enough evidence to link them to the scene of the crime, and oh well, case goes into the cold file bin, and nobody cares that much unless it's some white college kid or a pregnant white lady. --98.217.8.46 (talk) 20:05, 23 August 2008 (UTC)[reply]
Doing it right is tbe big thing and it's going to depend a lot on your and her circumstances (and your ability to lie). If for example your wife has family/close friends who don't particularly trust you, they might get suspicious from the moment you tell them she's on bed rest, but no she doesn't want you to visit or at least call. Even more so when she didn't contact any of them other then by e-mail. Indeed, they may be suspicious from the moment you call them and tell them she's going but no, neither of you need help or if you don't call them, the fact she disappeared so suddenly. Even your friends/family may be suspicious... Even if they don't get suspicious at the early stage, there's a good chance they're going to get suspicious once the 'she ran away' comes up or perhaps the 'she died' (well we lost contact but I still think I should go to her funeral/grave, can you give me the details you have?). And when they think about it and realise they haven't spoken to her since she mysterious went on bed rest. The fake e-mails too could give you away if you say or don't say something which makes someone suspicious. You'd need to be smart about it, and not send your e-mails from a UK IP if she's in Switzerland in case anyone notices. (Then there's also the question of "wait you could afford to send her to Switzerland? if you couldn't) Also if there's stuff in her name your going to either have to hope no one starts noticed you didn't change things into (solely) your name. Even worse if there's debts in her name and someone asks you 'why the fick are you paying off your (dead) cheating wife's debts' or later if there is insurance or some other death benefit 'why the fick didn't you collect the death benefit?' One key point of course here is it can easily fall apart the moment someone gets suspicious or even does something without asking you, e.g. when someone tries to contact this hospital/specialist... or even has circumstances which you don't expect (she's in Hospital X? What a coincidence, my cousin works in hospital X, I'll get him/her to see how she's going) Nil Einne (talk) 21:25, 23 August 2008 (UTC)[reply]
Upfield thought the perfect way to dispose of a body is to burn the body, dissolve the remains, bury any solid remnants, and spread the ashes into wind. When Rowls actually tried it, the police still found the victim's remains, partially because they were aware of Upfield's method. Moral of the story: don't use any disposal method mentioned on Wikipedia. --Bowlhover (talk) 20:13, 24 August 2008 (UTC)[reply]
Which would be the more difficult task though? Disposing physical evidence of the body, weapon, etc, or actually trying to lie your way through others convincing them that you had nothing to do with the murder. If a random joe is living with his wife, and like said above, the next day she's dead, most people I would assume would jump to the conclusion that random joe had something to do with it. Now of course, making it seem an accident would have more weight. Random joe could take his wife for a ride in his car out to the highway. Make sure she's on a tight seat belt, ride full speed into a tree and jump out at the right time to avoid yourself injury, hope she dies in the crash, perhaps the car may not be damaged enough that you could get back into the drivers seat, give yourself some cuts from glass. Then again, police might blame bad driving, etc and it was in fact intentional.--GTPoompt(talk) 18:39, 25 August 2008 (UTC)[reply]

Cooling soups

How much should a newly cooked bowl of soup be opened at the top to optimize the two factors 1) opening not enough will make the soup not cool fast enough --> soup stands too long at around room temperature --> bad soup; and 2) opening too much will let microorganisms in --> bad soup. Assume the soup will be refrigerated when at room temperature and there happens no thunderstorm (bonus question!). --Ayacop (talk) 08:11, 23 August 2008 (UTC)[reply]

See Food safety for links to the various official advice; usually food should not be left out for more than one or two hours total regardless of temperature. This advice would not change if you put a cover over the bowl of soup.--64.228.91.86 (talk) 15:29, 23 August 2008 (UTC)[reply]
It's a bit more complicated than that. If your soup is maintained at over 70 degC for at least 20 minutes or is raised over 100 degC for some shorter period - then the bacteria are all dead (see Pasteurization for more exact details). If you then immediately tightly cover it so that no more bacteria can get into it - then it'll last essentially forever. We don't worry about cans of soup going bad for example - people have opened canned food that's over 100 years old - and the contents are still safe to consume (cans older than that used poisonous lead solder - so while there was no bacteria, it's not so good to eat!).
But some soups don't taste too good after they've been heated too much...many canned soups tell you right there on the label: "Do not boil". Here's the problem: If you merely WARM the soup - then rather than killing harmful bacteria, you've actually put them into a situation where there is plenty of nice warm nutrients - kinda like being in the human body in fact - and they'll reproduce like crazy. Covering the warm soup will have little effect because you're getting more and more bacteria building up in there. If you wait too long, it'll go bad whether you cover it or not. If you take soup that's been heated enough to kill all of the bacteria - and then let it cool gradually without being tightly covered - then more bacteria will get into it - and again, they'll find a warm environment that lets them reproduce.
Refrigerating the soup will help because it gets the stuff down to such a low temperature that the bacteria can only reproduce extremely slowly - this means it'll keep for quite a while before there is too much bacteria in it.
Freezing the stuff will stop bacterial reproduction - but won't kill them. So as soon as you warm it up again, they'll start reproducing. So freezing freshly made soup will 'stop the clock' until you un-freeze it.
SteveBaker (talk) 17:00, 23 August 2008 (UTC)[reply]
If you're going to refrigerated the soup, there's no reason to wait for it to reach room temperature. If you're afraid it may melt something, then let it cool down a bit but this definitely doesn't have to be room temperature. Also, I wouldn't bother to open the soup. Not so much because of microorganisms/spores per se, although it may help depending on how well you cover, but to stop things like flies going in. Nil Einne (talk) 20:48, 23 August 2008 (UTC)[reply]
  • Clostridium botulinum spores survive 100°C and then they grow in the oxygen free and competator free enviroment! So you have to heat the soup a second time after a day to get them after they left their spore! But if you are not workin in a clean room the first mold is in your food seconds after it droped beneath the temperature they can survive.--Stone (talk) 22:18, 23 August 2008 (UTC)[reply]
  • Of course, it's not just whether the soup is sterile or not, as how much bacteria is present initially also impacts how long it will take to spoil. One method I use in the winter (which is the only time I cook soup), is to put the hot covered soup outside to cool (I have an enclosed, but unheated, porch I use for this). This allows it to cool quickly but remain covered. I then move it to the refrigerator once Wings don't matter much because (as you say) plenty of jet fighters can accelerate straight upwards. it cools off, or, if the fridge is full and the temp is low enough, I might just leave it outside. If you don't have an enclosed porch, a garage might also work, but don't leave soup where critters can get at it. StuRat (talk) 00:13, 24 August 2008 (UTC)[reply]
It's certainly true that bugs like botulism can survive pasteurization - but we generally work very hard to make sure there is no botulism in our food to start with. SteveBaker (talk) 04:30, 24 August 2008 (UTC)[reply]
In regard to the bonus question, the absence of thunderstorms will be heavily associated with specific weather patterns, probably correlated to high humidity and uniform air pressure with a lack of updrafting. This will reduce the probability of airborne electrical discharge and the consequential ozone generation and broadband impulsive electromagnetic signals (not to mention light flashes). The absence of thunder may cause the human participants in the soup-cooling experiment to maintain concentration slightly better than in other conditions, enabling more repeatable experiment (I assume you will be following sound scientific method with control experiment and so will be cooling many dozen soups). In thunderstorms, the possibility of electric outage and the likelihood of distractions due to loud thunder would potentially interfere with the experiment's repeatability. Nimur (talk) 15:30, 24 August 2008 (UTC)[reply]
My great grandmother always claimed that thunderstorms made milk go sour. After a storm she'd chuck all her milk down the sink. I have no CLUE where that theory came from - I'm pretty sure it's untrue. SteveBaker (talk) 04:54, 25 August 2008 (UTC)[reply]

Ortho, Meta and Para

According to the Online Etymologies Dictionary para means alongside [4], ortho means straight [5] and meta means in the middle [6]

In Chemistry para can be said to mean "straight", ortho "alongside" and meta in the middle: Arene substitution patterns

I imagine that this is due to the archaic nature of the naming convention and when they came up with it they may not have known the structures of the compounds.. is this true? Did they simply think that ortho compounds were 1,2 substituted and para 1,4 substituted? Thanks, --220.237.34.141 (talk) 12:34, 23 August 2008 (UTC)[reply]

Those prefixes are used in all sorts of different ways - but usually consistently within a single field. I work in computer graphics - we use "ortho" (for example) in the word "orthogonal" to mean (roughly) "at right angles to". In many sciences "meta" is used in phrases like "meta-study" (which is a study of other studies) or "meta-theory" (which is a theory about other theories) - it's hard to translate that into normal English but "meta-X" means "a kind of X that refers to other X". "para" is sometimes also used to mean "two". The problem is that science often "adopts" words and gives them different meanings to those used in general English. The word "Accelleration" means "going faster" in regular English - but in scientific usage, it can also mean "changing direction" or "slowing down". In common English "velocity" and "speed" are synonyms - but in scientific usage, "velocity" describes both the speed AND the direction of motion. So looking up a scientific term in a general dictionary will often produce contradictory results. SteveBaker (talk) 16:36, 23 August 2008 (UTC)[reply]
acceleration, a change in velocity :D Coolotter88 (talk) 20:15, 23 August 2008 (UTC)[reply]
I will have a look into the publications of Kekule. The structure of the compounds was unkown for a long time. Kolbe was never able to accept that anybody arranged the atoms in a molecule to a two or three dimensional formula. He stuck to the radicals and that they are the smallest things in the molecule, but because of the fact that nobody has ever seen a molecule, and he thought nobody ever would he denied the Kekule benzene formula. But even before the hexagonal formula was born the chemists found out that there are three disubstituted benzenes possible. For example 1,2- , 1,3- and 1,4-dichloro benzene. This made a lot of the other predicted structures of benzene (prismane, Dewar benzene) impossible. I do not know what the name of the three possible substances was, but I have to have a look in to the literature of 1860s to give you the answer you want. --Stone (talk) 22:10, 23 August 2008 (UTC)[reply]

fluocinonide 0.05%

I'm looking at a tube of this stuff (prescribed for poison ivy, also eczema I think) and the expiration date didn't quite print off on the label. Anyone know what kind of shelf life this stuff has? --MrE1 (talk) 15:33, 23 August 2008 (UTC)[reply]

Your best bet would be to call the pharmacy where you picked up the prescription. (You don't know how long it was on their shelf before you got it, remember; they may also want to counsel you on the proper storage and use of the product.) TenOfAllTrades(talk) 15:52, 23 August 2008 (UTC)[reply]
There are some ways to tell that a medication has gone bad, such as if it seperates, dries out, changes consistency, becomes cloudy, etc. If any of this has happened, I'd avoid using it. However, it doesn't necessarily mean it's good, if none of this has happened yet. StuRat (talk) 00:45, 24 August 2008 (UTC)[reply]

Homeopathy

Dears

I'd like to underestand the difference between Homeopathy and the other doctors. what the Homeopathy do?

Best regards Mustafa P.S. my address <email removed to prevent spam> —Preceding unsigned comment added by Mustafa363 (talkcontribs) 15:44, 23 August 2008 (UTC)[reply]

Have you read our article homeopathy? Algebraist 15:50, 23 August 2008 (UTC)[reply]
Homeopathy is the idea (which, it must be emphasised, has ZERO scientific backing) that a very small amount of a bad thing can cure you of that bad thing. To take a crude example - if someone has been poisoned by arsenic (say) - a homeopathic cure might entail taking the tiniest amount of arsenic and dissolving it in a gallon of water - shaking it up carefully. Then they'll take just one drop of that water and put it into a gallon of clean water and shake it up. Then take one drop of THAT water and put it into another gallon of clean water. You do this over and over again - with the arsenic getting more and more dilute. Homeopathic medicines are typically diluted to the point where there is almost no chance of even a single molecule of the original substance left in the water. Then (the homeopathists claim) you can have the arsenic poisoned person drink some of this water and they'll get better.
So - most homeopathic medicines that you can buy are nothing more than pure water. In scientific terms it's utter bullshit - there is simply no possible way these treatments can do what they claim. There are attractions to the manufacturers of these "medications" - they are very cheap to make - because they only consist of water, they don't have to undergo testing and medical trials - they can be sold over the counter and for some unthinkably bizarre reason, people believe in them.
To the extent that homeopathic treatments work, it can only be by the placebo mechanism. There is strong evidence that if people believe that they are being treated for something, they'll sometimes get better even if the treatment is a few drops of water or a sugar pill.
The "explanation" that the homeopathists give is that some trace of the original substance is somehow "imprinted" onto the water molecules and that this imprinting gets stronger the more dilute you make it. Since water is made up of polar molecules that form and reform continually - there is really no scientific possibility that this can be true. It's bullshit...pure and simple.
Worse still - if it worked - then small impurities in the original water (say a few atoms of copper that washed off the pipe through which the water flowed) would also get amplified by the same mechanism. If homeopathy worked then the "drug interactions" would be uncontrollable - as your body simultaneously tried to rid itself of copper, polythene (from the stirring mechanism), silica (from the glass vessels)...you name it.
So - it MIGHT work if you believe in it - but scientifically...no. If we were honest about using placebo's to treat diseases, we wouldn't be paying millions and millions of dollars to homeopathic medicine companies for tiny little bottles of water. The doctor could simply fill bottles with mineral water he bought in gallon quantities from WalMart. The whole thing is a nasty scam.
What bothers me most is that some drug stores here in the USA are putting homeopathic "cures" on the shelves right next to the real stuff - and the information that these are homeopathic is in teeny-tiny lettering. I was looking for some eye drops in Eccards only a week ago and it was only at the checkout that I noticed a tiny "HOMEOPATHIC" written on the packaging. I came very close to spending $8 for a bottle of water. That makes me angry beyond measure! Argh!
SteveBaker (talk) 16:22, 23 August 2008 (UTC)[reply]
"n" <-- The homeopathic edition of my ten-volume definitive treatise debunking the subject (available at fine bookstores everywhere for $39.95). Clarityfiend (talk) 17:44, 23 August 2008 (UTC)[reply]
The very basic idea at the heart of homeopathy is not so bad—that sometimes the best treatment for a symptom is more of the same, rather than an attempt at stopping it. For example, a homeopathic approach to a runny nose would not be to stuff it up with antihistamines, but to encourage the running. The theory here, basically, is that the symptoms one sees are actually the body's own defenses running their course—a runny nose flushes out bacteria in the sinus system and etc.—and that trying to counter the symptoms (i.e. plug up the runny nose) is actually counter-productive, because you will end up prolonging the underlying cause of the symptom. Now this doesn't work in all cases but it does work in some of them and it is what homeopathy was originally supposed to be about.
In a modern viewpoint, we might, if we chose to, see things like vaccinations as part of a similar mindset—giving the body weak form a disease can help the body to develop means of combating the disease, which helps it later if you do actually get exposed to a full version of the disease.
Unfortunately, what happened in the 19th century is that this very simple and not totally crazy idea got taken to very stupid extremes. The idea became to give the body very weak exposure to all sorts of things—so weak, in fact, that the actual presence of the thing to be exposed to was for all intents and purposes totally nonexistent. This is the serial dilution that SteveBaker refers to above. When you find "homeopathic remedies" at the store, that's what they are. Not worth a cent. But the basic idea that led to them was not itself fraudulent or stupid, but it's not the first time that the road to quackery was paved with not totally insensible reasoning.
It's worth noting that homeopathy is somewhat popular right now not because it is effective but because people have become increasingly suspicious of modern medicine. Sometimes their suspicions are well-founded—the capitalist aspects the modern pharmaceutical industry, for example, has led to the marketing of all sorts of medicines to "cure" all sorts of conditions that may or may not even really be conditions, and there have been a number of high-profile cases lately where it has been clear that the regulatory apparatus for making sure commercial pharmaceuticals are safe have severe and unpardonable holes in it. Similarly, in the US at least, the state of our healthcare system is bad enough to drive any sane person mad—I've yet to meet one person who didn't have some incredible horror story related to someone they personally knew regarding difficulties with getting their healthcare insurance company to pay for some necessary procedure. Additionally there have been many calls from within the medical community for reform of medical education in this country, as it seems to produce highly disinterested, arrogant, and heartless practitioners. So let's just put the exact veracity of all of that aside (there are arguments on both sides, and these criticisms are sufficiently mainstream as to be known to most people who read, say, the New York Times or the Wall Street Journal), and just say that there are some real and non-crazy concerns relating to the state of modern medicine. But to embrace the exact opposite of modern medicine just because there are flaws with modern medicine is not a rational decision. Unfortunately at the moment the degree of discontent and mistrust is high enough that even friends of mine who I consider to be extremely intelligent are doing things like this—irrational actions that, at times, even put their own lives in danger. Oh, what a world... --98.217.8.46 (talk) 19:32, 23 August 2008 (UTC)[reply]
You are completely right. But we are (as a whole) fawning subjects not deserving any better. 93.132.139.138 (talk) 21:05, 23 August 2008 (UTC)[reply]
A concept related to homeopathy is water memory, the speculation that water which used to have a certain chemical will retain the chemical's properties after dilution. Supposedly, an antibody solution diluted so many times that not a single antibody is likely to remain can still have an effect on basophils. The alleged effect's proponents were discredited by later experiments.
Homeopathy is an example of the law of contagion mentioned in Magical thinking#Overview, as its supporters claim "things which were once in physical contact maintain a connection even after physical contact has been broken".
I should note that the companies and ardent promoters of homeopathy likely know pure water can't treat diseases no matter what. But selling drugs and books brings in cash, and with such a well-known type of quackery, a lot of cash. Hiring a team of scientists to invent, test, and retest drugs is much more expensive and time-consuming that filling bottles with tap water. --Bowlhover (talk) 21:44, 23 August 2008 (UTC)[reply]
Yep - they also advertise "No side effects!" - which is a very peculiar claim. How does the drug know what you want it to do and to what degree?! SteveBaker (talk) 04:22, 24 August 2008 (UTC)[reply]
Homeopathy is a wonderful "science". We know infected food cause illness, so if you have illness, according to homeopathy your medicine will be diluted infected food. Otolemur crassicaudatus (talk) 14:59, 24 August 2008 (UTC)[reply]
PS: By using homeopathy, you can turn a dead man alive. Just give him diluted poison!!! Otolemur crassicaudatus (talk) 15:03, 24 August 2008 (UTC)[reply]
Do bear in mind that the placebo effect is extremely powerful and even oddly complicated, sometimes working even when you've been told you're not taking any actual medicine. It is involved in all medicines we take to some extent, and proper homeopathic remedies should have no side-effects if you've been led to expect that they don't. Let placebos 'unlock your body's healing potential': take homeopathic cures (when recommended by a licensed doctor)! :D 79.66.45.71 (talk) 20:58, 25 August 2008 (UTC)[reply]

Light sensitivity disease?

There's a very, very rare disease that causes the sufferers to be painfully allergic to light. I can't remember the name of it -- can anyone help? I think it may start with an X, but I could be wrong. (It's not lupus, photophobia or iritis either.) 99.245.92.47 (talk) 15:58, 23 August 2008 (UTC)[reply]

Light allergy redirects to Xeroderma pigmentosum – is that what you're after? Algebraist 16:00, 23 August 2008 (UTC)[reply]
YES, thank you! 99.245.92.47 (talk) 16:01, 23 August 2008 (UTC)[reply]

Why have I never had a hangover?

I'm a woman in my late teens, 5'1" and 110lbs. With the amount of drinking I do some nights, I should surely wake up miserable. But after a night of drinking I usually find myself getting up early and feeling great (TOTALLY contrary to my normal behaviour) while all my friends fall over themselves trying to get to the aspirin bottle. Why might this be? I do drink water when I drink, but probably not even as much as I should.

If this constitutes "medical advice" (I don't think so: it's just a matter of curiosity!) just imagine I'm talking about a purely hypothetical female of my age. 99.245.92.47 (talk) 16:12, 23 August 2008 (UTC)[reply]

I'm not entirely sure, I'm a 6'1" male and I'm underweight, but I don't get hangovers either, regardless of how much I drink. I've never been sick (physically, I've been ill clearly) in my life. I have no idea why, I assume some people just have better liver's than others ;) —Cyclonenim (talk · contribs) 16:16, 23 August 2008 (UTC)[reply]
Drinking water is going to help, one of the major causes of a hangover is dehydration. Different people also experience hangovers differently, I don't know why. Personally, I'll feel really sick (and sometimes am sick) after an excessive night out, but have never had a headache from drinking too much (I always drink at least a pint, preferably two, of water before bed if I've been drinking). I know a girl whose main reaction to a night out is waking up early - normally she won't get up until late morning if she can help it, but after a night out she's up at 6! (Having been out until the early hours, so she must get next to no sleep...) --Tango (talk) 17:14, 23 August 2008 (UTC)[reply]
OR: For me, one of the symptoms of intoxication is an inability to keep down any quantity of water. This is annoying. Algebraist 19:21, 23 August 2008 (UTC)[reply]
I've not had that problem before bed, but the morning after is another matter - it takes a long time to get over a hangover when you can't keep down water. --Tango (talk) 20:11, 23 August 2008 (UTC)[reply]
Avoiding dehydration is definitely the number one thing. If you can't keep it down - then you're probably emptying your stomach of residual alcohol - which can only help. If I'm not to far gone to remember to drink a pint of water before going to bed - I don't get a hangover either. If I do forget - it's not good. Please don't imagine that just because you didn't get a hangover that you're OK though. You can (for example) have enough alcohol still in your bloodstream the following morning to make it illegal for you to drive. That remaining alcohol might be the explanation for why you feel differently than usual the following morning. You're also building up a resistance to alcohol which can result in you drinking dangerous amounts in order to get the 'buzz'. SteveBaker (talk) 20:41, 23 August 2008 (UTC)[reply]
Age probably has a lot to do with it. I didn't start getting hangovers until my late twenties, and then they didn't get bad until my early thirties. Amytown (talk) 23:41, 23 August 2008 (UTC)[reply]
I usually find that after drinking a dangerous amount of alcohol that I miss the bus(zz)! —Preceding unsigned comment added by Moonspeaker (talkcontribs) 03:27, 24 August 2008 (UTC)[reply]

Diagnostic of brain injuries

After an accident, without high-technology, how can doctors detect a brain injury? —Preceding unsigned comment added by Mr.K. (talkcontribs) 19:16, 23 August 2008 (UTC)[reply]

Talking with the patient can give a lot of information up to a point, especially relating to serious injury. --98.217.8.46 (talk) 19:35, 23 August 2008 (UTC)[reply]
There are a number of diagnostic scales, such as Glasgow Coma Scale. Rancho Los Amigos Scale, Blantyre Coma Scale, and Paediatric Glasgow Coma Scale, which allow some basic insight into the patient's level of consciousness. Reaction to pain stimuli, motor reflexes, the pupillary light reflex, presense of aphasia or paralysis or an altered mental state can indicate brain injury and give some clue as to its nature. But pretty much all these symptoms can also be symptoms of stroke or tumor or some kind of intoxication or poisoning or metabolic snafu, so detecting and diagnosing a physical brain injury without further technical means is difficult. And of course just because someone has (for example) been hit for by a car doesn't automatically mean that any cerebral deficit is necessarily caused by that injury - they could just as easily be drunk or stoned or have had a stroke (precipitating the accident). -- Finlay McWalter | Talk 19:58, 23 August 2008 (UTC)[reply]
With most traumatic brain injuries, CT scans are needed to confirm the diagnosis but a high level of clinical suspicion is required for almost every brain injury that comes into the ER. We have a very good article on traumatic brain injury which can be found here. —Cyclonenim (talk · contribs) 22:27, 23 August 2008 (UTC)[reply]

Towels

What are those shake-dry towels called properly?

What are the made of? —Preceding unsigned comment added by 81.132.159.110 (talk) 19:17, 23 August 2008 (UTC)[reply]

Do you mean terrycloth or something else? --98.217.8.46 (talk) 19:48, 23 August 2008 (UTC)[reply]
Do you mean quickdrying microfibre towels? Nil Einne (talk) 20:33, 23 August 2008 (UTC)[reply]
Chamois leather? --Russoc4 (talk) 23:41, 23 August 2008 (UTC)[reply]

If a man and a cheetah were to run a marathon...

...then, assuming that the man was physically prepared to run a marathon, he would easily finish way ahead of the cat.

I had this factoid quoted to me today. True or not? --Kurt Shaped Box (talk) 23:28, 23 August 2008 (UTC)[reply]

Can cheetahs not run as fast as they do for 26.2 miles? --Russoc4 (talk) 23:39, 23 August 2008 (UTC)[reply]
No, they cannot. According to our article they can sustain top speed for less than half a kilometre, and they may need to rest afterwards. Algebraist 23:42, 23 August 2008 (UTC)[reply]
That doesn't answer the original question, though. The Cheetah wouldn't need to go anywhere near it's top speed to race against a human. It would need an average of only about 13MPH (21kph). I'll bet you'll have difficulty finding low-speed endurance statistics on Cheetahs, everything I've read about them suggests that jogging isn't their style in the wild, and the references to the WP article suggest that virtually no speed experiments have been done with captive cheetahs. (Except for a couple of very informal top-speed tests.) I'll bet the answer to this marathon factoid simply isn't known. However, See Man versus Horse Marathon for a similar contest, a horse seems to have better endurance than a cheetah, but even horses sometimes lose to humans in a marathon. APL (talk) 00:16, 24 August 2008 (UTC)[reply]
Forget cheetahs, Olympic marathoners run ~12-13 miles per hour over that 26 miles. Endurance racing horses are typically running 10-11 miles per hour over long distances, so a world-class marathoner is likely to beat a horse over that distance. Humans are better at endurance running (over long distances) than nearly any animal. Most animals, such as the cheetah, are pretty specialized for short distance sprints. Dragons flight (talk) 00:25, 24 August 2008 (UTC)[reply]
I agree that the cheetah would probably lose, but the horse might stand more of a chance without a giant simian on its back. Algebraist 00:29, 24 August 2008 (UTC)[reply]
No it wouldn't - it would go the wrong way. Horses are one of the dumbest mammals (low brain to body weight ratio). As for the Cheetah, you are all assuming that the Cheetah is going to understand the rule about not eating the other competitors, disregarding of which rule could prove to be a winning tactic. SpinningSpark 17:48, 24 August 2008 (UTC)[reply]
That would lead to the cheetah's immediate disqualification, and a win by default for the (remains of the) human. And who said the human wasn't carrying a kalashnikov, anyway? Algebraist 17:51, 24 August 2008 (UTC)[reply]
Cats, in general, are sprinters. A housecat can escape a person if there is a quick escape route, but in an open field they would be at a disadvantage. StuRat (talk) 00:42, 24 August 2008 (UTC)[reply]
While humans are good at endurance running, being able to hunt some faster animals by just plain chasing them till they collapse. Algebraist 00:46, 24 August 2008 (UTC)[reply]
There are actually very few animals that have greater endurance than a human who is in reasonably good condition. Marathon runner manage 26 miles - but that's nothing. I had a neighbour once who did the London to Brighton run (that's over 60 miles), the round-Isle-of-Wight race and the South Downs Way run - and IIRC, one or other of those was over 90 miles. SteveBaker (talk) 04:18, 24 August 2008 (UTC)[reply]
I think there are quite a few marathon runners that might disagree with you about 26 miles being nothing! ;) --Tango (talk) 04:32, 24 August 2008 (UTC)[reply]
Appreciate the answers, guys and girls. I actually suspected something along those lines, having heard of persistence hunting before... --Kurt Shaped Box (talk) 16:57, 24 August 2008 (UTC)[reply]
A marathon is nothing. Dick Hoyt completed six Ironman Triathlons, of which the running part alone is 26.2 miles, while moving his paralyzed, full-grown son with him. The horses aren't the only ones who have to carry people. — DanielLC 17:18, 24 August 2008 (UTC)[reply]
My problem with these hypothetical situations is that it pits an average animal against a highly trained human. An average human cannot complete a marathon any better than an average cheetah could. Now, assume we specially bred and trained marathon-running cheetahs. How would they compare to humans? To my knowledge, the only animals we train for long-distance running are horses, sled dogs, and camels. All of them are trained to run while under the heavy load of a human. We do not train humans to run marathons while dragging another human along. -- kainaw 17:45, 24 August 2008 (UTC)[reply]
You've never seen those guys running the London Marathon in pantomime horse costumes? ;) --Kurt Shaped Box (talk) 17:59, 24 August 2008 (UTC)[reply]
My wife ran the London marathon - it took her well over four and a half hours - but she didn't walk much of it - she just has short legs! That was after a mere handful of training runs over the preceeding few weeks - not one of which was over 13 miles. An average 21st century western-world couch-potato kind of a human might have some trouble - but a "natural" human who had to go out and hunt wild animals every day of his life shouldn't have any trouble whatever running 26 miles in a respectable time. I really don't think a cheetah would stand a chance after the first mile or so. Cheetah only do one of those amazing sprints every couple of days - they lie around in trees the rest of the time - they just aren't built for a jogging pace.
A nit: cheetahs can't climb trees. --Sean 15:04, 25 August 2008 (UTC)[reply]
I think even the sagest cheetah would have trouble grasping the wisdom of the Self-Transcendence 3100 Mile Race -- 5649 laps around a single city block in New York. --Sean 15:04, 25 August 2008 (UTC)[reply]
That race sounds like a taster of the kind of eternal punishment that an angry Greek god might hand out to a misbehaving runner in the underworld... --Kurt Shaped Box (talk) 17:08, 25 August 2008 (UTC)[reply]
If you want an *average* human to compare with your average cheetah, I think even a two mile run would still be a win for the human.
(You'd be better off picking someone who wasn't raised on a diet of McDonald's hamburgers, but that's not typical of humanity as a whole. If I really wanted an average human, I'd pick someone of multiethnic or multiracial ancestry who was raised neither in one of the top five or ten richest nations nor in a very poor country.)
Cheetahs go roughly 60 mph, maybe more, (it's hard to measure) for about 30 seconds. That's about a half mile. (Can cheetahs even run slower, for endurance? I don't think they work that way.)
As for protecting the human, I don't think cheetahs attack man very often (do they ever)? Vultur (talk) 23:26, 27 August 2008 (UTC)[reply]
Of course cheetahs can run slower for longer distances and that's part of their typical hunting technique. When a cheetah hunts, it jogs toward the herd of prey animals to get them running; it parries in and out and makes test runs to get a feel for where the old or young or sick might be. It does that because no cheetah will catch a healthy antelope on a straight run, not unless it's almost on top of it before getting noticed. After several half-effort jogs at the herd, the cat will pick its target (if any) and change gears to sprint. On a run of anything less than a few miles, the cheetah will finish way ahead of the human (assuming you could get it to understand the reasoning) - it jogs at about the speed Usain ran last week, then speeds up to two or three times that for the last half mile or so. Not close. Matt Deres (talk) 14:21, 28 August 2008 (UTC)[reply]

There is one other aspect that needs to be considered. Talking about "average human" that can run marathons. It is only of late that they can, since the development of running shoes that correct supination, pronation and prevent shin-splints etc. 50 years ago, running a marathon was something special as shown in the number of people in the starting line-ups. Not today. Even I have run 12 of them, incl. the London, and I am hardly biomechanically perfect. You should ask yourself whether the cheetah will win if the human runs on bare feet. Yes it will, because the average human will take weeks to complete the marathon, having to recover from incapacitating injuries along the way. I wager that cheetahs surviving in the wild are biomechanically in better condition than humans in their natural environment (i.e on Wiki...)

August 24

Earth's Magnetic Poles

Will Earth's magnetic poles reverse in the near future, causing chaos? 66.53.220.56 (talk) 01:42, 24 August 2008 (UTC)[reply]

Geomagnetic reversal#Future of the present field suggests that it's not likely to happen soon, but the whole thing's too poorly understood to predict with any certainty. Algebraist 01:47, 24 August 2008 (UTC)[reply]
I suppose it depends on what you mean by "near" future. The poles do reverse at intervals - but the intervals aren't very regular or predictable - so it could happen any moment now or not for hundreds of thousands of years. A reversed field would certainly cause some confusion and difficulties - but the real worry is that WHILE it's flipping, there might be no magnetic field at all for a period of years to decades. That's very bad indeed because the magnetic field is implicated in the processes for deflecting radiation from the sun harmlessly around the planet. When the magnetic field simply disappears prior to the flip - we're going to be irradiated. That's going to play hell with human health and do bad things to electronics and such. Life has survived these flips before - but there is a big difference between the species surviving and the horrors for the individuals of the species. SteveBaker (talk) 04:14, 24 August 2008 (UTC)[reply]
The dangers are far from certain - there are theories that the ionosphere may protect us sufficiently, even without a magnetic field. --Tango (talk) 04:31, 24 August 2008 (UTC)[reply]

Spontaneous human combustion

Is this real?? —Preceding unsigned comment added by Moonspeaker (talkcontribs) 02:20, 24 August 2008 (UTC)[reply]

Try Spontaneous human combustion. --Tango (talk) 02:47, 24 August 2008 (UTC)[reply]

Experiments with body gases and blow back

I herd some chemistry students say they could set fire to their own rectal gases like methane, but if the velocity of combustion is greater than the velocity of expelled gas, would that cause a blow back and severe injury? —Preceding unsigned comment added by Moonspeaker (talkcontribs) 02:23, 24 August 2008 (UTC)[reply]

While it doesn't mention it that I can find, there is an article on fart lighting (really??). There may be a problem with a lack of oxygen inside, but could certainly burn some sensitive areas. --Bennybp (talk) 02:38, 24 August 2008 (UTC)[reply]
According to Flatulence#Composition_of_flatus_gases, the oxygen content of a fart (I presume whilst still inside the body) is much less than the 21% you'd find in the air. So, I guess it would be substantially more difficult to ignite the internal gases - unless you were using something like a blowtorch. --Kurt Shaped Box (talk) 02:52, 24 August 2008 (UTC)[reply]
thanks for the link Kurt BennyP. I just edited that page to correct some inaccuracies! —Preceding unsigned comment added by Moonspeaker (talkcontribs) 03:16, 24 August 2008 (UTC)[reply]
I know a woman who claims such 'blow back' resulted in a trip to the emergency room for her. ike9898 (talk) 17:39, 25 August 2008 (UTC)[reply]

Earths core

What is the elemental composition of the earths molten core? —Preceding unsigned comment added by Moonspeaker (talkcontribs) 03:25, 24 August 2008 (UTC)[reply]

The earth's core isn't molten, it's solid, and it's mostly iron. The outer core is molten though, and I believe it's mostly iron too. ScienceApe (talk) 03:34, 24 August 2008 (UTC)[reply]
See Structure of the Earth. The core is mostly iron and nickel. --Tango (talk) 04:28, 24 August 2008 (UTC)[reply]
I think there is lots of gold at the center of the earth, at least considering the amounts in human terms and mining experience. Gravitation would have made it accumulate there when the entire earth was gaseous, and later molten. The only gold at the surface today would be a small amount taken there by turbulence in earlier times. Platinum and other heavy metals are probably in the center of the earth in copious amounts too, at least in human terms. What does the average density of the earth tell us about that possibility? If I remember right, the average density of the earth is 4-1/2 to 5 times the density of water. It follows that material (including iron) other than heavy metal is present in much greater volume and mass than heavy metal. But the amount of heavy metal near the center may be stupendous in every-day human terms and mining experience. Spectroscopic analysis of stars does not show an abundance of heavy metals in the universe. Quite the opposite - lots and lots of hydrogen everywhere. But the stars are still in gaseous form, and furthermore we can only do spectroscopic analysis of light that comes from the surface of stars. Who knows what goes on deep within stars, with the enormous pressure and heat. There could be lots of heavy metal there, retained below the surface by gravity. However, spectroscopic analysis of self-luminous (non-dust) nebulae, I think, shows mostly hydrogen. But is the pressure within nebulae relatively small? Andme2 (talk) 21:28, 24 August 2008 (UTC)[reply]
You're talking about this as if it's some undiscovered mystery - when in truth we know fairly well what's going on there.
The mean density of the earth is around 5.5 - iron is around 7 and nickel nearer to 8. This suggests that the nickel-iron core is NOT likely to contain massive quantities of heavier elements. However, by observing how seismic waves from earthquakes are reflected and refracted through the core, we can deduce a lot about the composition and density - so this isn't just a guess.
As for stars - sure, the majority of stars are busily converting hydrogen and helium into higher atomic number elements - but old stars (the vast majority of them that aren't so heavy that they collapse into neutron stars or black holes) will eventually run out of those lightweight fuels. They start to convert progressively heavier elements in to yet heavier elements. When eventually the star can't gain enough energy to do that anymore - it may go nova and explode. That causes it to send out massive quantities of heavy elements - carbon, oxygen, silicon, iron, nickel...you name it. That's the origin of all of the heavier elements in the universe and appears to adequately explain the prevelance of those materials in rocky planets such as the Earth.
SteveBaker (talk) 14:38, 25 August 2008 (UTC)[reply]

Pure fusion bomb

Over at pure fusion weapon the article states that the hypothetical weapon offers the possibility of generating very small nuclear yields. I'm assuming this is done simply by limiting the amount of fusionable materials in the bomb to a minimum. So it begs the question, what is preventing fission bombs from also being very small in explosive yield? I'm aware of the Davy Crockett (nuclear device) but it says that "The Mk-54 weighed about 51 lb (23 kg), with a selectable yield of 10 or 20 tons (very close to the minimum practical size and yield for a fission warhead)". What is stopping a fission warhead from being smaller in yield? Can't you just limit the amount of fissionable material in the bomb? ScienceApe (talk) 03:33, 24 August 2008 (UTC)[reply]

My guess is that the chain reaction in a fission bomb requires that there are enough neutrons to initiate it. There is a 'critical mass' of radioactive material - beneath which fission won't occur. That puts a limit on the minimum size of the resulting explosion. A fusion reaction relies on an external source of energy (which is currently going to be a small fission bomb) to sustain the reaction - but it's not chain reaction - so you can have much smaller amounts of material. As you point out though - pure fusion bombs don't exist - and that's probably because of the difficulty of packing a suitable energy source into the weapon. SteveBaker (talk) 04:08, 24 August 2008 (UTC)[reply]
That was a very inexact answer for you, Steve: the critical mass is not the lower limit below which fission won't occur, as fission occurs naturally all the time in radioactive material, but you don't even need those natural neutrons to initiate fission, as you can get a cheap and powerful neutron generator to get it going. The critical mass is the limit below which a self-sustaining (or runaway, in the case of a bomb) series of non-spontaneous fissions occur (i.e., fissions caused by a heavy nucleus being pinged by a flying neutron, which itself came from a just-fizzed nucleus, and whose own neutrons then go on to ping someone else). The reason there's a lower limit is that the smaller the lump of material, the greater the chance that a neutron will fly uselessly away before it happens to hit one of those wobbly nuclei (which of course make up only the tiny center of an atom, which is mostly empty space).
Think of your question this way, ScienceApe: imagine the smallest possible fission bomb: two plutonium atoms and one free neutron. You shoot one of the Pu atoms with the neutron, it splits, and its own free neutron(s) go flying off in a random direction. What are the chances that they happen to hit the other Pu atom? Pretty low. But if instead the target atom was surrounded by other atoms ... --Sean 15:27, 25 August 2008 (UTC)[reply]
See critical mass. --Tango (talk) 04:29, 24 August 2008 (UTC)[reply]
I think to go much below the critical mass is not possible. You can add additional neutron source and compress the material with explosives, both done in warheads to make it smaller. But with the method to tickling the dragon's tail I think you get the minimum explosion possible, just enough to seperate the two halfs of critical mass bay a short bounce. There is a filme where one guy smashes two halfs of a bomb together by hand in a ballroom. Only a small blue light and nothing eles.--Stone (talk) 08:06, 24 August 2008 (UTC)[reply]
Note to anyone with hemispheres of plutonium lying around: read criticality accident before trying what Stone suggested. --Sean 15:27, 25 August 2008 (UTC)[reply]
You can have sub-critical fission devices but they're not worth the expense (they've happened accidentally a few times—see fizzle—and are sometimes made as a way to skirt nuclear test ban treaties but still get hydrodynamic information about plutonium, etc.). You don't get much more bang out of them than the explosives necessary to set them off, and while you do end up with a little radioactive contamination, its not nearly as much as would be the case with a concerted "dirty bomb", and at maybe a million times the cost. (Note also that the Davy Crockett bomb, and other such small weapons, are actually extremely inefficient when you look at their yield to weight ratio—less efficient than Fat Man and Little Boy.) --98.217.8.46 (talk) 14:11, 26 August 2008 (UTC)[reply]

Heliocentrism

In light of Galilean relativity, which seems to say that "the question of who is at the center is meaningless" [7], in what sense were Copernicus and Galileo right about heliocentrism? I've heard that heliocentric models have more predictive power, but according to [[8]], "The ideas presented by Copernicus were not markedly easier to use than the geocentric theory and did not produce more accurate predictions of planetary positions." Is it right in the sense that in terms of classical physics, a point inside the sun is the center of mass of the Earth/Sun system? Or is heliocentrism right in the sense that an inertial reference frame can be constructed where the Earth revolves around the Sun but not vice versa? --Histirisis (talk) 03:43, 24 August 2008 (UTC)[reply]

Well, the Copernican view was getting very complicated - from the perspective of an observer looking at a two-dimensional sky, the idea of the planets being attached to spheres, attached to spheres...possibly attached to spheres - essentially boils down to the sum of a set of sine-waves. But Fourier analysis says that we know that any function can be represented by the sum of a set of sine waves. Hence, the Copernican model could have explained literally any planetary motion given enough spheres. If the orbit of Mars spelled out COPERNICUS in an elegant script font - with enough spheres, you could describe it.
So while it's possible to describe orbits that way, the resulting math tells you nothing about why the planets move like that. It has predictive value - but no descriptive power.
The modern view of planetary motion is a much simpler equation - the very form of the math predicts the gravitational forces that cause the motion.
It's also not true that there is no difference between the heliocentric view and the geocentric. The redshift of a distant star is different in summer and in winter due to the motion of the earth around the sun. If the earth was stationary and the sun were moving, there would be no such redshift.
SteveBaker (talk) 04:02, 24 August 2008 (UTC)[reply]
Your explanation regarding simplicity makes sense to me, but your last point about redshift seems to beg the question of whether the Earth-Sun system is heliocentric. If redshift is caused by relative motion between the stars and the Earth, we could just as well (though not as easily) construct a geocentric model where the earth is stationary and the stars move in epicycles and so on. --Histirisis (talk) 04:58, 24 August 2008 (UTC)[reply]
There is no such thing as absolute rest, so you could consider either the Earth or the Sun to be stationary, however if you consider the Earth as stationary then the motion of the other planets becomes extremely complicated. If you consider the Sun as stationary, then the other planets just move in ellipses. --Tango (talk) 04:24, 24 August 2008 (UTC)[reply]
Correct me if I'm wrong, but the question of whether the Sun or the Earth is at the centre is definitely not meaningless. The Sun is in an inertial frame of reference while Earth, because it is accelerating due to the Sun's gravity, is not. Physical laws do not apply in non-intertial frames unless they are specifically modified; thus, whichever object is not accelerating can be said to have the correct perspective. Saying the planets orbit the Sun makes perfect sense, but a geocentric model requires a model of gravitation because the question of why Earth's gravity should dominate or why celestial bodies follow their observed paths have to be explained.
Note that the Sun is not in a strictly inertial reference frame since the same gravitational force applied to Earth is exerted on it; however, because of the Sun's high mass, the force's effect is negligible. The Earth-Sun system's barycenter is well below the Sun's surface.
"The ideas presented by Copernicus were not markedly easier to use than the geocentric theory and did not produce more accurate predictions of planetary positions" Copernicus still assumed the planets follow perfectly circular orbits rather than the elliptical Kepler orbits.
"Is it right in the sense that in terms of classical physics, a point inside the sun is the center of mass of the Earth/Sun system?'
Yes, and this point is the barycenter I mentioned earlier. --Bowlhover (talk) 07:11, 24 August 2008 (UTC)[reply]
(e/c - indenting) Huh? I thought that Einstein said that anything could be defined as the reference frame? And Mach said that the reference frame was the distant stars. Franamax (talk) 20:46, 24 August 2008 (UTC)[reply]
The idea that only inertial frames of reference are equally valid is only part of special relativity. To introduce gravity, you need to work in general relativity and there all reference frames are equally valid (gravity is just curvature of space, so acceleration due to gravity doesn't really count as acceleration, you're still on a geodesic). --Tango (talk) 20:41, 24 August 2008 (UTC)[reply]

Is there any possible explanation of Foucault pendulum in geocentric theory? - 203.129.237.147 (talk) 05:41, 25 August 2008 (UTC)[reply]

Yes and no. The Foucault pendulum demonstrates the effects of Earth's rotation - so to that extent it is geocentric. However, the only way we know the pendulum is precessing is because we are able to measure by reference to the distant stars. If there were no stars in the sky, we would have know way of knowing whether the Earth was turning the pendulum, or the pendulum was turning the Earth (and the Sun). Without external reference points, it's a pendulo-centric theory. Franamax (talk) 07:25, 25 August 2008 (UTC)[reply]

Hygiene

When human beings defecate, they will clean/wipe the area afterwards ... not only for cosmetic reasons, but also for hygienic ones. I assume that cleaning the area afterwards reduces germs, bacteria, infection, disease, etc., ... ultimately promoting better health. Do animals engage in any similar conduct of cleaning/wiping ... or do they simply defecate and move on? Thanks. (Joseph A. Spadaro (talk) 03:46, 24 August 2008 (UTC))[reply]

Dogs and cats scooch sometimes. I assume it is to relieve an irritation.--80.176.225.249 (talk) 12:43, 24 August 2008 (UTC)[reply]
As far as I know, animals which can be seen in cities and villages like cats, dogs, cows etc. defecate and move on. I do not know the individual behavior of animals in wild. Otolemur crassicaudatus (talk) 13:09, 24 August 2008 (UTC)[reply]
Domestic cats clean themselves meticulously, which is partly why they make such good pets. As far as I know, no other animal is as fussy, although some animals, such as rabbits and rodents, produce quite solid pellets so there is really no mess to clean up. Birds eject a thick liquid so again no cleaning is necessary. Aquatic animals don't need to bother of course. Some tails help to keep flies away.--Shantavira|feed me 15:25, 24 August 2008 (UTC)[reply]
Budgerigars will clean their vent area meticulously with both beak and claws if any poop happens to stick to the surrounding feathers on the way out. They don't like matted plumage. --Kurt Shaped Box (talk) 16:55, 24 August 2008 (UTC)[reply]
We're also the only animals that wear clothes. Animals don't have to worry about dirty underwear like us. I think that when dogs and cats "scooch" (as 80.176 pointed out) it's because they itch. --Shaggorama (talk) 19:09, 24 August 2008 (UTC)[reply]
NADO - but don't dogs sometimes 'scooch' when their anal glands are overfull? --Kurt Shaped Box (talk) 00:57, 25 August 2008 (UTC)[reply]
I've noticed that when my dog shits, her anus is totally clean afterward, with no wiping or anything, and that she performs this magic trick by extending her anus/rectum out a little ways (like turning a sock inside out) so that it's like she's pooping from a tube. When the poop has plopped, she pulls the tube back in with the outer anus never having touched anything unpleasant. It probably helps that she doesn't have buttocks. Humans do that same tube thing a little, but not enough to avoid needing "bathroom tissue", I guess. You can see what I mean in this picture (graphic image warning): tubgirl. --Sean 15:37, 25 August 2008 (UTC)[reply]

What is the nature of consciousness?

What is the bridge between thoughts appearing in the brain and becoming conscious property? By "property" I mean your identity "claims" it and says "yes, I thought this" when, in reality, your brain thought of it. As one often perplexed by the inner light, I must ask what this energy is. The energy we're clearly borrowing that we then claim as our own that....controls us? It seems we exist as an odd mix of mental slavery and postthought free will. How do I know? It's what my brain just told me. Or did it? Perhaps I'm just at one with it and I didnt "think" anything at all. Of course my brain just "told" me that also. I'm so confused.--Hey, I'm Just Curious (talk) 04:14, 24 August 2008 (UTC)[reply]

Exhaust Consciousness and the related links near the bottom. You may also find Dendrite of interest. As for the fundamental nature - I must defer to Diogenes of Sinope - "When asked how he wished to be buried, he left instructions to be thrown outside the city wall so wild animals could feast on his body. When asked if he minded this, he said, "Not at all, as long as you provide me with a stick to chase the creatures away!" When asked how he could use the stick since he would lack awareness, he replied "If I lack awareness, then why should I care what happens to me when I am dead?" 98.169.163.20 (talk) 04:55, 24 August 2008 (UTC)[reply]
See also Epiphenomenon. Some have argued that what seems like thinking, reasoning, and decision making, or intentional movements are mere associative byplay, like a television viewer imagining he affects what happens on the screen by wishing or by yelling orders. Edison2 (talk) 05:36, 24 August 2008 (UTC)[reply]
This is a very vigorous field of research right now - new findings are popping up from brain scanners and subtle experiments on an almost weekly basis. From the way things are going, I'd say that we've learned these things:
  • Our consciousness is a relatively recent evolutionary development - which seems reasonable given that so few animals seem to exhibit it. That means that we had to be fully functional working beings before we had consciousness. At least one researcher believes that we only developed it maybe 10,000 years ago! But either way, that in turn means that we don't need it in order to perform most mundane tasks. Evolution works by small incremental change - and switching over the entire way the brain works from "automatic" to "conscious" processing just doesn't seem like the kind of thing that could have happened.
  • The amount of data that can flow into and out of our conscious mind is between 11 and 40 bits per second. That's an actual measured flow rate. Anyone who does anything with computers knows that is an AMAZINGLY low data rate. It's not enough (for example) to allow us to read at normal reading speeds. It's certainly not enough to allow us to see everything we think we can see. With so few bits, it's clear that (for example) the conscious mind can't drive a car.
  • Many brain-scan experiments have found that we make decisions many seconds before we are conscious of doing so.
  • Other studies suggest that our consciousness operates several seconds behind "real time" and that the subconscious contains mechanisms to "edit" the timeline that the conscious mind perceives so that it merely seems like we're operating in realtime. If that's true then the conscious mind doesn't make any decisions whatever...it functions merely as a monitoring mechanism that operates long after the 'animal mind' has done everything that needed to be done.
So I guess the answer to your question is "many seconds".
SteveBaker (talk) 16:17, 24 August 2008 (UTC)[reply]
See Daniel Dennett's multiple drafts theory of conciousness, a response to the problems created by the Cartesian theater view. Gandalf61 (talk) 16:59, 24 August 2008 (UTC)[reply]
Very interesting what SteveBaker says. In fact, if you remember the sources where you came upon this information, we should add it to our article on consciousness.--El aprendelenguas (talk) 20:36, 24 August 2008 (UTC)[reply]
Keep in mind when reading SteveBaker's response that he's probably speaking about access consciousness, as opposed to phenomenal consciousness (which is probably appropriate since this is the science reference desk, and phenomenal consciousness, assuming it exists, cannot be measured independently). Steve, correct me if I'm wrong. --Allen (talk) 23:26, 24 August 2008 (UTC)[reply]
The book I've been reading is "The User Illusion" by Tor Norretranders ISBN 0-670-87579-1. I don't recommend it though - it was an exceedingly annoying read - it drifts FAR off-topic into areas the author clearly doesn't understand well. However, it covers a good number of the weird experiments that are popping up in reasonable detail. A fairly entertaining take on it is from Radio Lab on NPR - all of their shows are podcasted - I forget the exact titles. My wife had some journals I was reading also - sadly they are 180 miles away from my present location so I can't quote exact refs. Sorry I can't be more exact - it's not exactly my field. SteveBaker (talk) 00:25, 25 August 2008 (UTC)[reply]
Then there is instinct. Sometimes the conscious human mind can overcome it and not act in an instinctive manner. With animals, if you have seen how a domestic house cat acts when it spots a bird on the ground, it is amazing how it switches from being a sweet, friendly animal to being a hunting animal guided solely by instinct - it is a binary switch. The cat will even instinctively crouch behind a rock or other obstruction to hide its body as much as possible from the bird. Even kittens with no previous hunting experience will conceal themselves like that. Humans, also, can switch suddenly from civilized to violent behavior - from being consciously guided to being guided by instinct. As for learned (non-instinctive) behavior in humans, it may start by being consciously accepted and enacted, and in some cases may then become mainly involuntary and unconscious - that can be seen in soldiers as a result of their training. The military endeavors to make them automatons - to obey orders instantly without even thinking about it. Andme2 (talk) 06:08, 25 August 2008 (UTC)[reply]
Every one of Steve's claims is pretty significant, it seems, but I too would like to see sourcing and just how such experiments were performed. Certainly our "blink" response could not occur with an 11bit/s data processing rate. Also, 11 bit/s in what? Input, mid-level processing, or conscious processing? Because certainly we see and hear much more than 11 bit/s. SamuelRiv (talk) 06:38, 25 August 2008 (UTC)[reply]
The screen of a computer monitor displays hundreds of thousands of pixels of data. Each pixel is a dot of light, and it requires several bits of data to indicate its color, brightess, and screen position. Similarly, when we move our eyes to change the field of vision, a great many pixels of data (detected by rods and cones) are almost instantaneously changed and perceived by the conscious mind. Some of it will be in peripheral vision, but the central part would still be many pixels, with the mind fully aware of the change of these many pixels of visual data. My monitor screen displays 480,000 pixels; other monitors display even more. Andme2 (talk) 07:33, 25 August 2008 (UTC)[reply]
Both SamuelRiv and Andme2 are not understanding what I'm saying. I'm not saying that the entire brain can only process 11 bits per second - that's OBVIOUSLY not true. I'm saying that the very small portion of the brain that is the presumed seat of "consciousness" can only process between 11 and 40 bits per second. So we can blink (that's an autonomic system thing - we CAN do it consciously - but generally it happens automatically). When we move our eyes, the claim is that our SUB-conscious mind processes the multi-megabytes of data involved - takes any necessary actions and then (possibly some seconds later) tells the conscious mind what was there - at a level of broad concepts, not details. The claim is that the conscious mind may subsequently (and S-L-O-W-L-Y) interrogate the subconscious about details within the scene - but we only hear about things we want to hear.
I'm sure you're going to complain about that. But before you do, try this for me. Go to this page [9] - look where it says "View the "basketball" video" - read the instructions very carefully (that's important) - then click on the link and view the video. Then come back here and post your answer - then I'll tell you something amazing. SteveBaker (talk) 14:19, 25 August 2008 (UTC)[reply]
I actually had alot of problems with that one too. Although it is a huge simplification, the mind is sort of like software operating on the hardware of the brain. It doesn't make sense to talk about the speed at which a program handles data, since that speed is dependent on the hardware it runs on. I'd like to know what is considered a "bit" of information with respect to the conscious mind, because I think the numbers you give are pretty silly and quite frankly meaningless. Mreover, there is no identified seat of consciousness in the brain; there is no homonculus in there either. That basketball video is about attention and doesn't make any claims about bits of processed information. If you really wnat to be wowed by the conscious experience of vision, read about blindsight. I'd still like to see the study you mentioned.
I also had problems with Steve's first point about the evolution of mind. Our ancestors were using tools millions of years ago, and Homo sapiens evolved over 200,000 years ago, so the timeline you give makes no sense. Also 10,000 years precludes the possibility of animal consciousness being similar to ours (without convergent evolution on a massive scale).
The fourth point, about experience of time, is extremely exagerated. It's impossible for us to experience the world in "real time" because we only have access to sense data, but we receive it sufficiently fast for our experience to match up enough to respond to the world very quickly, otherwise we'd notice the time lapse. Parallel processing is amazing stuff.
The mind and brain don't exist in the kind of dualistic dichotomy Steve seems to suggest. Although I earlier threw out the functionalist computer analogy, the fact is your mind IS your brain. The are some facets of your cognition that you have conscious access too, and some you don't. More than investigating epiphenomenalism, I'd suggest you read into the unity of mind (much scarier and more compelling stuff, IMHO). To Steve's credit though, I can at least vouch for his third point: the research he's talking about is into something called the Readiness potential and the interpretation of the results he mentions is one first advocated by Benjamin Libet. The results of the experiment have since been interpreted differently and are still contentious, but it's an interesting read. If you get back to me later, I'll dig around my notes and find some fun UOM stuff for ya. --Shaggorama (talk) 14:34, 25 August 2008 (UTC)[reply]
"The User Illusion" by Tor Norretranders ISBN 0-670-87579-1 - chapter 6 "The bandwidth of consciousness" pp 124 to 156 in my hardcover copy. It cites half a dozen references and experiments starting in 1952 and ranging all the way up to the present - these appear to all have been printed in peer-reviewed journals or books that are considered standard texts - so they aren't whacko-theories, they reach at least Wikipedia's standards for mainstream science. What more can I say? Find a copy in a bookstore or library and check it out. SteveBaker (talk) 03:39, 26 August 2008 (UTC)[reply]
Thanks, I'll try to. If I don't get around to it before this thread gets archived, I'll try to get a response to your talk page. --Shaggorama (talk) 06:27, 26 August 2008 (UTC)[reply]

Oxycodone and Morphine: Detection periods?

I was wondering how long these two opioids are detectable in the urine and if quantity use can be measured. Have attempted to find a definitive answer, such as a medical journal, but have only found sites recommending one detox product or another with no definitive scientific data.

Ruairí Óg the Rogue (talk) 05:37, 24 August 2008 (UTC)[reply]

If you Google Scholar and search for: "Morphine detection urine" you will find lots of scholarly articles that address your question. A definitive answer is difficult to give because it depends "mainly on the dose and sensitivity of the method used and also on the preparation and route of administration, the duration of use (acute or chronic), the molecule or metabolite that is looked for, the pH and concentration of the urine, and the inter-individual variation in metabolic and renal clearance." [10] Some of the methods available include Latex agglutination inhibition (LAI), Thin layer chromatography (TLC), Radioimmunoassay (RIA), High Performance Liquid Chromatography (HPLC) and Gas chromatography and mass spectrometry (GC/MS). But, as an example, in addicts that used on average 0.635 grams of heroin per day, using a RIA method, morphine could be detected in the urine every day for seven days after last use. [11] Dostioffski (talk) 19:34, 24 August 2008 (UTC)[reply]

Inhalent detection (inspired by question above)

Why is it that inhalants are difficult/impossible to detect in drug tests? I would like to add this to our article but couldn't find anything through a cursory search. --mboverload@ 06:06, 24 August 2008 (UTC)[reply]

Just a guess but I would think that inhalants can range from gasoline to paper cement to Clorox Bleach, etc. So it offers a huge variety of chemicals that wouldn't be practical to test for all of them. ScienceApe (talk) 06:42, 24 August 2008 (UTC)[reply]
I presume some inhalents can't be detected since they are absorbed straight into the blood rather than going through the usual digestion process (which ususally leaves behind a remnant of some sort). Presumably, some inhalents can be completely broken down by the liver into compounds which are indistinguishable from other non-drug compounds. This is complete speculation, I'm just guessing. —[[::User:Cyclonenim|Cyclonenim]] ([[::User talk:Cyclonenim|talk]] · [[::Special:Contributions/Cyclonenim|contribs]]) 18:08, 24 August 2008 (UTC)
Actually, I think it's the reverse - digested compounds are processed by the liver to yield the active compound, and a secondary metabolite is left over which can be detected? Whereas an inhalant is the active compound itself, broken down completely within the cell whose receptor it docks with during normal cell-receptor recycling. I too am guessing! Franamax (talk) 20:35, 24 August 2008 (UTC)[reply]
Well the liver breaks down what is in the blood stream, not just digested. Digested stuff gets absorbed into the blood stream and is then processed by the liver. My point is, say, caffeine is processed by the liver to form paraxanthine, theobromine and theophylline, each of which are then metabolised a bit more then excreted. Often this metabolism process doesn't completely break things down into simple compounds or elements and they leave remnants. Perhaps some inhalents are made of substances which, when metabolised, are broken down into very simple substances in which is becomes impossible to tell what substance they originally were part of. But yeah, your explanation makes sense too, I guess we should wait for someone who knows what they're on about! —[[::User:Cyclonenim|Cyclonenim]] ([[::User talk:Cyclonenim|talk]] · [[::Special:Contributions/Cyclonenim|contribs]]) 22:40, 24 August 2008 (UTC)

Hairspray vs. Houseflies

I'm wondering what happens to a housefly when I spray it with hairspray. I've been doing this for years as a cheap alternative to insect sprays, which smell bad and can't be sprayed near food because of their toxic nature. It works like a dream, and the fly will generally get up and fly around for a minute or so before settling down and not doing anything ever again (because you can pick them up with tissue and throw them in the toilet). My theory was that the hairspray was sticking the wings together, but now I doubt this is the case, because while they are flying the wings are permanently moving. Now I think it may be the eyes. Anyone with any ideas? --ChokinBako (talk) 10:23, 24 August 2008 (UTC)[reply]

I doubt that even if the hairspray blinded the fly that it would stop it from flying. I think, probably, it is more likely that the hairspray has blocked the insect's spiracles and it is having difficulty breathing.
Your economics are a bit dubious by the way. A branded hairspray costs 70-100p per 100ml according to the ASDA website. A branded flyspray costs 33p/100ml. The supermarket's own brand of hairspray is 19p/100ml however. SpinningSpark 11:07, 24 August 2008 (UTC)[reply]
Thanks. That makes a lot of sense. My economics is perfect, considering I already use hairspray anyway so I don't have to go out and buy anything. :) --ChokinBako (talk) 11:12, 24 August 2008 (UTC)[reply]
Insects don't have lungs - the get their oxygen by absorbing it through spiracles - which are little holes down the sides of their bodies. I kinda suspect that you are clogging those up - so the animal suffocates. It could still be the wings though. Remember, insects don't have soft, flexible skin - they have chitinous plates that fit together like a suit of armor. If you are gumming up the joints, that could stop them from flying or walking. The effect might be a little delayed because the stuff has to dry out and gradually get stickier and stickier. Hair spray is basically a polymer glue dissolved in alcohol to keep it liquid. As the alcohol evaporates, the hairspray turns into a gummy glue. Hairspray is not exactly nice stuff - I wouldn't be spraying it near food!
(True story of husband and wife who find a very large spider: Wife screams - husband rushes off to find some kind of spray to deal the spider. He comes back with the only thing he can find...Windex...she says "You can't kill a spider with Windex!"..."Just watch me"...at which point he whacks the spider with the windex bottle and squishes it. QED)
SteveBaker (talk) 15:41, 24 August 2008 (UTC)[reply]

It does take a while, around a minute, for the creature to stop flying and to sit down for a bit. If I leave it alone, it eventually gets up again and will start flying around, but I've found that another quick spray permanently disables it after that. Oh, and when I say 'near food', I didn't really mean directly at it, I just meant in the same room. It may not be the toxicity of the fly-spray that makes me sick, but the thought of eating with that sickly sweet smell wafting round me certainly does! Hairspray, however, though not my favourite choice of condiment, certainly is preferred over fly-spray!--ChokinBako (talk) 16:05, 24 August 2008 (UTC)[reply]

Interesting! But I don't think that changes my theory - it maybe just takes a couple of layers of goo to finally gum up the mechanism. SteveBaker (talk) 00:14, 25 August 2008 (UTC)[reply]

neo cortical physiology and art/drama/music therapy

On a neo cortical level what effect does art/music/drama therapy have upon the brains functionality? Could art therapy be used for the development of weaker hemisphere functionality, for example in the case of single hemisphere brain damage. What research is there available in this vein? —Preceding unsigned comment added by 193.36.79.207 (talk) 10:42, 24 August 2008 (UTC)[reply]

The Size Of The Universe

I was watching a Carl Sagan tribute a few nights ago, and it said that the universe is 78 billion light years across. How is this possible? The universe is only approx. 14 billion years old. Since nothing can travel faster than the speed of light, how can it have grown to this size, considering everything would have had to travel at least 39 billion light years (the radius of the universe) in only 14 billion years? Can someone explain what I am missing?--ChokinBako (talk) 15:54, 24 August 2008 (UTC)[reply]

Hmm, this seems to have answered my question, but says that the universe is actually 156 billion light years across......--ChokinBako (talk) 16:38, 24 August 2008 (UTC)[reply]
I believe the furthest that we can see is light from 14 billion years ago. --Russoc4 (talk) 16:42, 24 August 2008 (UTC)[reply]
Good question. Read our article on the observable universe, especially the sections on Size and Misconceptions. Simplified explanation is that light from (soon after) the Big Bang has been travelling for 14 billion years but space has been expanding during that time so the point from where the photon that reaches us was emitted (or rather the matter than was in that vicinity) is now much more than 14 billion light years distant in each direction. Gandalf61 (talk) 16:47, 24 August 2008 (UTC)[reply]
(ec) Both numbers are wrong, as mentioned in Observable universe#Misconceptions. But the correct figure is larger than c times the age of the universe. The picture in Metric expansion of space#Understanding the expansion of space might be helpful in understanding what's going on. (edit: Incidentally, where did you see the Carl Sagan tribute? Was it on real-world commercial TV? I'm curious how far that humble space.com article has spread.) -- BenRG (talk) 16:48, 24 August 2008 (UTC)[reply]
(ec) See also cosmic inflation. TenOfAllTrades(talk) 16:55, 24 August 2008 (UTC)[reply]
It has nothing to do with inflation, actually. Big bang cosmology without inflation still says that the observable universe is larger than c times the age of the universe. -- BenRG (talk) 17:18, 24 August 2008 (UTC)[reply]
So, basically, what we're saying is, because the furthest we can see is 14 billion light years away, that means that the universe must be 14 billion years old, because these are the first photons hitting us? But, the objects that emitted those photons are further away than that because the universe is expanding, right? As much as 5 times further away? Shouldn't it take them 5 times the present age of the universe to get that far, IF they were travelling at the speed of light? (The Carl Sagan tribute I saw on a website I stumbled upon. I just tried looking for it again but couldn't find it). --ChokinBako (talk) 19:18, 24 August 2008 (UTC)[reply]
BenRG will have to comment whether this is appropriate to this question, but I bookmarked their previous response to a similar question here - I was quite impressed at the time, since I'd never seen "loxodrome" used in a sentence before. :) Franamax (talk) 20:28, 24 August 2008 (UTC)[reply]
Yes, thanks for linking that because it's what I would have said here anyway. Basically you should abandon the idea that light travels a distance ct in a time t. The universe is about 14 billion years old, but light doesn't travel 14 billion light years in that time. Talking about "the distance something travels" is always a bit iffy because measuring along the worldline doesn't get you a distance (it gets you the elapsed proper time, which for light is always zero). To get a distance you have to measure along some other line, and there's no one correct choice. In that diagram, the path of the light is the red line, whose length is zero. The distances quoted in Wikipedia articles (the ones I've gotten my hands on, anyway) are measured along the orange line, which is not the path taken by the light. The reason to use that particular distance measure is that it's the only one with respect to which Hubble's law holds and with respect to which the universe is homogeneous. Other distance measures make the Earth's location look special. Ned Wright has a page about this (and read his excellent cosmology tutorial if you haven't). -- BenRG (talk) 01:09, 25 August 2008 (UTC)[reply]
The way I think of it (which I think is accurate) is that after the photon was emitted, the space between where it was emitted and where it is now expanded, so after the photon has travelled for one year and has gone one light year, it's source is more than one light year away because that light year of space has expanded. --Tango (talk) 21:00, 24 August 2008 (UTC)[reply]
This makes me think ... . If a photon was emitted with frequency, say, 46THz at what now is the edge of the universe, at what frequency dose it arrive at a detector on earth now? 93.132.191.111 (talk) 19:57, 24 August 2008 (UTC)[reply]
We can't see the edge of the universe (if it even has one). If you mean the most distant object we can see, which is a part of the primordial fireball that produced the CMBR, then the photon would be redshifted by the same factor as the CMBR, which is about 1100. So, about 40 GHz. That's assuming it wasn't absorbed along the way. -- BenRG (talk) 01:09, 25 August 2008 (UTC)[reply]
Also note the age of the universe is often approximated by putting Hubble's constant in reverse, assuming the rate of expansion of the universe has remained constant (which it has not). This also gives an approximation of size vs. age of the universe. SamuelRiv (talk) 05:52, 25 August 2008 (UTC)[reply]

I just heard a (popular science) talk by Brian Greene in which he claimed that the size and shape of the universe was unknown and that the size may be infinite. ike9898 (talk) 17:02, 25 August 2008 (UTC)[reply]

Yes. We've been discussing the size of the observable universe. Algebraist 17:12, 25 August 2008 (UTC)[reply]
It seems to me like the OP referred to the actual size of the universe. If I had intend my post to be part of the main thread above, I would have indented accordingly. ike9898 (talk) 20:04, 25 August 2008 (UTC)[reply]
I thought I had said this above, but I now recall it got lost in an edit conflict: We have no knowledge of the size of the full, not necessarily observable, universe. Algebraist 23:10, 25 August 2008 (UTC)[reply]

Terminology re: motion created by amniotic fluid

Pkdisme (talk) 17:09, 24 August 2008 (UTC) Please help![reply]

I know that there is a specific term/phrase for "the motion in the womb simulated after birth by rocking" and I cannot find it.

I have checked for 2 days now and would very greatly appreciate your help.

Thank you.

After birth, the amniotic fluid is already released. Did you mean pelvic rocking to help detatch the placenta ? SpinningSpark 18:19, 24 August 2008 (UTC)[reply]
Perhaps "prenatal vestibular stimulation" ? You may find something more specific in the pre- and perinatal psychology literature. See also [12] Dostioffski (talk) 19:16, 24 August 2008 (UTC)[reply]
You're talking about how rocking a newborn baby is comforting to it because it simulates the motion of the amniotic fluid? SteveBaker (talk) 00:10, 25 August 2008 (UTC)[reply]

Future studies in areospace

hi, I am rigth now a undergraduate pursuing my mechanical degree. I wish to join the best university for my post graduation in aerospace. So what are the feilds I should research in as a mechanical engineer. —Preceding unsigned comment added by 125.16.89.84 (talk) 17:43, 24 August 2008 (UTC)[reply]

find definitions

the following words is what i need definitons for and i cant seem to find them. 1. advanced treatment options 2.level one trauma center 3.reactionary 4. proactive 5. not-for-profit, 501(c)(3) corporation 6. procuring 7. interns&internships 8. public service announcement 9. data support topic —Preceding unsigned comment added by 74.46.245.252 (talk) 21:31, 24 August 2008 (UTC)[reply]

  1. No idea without context
  2. See Trauma center
  3. See wikt:reactionary
  4. See wikt:proactive
  5. See 501(c)(3)
  6. See wikt:procure
  7. See intern
  8. See public service announcement
  9. No idea without context
Hope that helps. --Tango (talk) 21:39, 24 August 2008 (UTC)[reply]
"Advanced treatment options" is a common medical phrase for "forms of treatment that are not considered until normal forms of treatment are exhausted." It mainly has to do with insurance. You can bill more for advanced treatment than you can for normal treatment, but you must meet eligibility restrictions mandated by the insurance company. -- kainaw 22:10, 24 August 2008 (UTC)[reply]
Ah, that would explain why I hadn't heard of it - I'm British, so don't have medical insurance (see NHS!). --Tango (talk) 23:47, 24 August 2008 (UTC)[reply]
It is also used under national health care systems. While healthcare is free, doctors are not free to perform any healthcare they like. They must justify use of "advanced treatment options" if the options are allowed at all. -- kainaw 02:06, 25 August 2008 (UTC)[reply]
(One should also explain that the NHS is free in the UK - but just like the USA, you can purchase health insurance if you want to. When I last worked in the UK, I got company-provided private health insurance just as I do here in the USA. However, our family doctor did a pretty good job and the only time I actually used the private insurance was for major surgery where it was possible to get treatment sooner and get perks like a private room and restaurant quality food while recovering. People complain a lot about the NHS but that's because they haven't experienced the alternative!) SteveBaker (talk) 04:49, 25 August 2008 (UTC)[reply]

What is this bird?

2 birds.

Per instructions on the WP:PPR page, I'm supposed to ask you guys if I need to identify a bird. The birds I need IDing are the 2 birds to the right. Any other info like what gender they are, what they're doing would be appreciated. Pie is good (Apple is the best) 22:30, 24 August 2008 (UTC)[reply]

They're young gulls. At that age, they'll either be looking for food or looking for their mother. It's impossible to determine sex visually when young (it's *virtually* impossible when they're adults!). As for species, I don't really know for sure. I like interacting with gulls but I've never been a 'gull spotter', if you know what I mean. Young gulls can be tricky to identify but they're definitely 'large white-headed gulls', as described at Gull#Taxonomy. Only way I could tell you for sure is if I saw them with their mother. --Kurt Shaped Box (talk) 23:06, 24 August 2008 (UTC)[reply]
Concur, compare Image:Herring Gull Fledgling.jpg. Nanonic (talk) 23:12, 24 August 2008 (UTC)[reply]
Concur with everyone so far, and add that the folks at Wikipedia talk:WikiProject Birds are usually helpful with bird ID requests. -- Coneslayer (talk) 15:07, 25 August 2008 (UTC)[reply]

advantage of youth in gymnastics

The article Age falsification in gymnastics says younger (female) gymnasts have an advantage because "Smaller, lighter girls generally excelled in the more challenging acrobatic elements required by the redesigned Code of Points". But if this is correct, wouldn't the sport set minimum heights and weight requirements rather than age limits? So... is it really correct or is there some other advantage? —Preceding unsigned comment added by 64.228.91.86 (talk) 23:18, 24 August 2008 (UTC)[reply]

I don't know if it's true, but what they've been saying on the teevee is that the younger gymnasts are more fearless... that they have less visceral appreciation for how badly one can get hurt doing these gymnastics, and can therefore do more dangerous, impressive moves with confidence and steadiness. So that would be an advantage directly related to age. If sourced, maybe this would be good for the article. --Allen (talk) 00:06, 25 August 2008 (UTC)[reply]
I saw a news special (CNN?) at the airport on this and the gymnastic expert they had (some former gold medalist—maybe Mary Lou Retton? I was only paying half attention) said that younger girls are much more flexible and pliable and that this helps a lot in certain events. They said the downside though is that lack of experience can really hinder you, so it's not a surefire tradeoff at all. --98.217.8.46 (talk) 02:04, 25 August 2008 (UTC)[reply]
Another advantage to using underage gymnasts is that because there is more of the population to choose from, finding extremely competent athletes is easier. If younger gymnasts also have a physical advantage, the two factors combine to give a very significant advantage to the team with underage athletes. --Bowlhover (talk) 09:05, 25 August 2008 (UTC)[reply]
What would be the point of banning short people? I'm guessing here, but I suspect that they ban children to try to prevent the sport from being an exercise in child exploitation like Camel racing. APL (talk) 03:06, 25 August 2008 (UTC)[reply]
One expert on child development was quoted as saying that the if he had to design an activity that would be most likely to harm young children, the Olympic gymastic exercises would be a pretty close match. He went on to explain how the stresses imposed on only partly developed muscular/skeletal system by gymnastics are almost certain to cause damage that would persist into adulthood. It's certainly not just height and weight - and it's not even a matter of fairness for the older athletes - it's about not harming children. SteveBaker (talk) 04:44, 25 August 2008 (UTC)[reply]

FYI - I have amended the article mentioned by 64.228.91.86 (Age falsification in gymnastics) with this info provided by y'all and added fact tags where I got stuck for sources – so if anyone does remember your sources, please drop by and add them! The two new sections are called Reasons for age falsification and Reasons for age restrictions. Best, WikiJedits (talk) 17:19, 26 August 2008 (UTC)[reply]

Space Questions

Why do all the planets orbit in pretty much the same plane, and not spherical? The best reason I can come up with is that because of the suns rotation, it isn't a perfect sphere, even though at distance I don't think that would matter.

Why is it so hard to get out of Earth's (or any objects) gravity? Why can't say a specially fitted F-22 just keep applying thrust and getting higher and higher?59.100.251.111 (talk) 23:45, 24 August 2008 (UTC)[reply]

The solar system formed out of a big cloud of dust and gas that collapsed. Due to conservation of angular momentum, as it collapsed, it rotated faster and faster (spin round and round on the spot with your arms out, bring your arms in and you suddenly speed up - it's the same principle). When something rotates, in naturally spreads out into a disc (ever seen someone making pizza bases by spinning them around rather than using a rolling pin?), the planets them formed out of that disc, which is why they are all in the same plane. The reason an aeroplane can't keep thrusting and go higher and higher is because the atmosphere gets thinner and thinner and aeroplane engines need air to work (there are specially designed engines for flying at really high altitude which require less air, but there is still a limit to how high they can go). To get into space you need rockets, which don't need air to work (actually, they are more efficient in a vacuum, I think). --Tango (talk) 23:53, 24 August 2008 (UTC)[reply]
That's all correct, but there are two additional reasons why you can't "get out of Earth's gravity" using an airplane. First, not only do the engines need air, so do the wings. In the thin atmosphere at high altitude, they won't provide any lift and the plane can't keep rising unless it uses engine power alone to overcome its weight, i.e. climbing more or less straight up. Fighter airplanes do have engines powerful enough to do that, or some of them do, but they don't carry nearly enough fuel to do it for very long.
And second, in order to get free of the Earth's gravity, you need to reach escape velocity. If you're anywhere within a few hundred miles of the Earth's surface, this is about 25,000 mph (40,000 km/h). Airplanes don't carry nearly enough fuel to accelerate to such speeds, even if the atmosphere was not an issue. At a higher altitude the escape velocity is lower, but to first lift to a high altitude and then reach escape velocity would require even more fuel, so this isn't impossible either. If you look at a space shuttle or any sort of rocket intended to launch things into orbit or deeper into space, when it's sitting on the launch pad, most of its volume is fuel (and oxidizer). That's not true of airplanes.
So escaping Earth's gravity with an airplane is impossible for three different reasons. --Anonymous, 03:57 UTC, August 25, 2008.
No - you don't need to reach escape velocity. If you have a rocket engine and enough fuel you can power your way out of earth orbit at (say) ten miles per hour. Escape velocity refers to ballistic objects. If you can reach escape velocity then you can turn your engine off and still escape from the earth's gravity before falling back to earth. But if you have engines - then that's irrelevent. Think about it this way. Suppose you launch at half of escape velocity. You'd shoot up into the air - then gradually slow down more and more until your upward speed was zero. But if you fired your engine again at that exact moment, then instead of falling back down, you'd go up a little bit more. If you have enough fuel, you can keep doing that until you reach any desired altitude without ever reaching escape velocity. The only real problem is that jet engines need oxygen which they get from the air. There have been attempts to design engines that could switch modes as they change altitude. One that I happen to recall was the (unfunded) British HOTOL craft with it's RB545 engines. It would have been able to take off from an ordinary airfield just like any other plane - flown up until it's engines were gasping for oxygen - then started using liquid oxygen to supply them - effectively converting them from jets into rockets to provide the last boost into orbit. No escape velocity - no dramatic vertical launch - a true space plane. Sadly, it was never built. SteveBaker (talk) 04:39, 25 August 2008 (UTC)[reply]
You don't need to reach escape velocity if your desire is to reach orbit (you need to reach orbital speed, though), but if you want to escape Earth's gravity then you'll need to reach escape velocity at some point - you can't continue thrusting forever unless you are able to collect reaction mass from the space you travel through, which isn't a feasible way to do it. If you can thrust long enough to get a significant distance from Earth, then escape velocity is less, but you still have to reach it or you'll fall back to Earth once you run out of fuel. --Tango (talk) 17:26, 25 August 2008 (UTC)[reply]
So, essentially, you're saying that as you leave earth's gravity under power the escape velocity will go down until it reaches the speed you're already going? That makes sense. But if you were in such a space ship you wouldn't say that you were "trying to reach escape velocity" You'd just say "I'm trying to reach Mars (or wherever) under full thrust.". If you never intended to shut down your engines then escape velocity wouldn't have much practical meaning. APL (talk) 18:14, 25 August 2008 (UTC)[reply]
Yeah, that's about it. I was kind of assuming you were heading off to infinity, since that's how escape velocity is defined. If you're talking about the real world, then things are slightly different. A journey to Mars requires you to account for the Sun's gravity and Mars' gravity in addition to the Earth's. --Tango (talk) 19:01, 25 August 2008 (UTC)[reply]
I'd just like to point out that I mentioned this method in my previous item, and also pointed out that it consumes more fuel than rapidly reaching the higher escape velocity near ground level -- which is why real-life space launches always accelerate rapidly. The reason is that the engines must produce an amount of energy equal in magnitude to the spacecraft's gravitational potential energy not matter whether they do it rapidly or slowly, but if doing it slowly, they also need to use additional energy to keep the craft from falling to Earth in the meantime. Incidentally, in contrast to the physics goofs common to many movies, this bit of orbital mechanics was correctly depicted in the 1963 comedy The Mouse on the Moon. --Anonymous, 21:59 UTC, August 25, 2008.
Not - so - you don't need to expend energy to "keep the craft from falling to Earth in the meantime" - you can increase your distance from the earth by flying in a gradual spiral path - you don't need to expend energy to avoid falling. Theoretically, the energy you need to expend to thrust out of orbit gradually is exactly the same as blowing it all up in a couple of minutes to get escape velocity. It's slow, boring - but possible - although if you do that, you'll still have to reach escape velocity - but you could do it gradually. Remember: The ISS has been maintaining its distance from earth without falling back OR expending energy for years. SteveBaker (talk) 02:59, 26 August 2008 (UTC)[reply]
You do need to thrust to maintain position, see gravity drag. A gradual spiral path only works once you're outside the atmosphere, inside it atmospheric drag would make it extremely inefficient. Also, at the risk of being pedantic, the ISS has required frequent boosts from other craft over the years to compensate for drag caused by the very thin, but still present, atmosphere at it's altitude. --Tango (talk) 20:23, 26 August 2008 (UTC)[reply]
Okay, I concede that once you reach orbital speed you don't need to expend energy to maintain altitude. But until then you do. --Anon, 05:21 UTC, August 27.
Woah there. In THEORY to "escape the earth's gravity" you need that much speed - but to escape earth's gravity you have to travel an infinite distance away - which is fine if this is a thought experiment that requires me to travel an infinite distance but doesn't let me have infinite fuel. But otherwise - for any practical purpise - it's bogus - you'll enter the gravity well of some other object sooner or later so you DON'T need escape velocity to get wherever you want to go...and even with escape velocity, the friction due to teeny-tiny amounts of gas and dust will eventually slow you down below escape velocity and you'll fall back down again.
For any practical mission - those infinities are irrelevent. If I only have to get a hell of a way from earth (to the nearest star - or to the next galaxy over to the left) - then with enough fuel we can do it all at 10mph. Sure, it's a lot of energy - but it's not forbidden by the laws of physics. I can get anyplace in the actual universe without needing to attain escape velocity - and having escape velocity is not guaranteed to be enough to reach some places in the universe because of friction.
Sure, it's currently more efficient to set off a gigantic rocket more or less all at once - hit escape velocity and then coast - but that's because of the nature of the technology we're currently using. With energy extracted from sunlight and relativistic rocket exhaust velocities (think Ion drive) - or solar sails powered from gigantic lasers - or magnetic ram-scoops it's OK to thrust continually and never come close to escape velocity - and you'll still get where you're going.
So the correct answer to the OP's question is "Sure you could thrust your way out there" - not "No, you can't". SteveBaker (talk) 02:54, 26 August 2008 (UTC)[reply]
The OP's question regarded doing it with an aeroplane, not some hypothetical future technology. --Tango (talk) 20:23, 26 August 2008 (UTC)[reply]
We're allowed a "specially fitted F-22" - I decided to fit an ion drive and some solar sails. :-P SteveBaker (talk) 21:42, 26 August 2008 (UTC)[reply]
Regrettably, neither of which are likely to do the job, since they don't work all that well inside the atmosphere! ;) --Tango (talk) 22:08, 26 August 2008 (UTC)[reply]
Would I be correct to say that you need a sufficient amount of energy to escape the dominance of Earth's gravity? So starting from ground level, a rocket acquires a large amount of kinetic energy (it accelerates to escape velocity) and this kinetic energy gets traded for gravitational potential energy (the rocket flies higher) - and at a certain point Earth's gravity becomes just a small factor considered along with all the other small gravity factors (Jupiter, Alpha Centauri, etc.)?
The "do it at 10 mph" thing seems somewhat of a canard, moving through the atmosphere at 10 mph requires a definite amount of energy expended as friction (drag), whereas moving through space at 10 mph requires almost no energy expenditure if you're moving in an orbit - but moving upwards at 10 mph needs a whole bunch of energy.
I struggle with these concepts, so please fill me in. Let's assume that the F-22 has a constant succession of refueling tankers showing up just in time, and there's another F-22 flying beside it with a big oxygen tank so the engines can maintain thrust. Franamax (talk) 07:04, 27 August 2008 (UTC)[reply]
Yes, the minimum possible amount of energy required to reach a certain altitude is given by the gravitational potential energy at that altitude. But because the earth's gravitational field gets weaker the higher you go, the energy required to get to INFINITE altitude can be imparted on the object as kinetic energy on launch - hence, you can get an object moving at escape velocity - and it's kinetic energy will exceed the amount of gravitational energy it'll have - even at infinite altitude. But (and this has been my point to the nay-sayers) there is no reason why you have to expend all of that energy at the get-go. You can also get to an arbitary height by expending the energy more gradually.
Certainly, doing it slowly will require a lot more energy in practice than in theory - and that's why we don't usually do that. But certainly, your thought experiment with the F22, oxygen tanks, etc, etc would work. And (as I said WAY up the top of this discussion) spacecraft like the (failed) British HOTOL would have done exactly that. The HOTOL was intended to use engines that would use atmospheric oxygen - like an F22 - up to whatever altitude that works at - then start using on-board oxygen tanks to turn the engines into rockets in order to fly up to orbit. The HOTOL failed for reasons relating to funding - it's clear that it's possible technologically. So it's more than just a thought experiment - you really could build a practical space-plane that would fly conventionally up to enough height and then keep on going. SteveBaker (talk) 13:59, 27 August 2008 (UTC)[reply]

August 25

Frequency of Meiotic Crossover

OK, if human gametes always acquired their chromosomes intact, then I'd have, on average, 11.5 from each of my grandparents, 5.75 from each great-grandparent, etc. Thus if I went back in my pedigree 7 generations—and with the further simplifying assumption that all of my ancestors were unique—then since there are 128 people back there and only 46 opportunities, the expected contribution from any one of them to my genome would be zilch. And seen the other way round, any one of my descendants 7 generations hence could expect to inherit nothing from me.

But, of course, meiosis doesn't work so tidily. So my question is, in practice how far off is the above reasoning? To state it more rigorously, how likely is it that any particular human gamete is free of crossovers, that each of its 23 chromosomes is simply a complete copy of one from the homologous pair?—PaulTanenbaum (talk) 03:41, 25 August 2008 (UTC)[reply]

The crossover rate varies by chromosome, but one would expect in general one crossover per chromosome. So one would have to go back more than seven generations to be reasonably certain that at least one ancestor in that generation had no genetic contribution to your makeup. - Nunh-huh 03:52, 25 August 2008 (UTC)[reply]
So, for instance my son's X chromosome likely contains some scraps from both of my in-laws?—PaulTanenbaum (talk) 04:04, 25 August 2008 (UTC)[reply]
Yes. And, in general, though not invariably, your son's autosomal chromosomes will likely contain some scraps from all four grandparents. The bigger the chromosome, the more likely there was a crossover event. - Nunh-huh 04:22, 25 August 2008 (UTC)[reply]
The number of crossover events per chromosome is higher than Nunh-huh suggests. According to this book, "On average, between two and three crossover events occur on each pair of human chromosomes during meiotic division I." I was surprised by the confirmatory answer to the question about the likelihood of crossovers between the X and Y chromosomes, but Nunh-huh is right on this one - there are indeed tiny regions in the Y chromosome which do cross over with their X chromosome counterparts, see this figure from this article. --NorwegianBlue talk 18:17, 25 August 2008 (UTC)[reply]
While it's true that there is a small area of the Y & X chromosomes that are homologous, and so permit crossover, it's not pertinent to the question asked about Paul's son's X chromosome, which came from Paul's wife (the crossover here occurred between the two X chromosomes of the mother, each of which contain portions of an X chromosome from her mother, and an X chromosome from her father, again due to crossover). - Nunh-huh 03:13, 26 August 2008 (UTC)[reply]
Nun-huh is right about the point of my question. But I do appreciate NorwegianBlue's info about crossovers between X and Y, and the pointer to the NIH article is extremely helpful. Muchos gracias to both of you!—PaulTanenbaum (talk) 03:25, 26 August 2008 (UTC)[reply]

Human growth hormone production.

Is it possible to have normal growth hormone function without a pituitary gland? Danlius (talk) 05:14, 25 August 2008 (UTC)[reply]

Highly unlikely without treatment, considering GH is secreted by somatotropes, which are found in the anterior pituitary. If one has no somatotropes, then one will almost certainly suffer from a Growth hormone deficiency. However, you should ask an endocrinologist if you want a definitive answer about what is and is not medically possible. Dostioffski (talk) 05:36, 25 August 2008 (UTC)[reply]

Cooked meat and air in a refrigerator

Take two slices of cooked ham. Put one slice in an open-air container, and put the other slice in a container with a lid. Now put both ham-containing containers inside the refrigerator, and leave the two containers in there for two days. At the end of those two days, the ham in the open-air container will have become hard (in texture), while the ham in the closed container will still be soft. Why? —Lowellian (reply) 08:11, 25 August 2008 (UTC)[reply]

Because a refrigerator efficiently removes moisture from the air, and the dry air efficiently removes moisture from the meat. Franamax (talk) 08:28, 25 August 2008 (UTC)[reply]
A fridge may speed up the process, but it isn't necessary. See Dried meat. --Shaggorama (talk) 14:14, 25 August 2008 (UTC)[reply]
Beef aging (in particular, dry-aging) is also close enough to be of interest. -- Coneslayer (talk) 18:04, 25 August 2008 (UTC)[reply]

Alcohol-and-cigarette-based human combustion

In the film Better Luck Tomorrow, there is a scene where a character consumes a lot of alcohol, and another character jokingly warns him not to light a cigarette in his mouth because he "might explode". Is it actually possible to drink enough alcohol to combust by lighting a cigarette in one's mouth? —Lowellian (reply) 08:30, 25 August 2008 (UTC)[reply]

Does a lighter explode when you light it just because it is full of ethanol and there is a flame at the top? The simple answer is 'no'. You would have to drink pure alcohol (not a dilute like whiskey or vodka), and LOTS of it, because by the time it got into your body it would be diluted by whatever is in there. You would die long before you could drink enough pure alcohol for your body to be over the 'inflammable mark' of 100% proof. Alcohol fumes in the breath may be a very unlikely source of a flame, but certainly no explosion. Like setting your farts on fire.--ChokinBako (talk) 10:06, 25 August 2008 (UTC)[reply]
Flammable materials don't always explode. There needs to be just the right mixture of material and air/oxygen. This is why grain silos can explode. I doubt you would ever be in a situation where your body was in danger of exploding due to alcohol consumption. Combustion does not always imply explosions, but more often burning. It is entirely possible to ignite alcohol that is in your mouth, similar to what fire-eaters do, but unlikely that you would be able to ignite the alcohol in your body. --Russoc4 (talk) 14:50, 25 August 2008 (UTC)[reply]
According to alcohol poisoning, drinking enough to make alcohol compose 0.55% of one's blood will kill about half the people who try it, so let's use double that (1.1%) as the maximum amount of alcohol you can reasonably have in your blood. Now pour a 1.5 oz shot glass of pure alcohol into a gallon bucket of water, which by RefDesk serendipity comes out to give the mixture a 1.17% alcohol level, and consider trying to light that bucket of water. --Sean 15:53, 25 August 2008 (UTC)[reply]
I think "reasonably" is a poor word to use in talking about a probably lethal dose of alcohol. Wanderer57 (talk) 17:24, 25 August 2008 (UTC)[reply]
I think you could probably hold enough alcohol vapor in your mouth to give yourself a pretty bad burn. Especially if your nose neighbor was doused. Plasticup T/C 18:52, 25 August 2008 (UTC)[reply]
I agree with Russoc4 that it's impossible to render a human body flammable by consuming alcohol – we've got too much water in our bodies – but I think we might be choosing the wrong interpretation of the question. Instead of the entire body exploding, could we conceive of a case where there was sufficient high-proof alcohol (and alcohol vapor) remaining in the drinker's mouth to result in a fire or small explosion when triggered with a lit cigarette?
There are a quite a number of alcoholic drinks which are flammable, some which are deliberately set alight, and a few which are even deliberately ignited in the drinker's mouth (e.g. the Sambuca volcano—Attempt at your own risk). In any event, it's certainly possible to ignite some hard liquors in a person's mouth. Can you do it with a cigarette? That's a tougher question. Is is possible to retain enough alcohol and fumes in the mouth after swallowing? Also tough to answer (though high alcohol content would help—think Bacardi 151, or straight grain alcohol...). In principle, if one had the right mix of ethanol and air in one's mouth and lungs, it could ignite violently enough to be described as an explosion. If it also ignited liquid alcohol in the mouth, it could cause serious burns. TenOfAllTrades(talk) 19:11, 25 August 2008 (UTC)[reply]
Fire eating might be of interest here. The situation we're talking about would be fire eating gone horribly wrong! --Tango (talk) 19:22, 25 August 2008 (UTC)[reply]
According to our flash point article, ethanol has an autoignition temperature of 365°C. According to this site, a cigarette has a temperature of 400°C. I'd say that if you held a shot of grain alcohol in your mouth and stuck a lit cigarette in there, you could create the local conditions for autoignition and the ethanol flashpoint of 12.8°C would be more than enough to burn you badly. Just to be sure, have someone else draw on the cigarette when they put it in your mouth - that way you will have a friend in the hospital bed beside you. Franamax (talk) 23:22, 25 August 2008 (UTC)[reply]
I would say that the most dangerous part of this is not that the body literally "explodes", but rather that you can get fairly severely burnt. If you drink alot of alcohol, you might pass out to the degree where if the cigarette where to fall out of your mouth and set your clothes on fire, you might not wake up. This could prove fatal, or at the very least leave you with some nasty burns. Be careful with your booze, people! 90.235.4.253 (talk) 02:06, 26 August 2008 (UTC)[reply]

power system

why 15 nos. disc insulator connect for 220kv o/h line? —Preceding unsigned comment added by Manir b4u2002 (talkcontribs) 08:45, 25 August 2008 (UTC)[reply]

Please write more clearly - it's impossible to understand what the heck your question is! Something about disk insulators on 220 kvolt overhead lines....but what is "15 nos." ? SteveBaker (talk) 13:59, 25 August 2008 (UTC)[reply]
(to Manir b4u2002) Also please sign your post in future. Otolemur crassicaudatus (talk) 16:25, 25 August 2008 (UTC)[reply]
Guys, there is no need to be so snotty with someone who is asking a good faith question and is just having trouble expressing himself. My interpretation of the question is that it is regarding the stack of insulators used on overhead power transmission lines and why there are that number of them. The insulators are made of glass or ceramic or a modern composite depending on age of the installation. Each disc is able to insulate around 15kV without arcing over. Consequently for the high voltages used in power transmission a stack of them needs to be used. Our article here shows a 275kV pylon with a stack of 18 discs. For 220kV, 15 discs, as Manir states, would be about right, but there are no examples I can go and look at locally as 220kV is not a standard voltage used in the UK. The number of discs required depends also on other factors such as the material and the local climate (damp countries need more than dry deserts). SpinningSpark 18:17, 25 August 2008 (UTC)[reply]
And that also accounts for the "crackly" sound you often hear on rainy days when walking near high-voltage lines - current leakage over the surface of the insulators. Franamax (talk) 22:59, 25 August 2008 (UTC)[reply]
No. I rather think that sound is caused by corona discharge —Preceding unsigned comment added by ThornCider (talkcontribs) 00:22, 26 August 2008 (UTC)[reply]

Medical Term for No Fear in Crisis.

I read in some medical publication of a woman who instead of flight, chose to fight in all emergency situations. There was an example where she was alone in her house and an intruder broke in. Instead of fleeing and getting help, she says that the impulse takes over immediately and she went after the intruder with a wooden spoon. She managed to chase away this intruder but afterwards she realized that she could have put herself in extreme danger. Then this publication named her condition as some medical condition and states than so many percent of Americans have this "disorder". I don't remember the name of this condition. Please help. --Anilmanohar (talk) 13:20, 25 August 2008 (UTC)[reply]

I didn't know that the fight-or-flight response was a now considered a 'disorder'; I thought it just meant that her nervous system was working. TenOfAllTrades(talk) 13:38, 25 August 2008 (UTC)[reply]
The OP sounds like the article stated that she actually lacks the fight-or-flight response and only possesses a will to fight. -- MacAddct  1984 (talk &#149; contribs) 14:49, 25 August 2008 (UTC)[reply]

Yes, that is correct Macaddct. She only had a will to fight as if she had no choice. --Anilmanohar (talk) 15:14, 25 August 2008 (UTC)[reply]

The decision of whether to run or stand and fight is often made in a fraction of a second based on all kinds of things. If you're in your house, you may well not have anywhere to run - presumably the intruder came through the door, so is between you and the door. Deciding to stand and fight when others would most likely run would be an example of courage (possibly combined with stupidity!), I've never heard of it being considered a medical disorder. --Tango (talk) 19:09, 25 August 2008 (UTC)[reply]

Ok, I just thought about it more. Is there a medical condition in which a person creates more adrenaline than a normal person would? I mean if you have high amounts than normal of adrenaline coursing through your body, it could lead to unusual aggression? I was also thinking of the Geronimo Syndrome, but that is just an unusual desire to jump, I think. --Anilmanohar (talk) 13:12, 26 August 2008 (UTC)[reply]

Is copper(II) ion more stable than copper(I) ion?

Is copper(II) ion more stable than copper(I) ion? If so, why?Tetsuya26 (talk) 14:14, 25 August 2008 (UTC)[reply]

Hi Tetsuya! You should check out our ion and copper articles, then get a healthy snack, then go finish your homework. Let us know if you still have questions after reading those articles. --Sean 16:02, 25 August 2008 (UTC)[reply]

Yes but why is cu2+ more table? —Preceding unsigned comment added by Tetsuya26 (talkcontribs) 18:13, 25 August 2008 (UTC)[reply]

Something to do with its electron configuration. Look for all the usual things: spin multiplicity, filled or half-filled shells, etc. DMacks (talk) 14:44, 27 August 2008 (UTC)[reply]

Useful life of commerically packed grated ginger

I've occasionally purchased chopped garlic packed in oil. I'm pretty sure that this product is safe to open, use a little, and store the remainder in the fridge, using it as needed over an extended period of time. Recently, I've purchase grated ginger in a very similar looking package. It is not packed in oil. The ingredient list includes a preservative, the name of which I can't remember. Do you think this product will be stable in the fridge once opened, over a period of months? The labeling doesn't give any indication. ike9898 (talk) 16:54, 25 August 2008 (UTC)[reply]

Check the label again, is really should have storage instructions on it. If there is no "use within X weeks of opening", then it should be fine until the best before date. I'm not sure there's any need to refrigerate ginger, but the packet should say. --Tango (talk) 19:13, 25 August 2008 (UTC)[reply]
Yeah, but it doesn't which is why I asked. ike9898 (talk) 19:50, 25 August 2008 (UTC)[reply]
Unfortunately, personal experience only, it's only "fresh-n-nummy"(c) for 1-2 weeks. Saintrain (talk) 21:40, 25 August 2008 (UTC)[reply]
I'm more concerned about the garlic in oil, I thought that was a good place for botulism to develop. Per Saintrain, grated anything will likely oxidize pretty fast and lose its flavour. It will still be "good" in a month, but you won't taste anything. Franamax (talk) 22:55, 25 August 2008 (UTC)[reply]
Not strictly answering the question but I can't see why anybody would ever need t buy grated ginger, surely it is always better to simply buy some ginger and grate it yourself? It will be fresher, have more flavour and probably have a much much longer shelf life. Jdrewitt (talk) 10:14, 26 August 2008 (UTC)[reply]
The same reason people buy ready grated cheese: convenience. (Admittedly, I don't believe I've ever bought ready grated cheese, but I've seen it in shops, so there must be someone buying it.) --Tango (talk) 20:25, 26 August 2008 (UTC)[reply]
Maybe they don't own a grater and are under 18 in the UK, thus unable to buy any knives to chop it instead (although they can join the army and get access to their cutlery and graters)? 79.66.44.182 (talk) 00:27, 27 August 2008 (UTC)[reply]
I'm not convinced (unprocessed) ginger would necessarily have a longer shelf life then packed grated/minced/crushed ginger with preservatives. Cost wise, it doesn't make much difference (as opposed to cheese) particularly if you're only using a small amount. Of course convinience as Tango alluded to is the biggest reason. And from my experience it does still have a fair amount of flavour after a month (kept in the fridge of course) Nil Einne (talk) 23:44, 27 August 2008 (UTC)[reply]

WTC floor plan

Referring to the floor plan image

A typical floor layout and elevator arrangement of the WTC towers

Why is there so much "empty" space around the actual occupiable space, and what was this "empty" space used for? --82.152.214.49 (talk) 19:47, 25 August 2008 (UTC)[reply]

If you mean the section of the diagram that is in yellow, it is labelled for "open plan office", thus would be filled with cubicles or something similar. That is the actual "occupiable space" the central core held elevators, lavatory facilities and such. --LarryMac | Talk 19:51, 25 August 2008 (UTC)[reply]
The yellow area might even have walls that look like 'real' walls, but are not structural and can be reconfigured to match the needs of the renter of the space. ike9898 (talk) 19:53, 25 August 2008 (UTC)[reply]
And here's a stubby article about Open plan offices. --LarryMac | Talk 20:12, 25 August 2008 (UTC)[reply]
A company I worked for had a branch office in one of the twin towers, long before their destruction. On my only visit, I don't know what I had been expecting, but it was not the poky little suite I found. As noted above, on that floor (mid-80s, as I recall), floor-to-ceiling walls had been erected to form offices and suites. The walls served mainly to divide space between tenants, provide privacy, hold up pictures. I'm sure some large organizations had very impressive offices, all built within the framework of the space labeled "open" in the diagram. Cantor-Fitzgerald alone occupied five floors of One World Trade Center. — OtherDave (talk) 01:16, 26 August 2008 (UTC)[reply]

Why do we laugh at people who get hurt

http://www.youtube.com/watch?v=B0PEIMGgKH4

^Watch this video. Most people would find it funny even though that kid was pretty hurt. But why is it funny? ScienceApe (talk) 21:25, 25 August 2008 (UTC)[reply]

I don't think "most people" would find it funny. Maturity and empathy are big part of one's sense of humour. I'd say that 99% of western under 13s would find it funny. --Mark PEA (talk) 21:32, 25 August 2008 (UTC)[reply]
Yeah, I didn't find that funny. Someone just getting hurt isn't generally funny, someone getting hurt in an unusual way (particularly if it's through their own stupidity) can be funny. Schadenfreude will give you some more info. --Tango (talk) 21:42, 25 August 2008 (UTC)[reply]
Look at the comments section. Most people did find it funny. ScienceApe (talk) 21:46, 25 August 2008 (UTC)[reply]
Most people that post to the comments section of youtube found it funny - hardly a representative sample. --Tango (talk) 22:08, 25 August 2008 (UTC)[reply]
Indeed. Comment-posters on youtube aren't considered the sharpest tools in the box. This onion article hits the nail on the head, in my opinion. Fribbler (talk) 23:16, 25 August 2008 (UTC)[reply]
If you ever watched America's Funniest Home Videos, most of the videos which are considered funny are accidents where someone appears to get hurt. And adults, many of whom are probably considered mature, laugh at it. Tango, I guess it depends on what you consider to be "most people" of a representative sample. After all, what is representative? In fact by saying "Comment-posters on youtube aren't considered the sharpest tools in the box." could easily be argued that you are drawing an unverified assumption about comment-posters on YouTube. ScienceApe (talk) 00:46, 26 August 2008 (UTC)[reply]
Home video shows usually show people getting hurt in interesting or dramatic ways (at least, the ones in the UK do), not just falling off a bike (they might fall off the bike into a swimming pool, or something). "Representative" is a perfectly well defined word, and youtube commenters are not representative of humanity as a whole. Humanity as a whole can spell better, for a start! --Tango (talk) 02:17, 26 August 2008 (UTC)[reply]
Nope, they don't. At least not more dramatically than the video I linked. Nope. It isn't. You have to qualify representative. Representative of what demographic? Humanity as a whole? So according to you, a representative sample would be individuals from every single country in the world, which is absurd. Also, you were the one to make the "humanity as a whole" qualifier, not me. I used "most people" in the casual sense, not the literal sense, so really you are nitpicking words which isn't very helpful. With that logic, you made a broad claim that people getting hurt generally isn't funny, while based on what I've observed from other people's behavior, it is. However you don't have a representative demographic to support your viewpoint, other than your own opinion that you thought it wasn't funny. Actually most of humanity can't write in English, so that disproves your last point. ScienceApe (talk) 06:32, 26 August 2008 (UTC)[reply]


To me anyway most people does in fact imply humanity as a whole. You don't have to sample people from every single country but I would expect a sample from the major cultures. I don't for example, see how you can say most people do something if few Chinese, Indians and Africans (or perhaps South Americans/Muslims) do it. Thats why I personally avoid saying "most people" unless it's something I don't expect to not have a strong cultural bias. And I often challenge those who say "most people" to question whether they have considered other cultures. IMHO people should be more specific, and not group everyone together without knowing much about them. You could say; most Americans, most English first language speakers, most people from the Western world or whatever. Of course, again you should make sure what you're saying is indeed true. For example, if you're saying most people from the Western world, does it apply to the French, Germans, Spanish or Italians? P.S. And you're right, most people don't speak English P.P.S. I agree we have no way of knowing whether most people find getting hurt funny. P.P.P.S. In this specific case, "a lot of people" or "many people" find this funny would have worked without sandboxing people of large variety of cultures into one box. Nil Einne (talk) 23:32, 27 August 2008 (UTC)[reply]
Well that's nitpicking semantics then. Most people on YouTube did find it funny, and I was going off that. If you prefer, you can take the original statement to be based off of that instead of nitpicking semantics, which isn't very helpful. I've noticed some people here taking certain statements too literally, and then dissecting it. That sort of thing should be avoided, it just bogs down discussion into a meaningless debate on semantics. ScienceApe (talk) 02:02, 28 August 2008 (UTC)[reply]
I think that people sometimes find it amusing to see someone of a particular group they happen to dislike injuring themselves due to their own stupidity. Suppose that you dislike skateboarder/biker kids doing their stuff on your property (not my view - just an example) - there may be certain pleasure to be taken in seeing a kid, just like the ones who were rude and antisocial to you when you told them to move on getting injured performing a particularly dangerous, ill-thought-out, possibly drunken Jackass-style stunt in a public place... --Kurt Shaped Box (talk) 01:48, 26 August 2008 (UTC)[reply]
I would imagine that, for the U.S. immature teenagers who post comments on that video, the "humour" comes from the fact that the kid is described as "fat". There appear (from the side bar recommending other videos) to be a lot of YouTube videos of large children being hurt. Bart133 t c @ How's my driving? 01:40, 26 August 2008 (UTC)[reply]


Sometimes

Re: above; It does seem to be universal, when old farts watch kids (or puppies, fawns etc.) "growing up", the reaction is "Awwww. Hahaha." Toddlers sitting down suddenly, or Bambi on ice or watching a kid cook onions for the first time. When there's actual danger involved, climbing a bookshelf or playing with drano it's an entirely different reaction. Certainly there's empathy but there's something else too. Whatsit? Saintrain (talk) 21:55, 25 August 2008 (UTC)[reply]

Sometimes the more excessive or ridiculous the fall the funnier it can be perceived, up to a point. Most people would agree watching a bad car crash is not funny, but watching one of the Marx brothers take a frying pan to the face can be hilarious. Of course the latter is not "real" but the effect is related. Perhaps Slapstick is an article worth citing for that reason. Vespine (talk) 06:37, 26 August 2008 (UTC)[reply]
Calls to mind the saying, variously attributed, that comedy is tragedy plus distance. --98.217.8.46 (talk) 23:52, 26 August 2008 (UTC)[reply]

why do I look worse in photographs than in mirrors?

I'm not a very good looking chap but when I look in the mirror do I think that I look okay, and then when I look at photographs of me I think I look bad? I'm the same me, right? So why? —Preceding unsigned comment added by 86.128.219.33 (talk) 21:39, 25 August 2008 (UTC)[reply]

I don't really know, but see photogenic for some explanation of the opposite effect, it's probably the same thing in reverse. --Tango (talk) 21:42, 25 August 2008 (UTC)[reply]
It's not the same you. The one in the mirror is flipped left to right. Since that's the one you're used to, it can make actual pictures of you look somewhat strange. I don't know how much of the effect is caused by that as opposed to the explanations listed in the photogenic article. — DanielLC 00:38, 26 August 2008 (UTC)[reply]
You can experiment by using a photo editor such as the GIMP or photoshop to flip your photos of yorself. -Arch dude (talk)
My theory is that when person A looks at person B, (in general, not in an intimate situation) they usually look less carefully and for a shorter time than is polite or possible if A is looking at a photograph of B. Also, in the first case, B is likely to be moving, whereas a photographic image is fixed and more susceptible to detailed study. Because of these factors, A is less likely to notice "imperfections" of various types in B. Similarly, if A is looking at his or her self in a mirror, they are less likely to notice "imperfections" of which to be critical than when looking at their photo.
As I say, only a theory. Wanderer57 (talk) 05:26, 26 August 2008 (UTC)[reply]
But considering that A's image is also fixed when A is looking in the mirror, and A probably is applying detailed study to himself or herself, why would A be less likely to notice imperfections of which to be critical when looking in a mirror rather than in a photo? —Lowellian (reply) 11:49, 26 August 2008 (UTC)[reply]
Don't have much time now, but a common saying for the phenomenon 86.128.219.33 is describing is "the camera adds ten pounds". This blog thread has a discussion; I particularly like "your not supposed to eat the camera..." The thread links to this more credible webpage, which has a serious one-page explanation. --Bowlhover (talk) 08:22, 26 August 2008 (UTC)[reply]
Also have a look at: Perspective distortion (photography) - the lens itself provides a 'perspective' of reality. Sometimes that distortion is beneficial, sometimes it is not. As also noted a mirror is a constant changing image so doesn't capture 1/100th or 1/500th of a second of your face - at which point any number of problems could be evident such as your pose, lighting etc. I've heard that the 90-110mm range is consider a 'sweet spot' for portrait for photography. I have to agree that I also find most photos of me don't capture me as well as I think I look in real-life! 194.221.133.226 (talk) 13:08, 26 August 2008 (UTC)[reply]
It can depend a lot on lighting. Even in natural lighting a camera will perceive light and shadow differently than the naked eye. If care isn't taken with the lighting, Shadows can appear darker and "deeper" on film and change your perception of the shape of a face. Worse is if you're thinking of photographs taken with a flash that is only a couple inches away from the lens, that can cause all sorts of weird effects. APL (talk) 14:47, 26 August 2008 (UTC)[reply]

Human eggs

How many eggs does woman start off with, and what happens to those that are left over after menopause? —Preceding unsigned comment added by ThornCider (talkcontribs) 21:45, 25 August 2008 (UTC)[reply]

According to this (from Google), women have one or two million at birth (another, unsourced, Ghit claims they have as many as seven million while still in the womb), are down to four hundred thousand or so by puberty, and are lost at about a thousand a month after that. Menopause occurs when the supply runs out entirely. Algebraist 23:07, 25 August 2008 (UTC)[reply]
What mass and volume would that be? 190.244.186.234 (talk) 00:47, 26 August 2008 (UTC)[reply]
The human ovum is about 0.1-0.2mm across. I can't find any figures on mass. Algebraist 02:58, 26 August 2008 (UTC)[reply]
An ovum would probably have a density close to that of water (it's just a specialized cell - and cells are mostly water) - so we can make a rough estimate - an 0.1 to 0.2mm object will have a volume somewhere around 0.001 to 0.008 cubic millimeters - which is 0.000001 to 0.000008 grams. A million of them would weigh between one and eight grams and take up between one and eight cubic centimeters. For comparison, a US 1 cent coin weighs 2.5 grams. That seems perfectly reasonable.
SteveBaker (talk) 15:52, 26 August 2008 (UTC)[reply]
What, you're saying a woman is worth three cents? Well, I never! ;) Franamax (talk) 09:02, 27 August 2008 (UTC)[reply]
I'm pretty sure that's not what I said...um let me check...no?!?! SteveBaker (talk) 13:49, 27 August 2008 (UTC)[reply]

Infinity

I am aware of the mathematical idea of infinity, but what I want to know is: is there any physical entity in the universe that can be said to be infinite? —Preceding unsigned comment added by ThornCider (talkcontribs) 23:38, 25 August 2008 (UTC)[reply]

"Only two things are infinite - the universe and human stupidity, and I'm not sure about the former." (and variants, attributed to Albert Einstein). Confusing Manifestation(Say hi!) 23:43, 25 August 2008 (UTC)[reply]
"There is no limit to stupidity. Space itself is said to be bounded by its own curvature, but stupidity continues beyond infinity." (Gene Wolfe) Algebraist 00:16, 26 August 2008 (UTC)[reply]
Spacetime itself, possibly (it's unknown if it's infinite, and I don't know if you would count it as an entity). Other than that, I don't think so. There are some things in physics that end up with infinities in the maths, but that usually means we don't properly understand it (the density of a singularity in a black hole, for example). Negative temperature can, apparently, be thought of as infinite (or even more than infinite) temperature, but I've never understood that myself. The ranges of electromagnetism and gravity are infinite, but they both move at finite speed and the universe is a finite age, so no EM or gravitational field is actually infinite. That's about all I can think of that even comes close. --Tango (talk) 23:48, 25 August 2008 (UTC)[reply]

We don't really know. There might be. Infinite divisibility parallel universes and multiverse are interesting hypothesis's. ScienceApe (talk) 01:06, 26 August 2008 (UTC)[reply]

It a little tough to draw the line with "physical entity". The basic "measurements" of physical entities are the fundamental units: Mass, Length, Time, Current, Temperature, Amount of substance and Brightness. But clearly there can't be infinite lengths or times because the universe isn't old enough. Infinite amounts of the other things imply infinite mass/energy and therefore infinite gravity - and we couldn't stand that.
So - what's left? We pretty much have to resort to measurements that boil down to ratios of things where one of the things is literally zero.
We can say (for example) that the singularity at the heart of a black hole is infinitely dense. But that comes about from a boringly finite mass and a literally zero size - so the infinity really comes about because density is mass divided by size.
So anytime you can find something that has a property that is zero you can probably find a ratio that'll pop up an infinity. The ratio of mass of a photon to it's rest mass is infinite for example.
Dunno - it's a bit of a stretch. SteveBaker (talk) 02:18, 26 August 2008 (UTC)[reply]
Although a physical measurable property cannot literally be infinite, mathematical models of reality may in some circumstances give an infinite answer, and it may sometimes be possible to assign a physical meaning to these "infinities". There is a story of a mathematician, a physicist and an engineer who were each asked by a firm of architects to calculate the strength of an especially complex structure. All three got the answer "infinity". The mathematician recommended that one of his graduate students should research better mathematical models for solving the relevant non-linear differential equations. The physicist recommended that the architects should design a simpler structure. The engineer wrote a report which concluded "There appears to be an adequate factor of safety for almost all feasible scenarios". Gandalf61 (talk) 09:10, 26 August 2008 (UTC)[reply]
Renormalization is one example of that - if you "do the math" relating to things like quantum electrodynamics, then all sorts of ikky infinities keep popping up - but the mathematical "trick" of renormalization fixes that problem and gives you nice, finite results. That's not really an example of a physical infinity though - the infinity appears in the math - but NOT in reality. SteveBaker (talk) 21:39, 26 August 2008 (UTC)[reply]


August 26

Gas snorting

My freind said that if you keep snorting a mixture of 80% nitrogen and 20% oxygen you will eventually die. Is that true? —Preceding unsigned comment added by ThornCider (talkcontribs) 00:06, 26 August 2008 (UTC)[reply]

Yes. Algebraist 00:10, 26 August 2008 (UTC)[reply]
How does that link explain anything about the original question? ScienceApe (talk) 00:56, 26 August 2008 (UTC)[reply]
It was supposed to indicate that yes, you will eventually die. Breathing something almost identical to the earth's natural (dry) atmosphere isn't going to be very relevant though. Algebraist 01:00, 26 August 2008 (UTC)[reply]
I think your friend is having a bit of fun with you. Regular old air (like the one you are currently breathing) is about 80% nitrogen and 20 oxygen (with some argon, water and CO2 thrown in for good measure). However, eventually, we will all die (it's like that line from Fight Club, "on a long enough timescale, the mortality rate of everyone drops to 0"). If the question is "Will breathing 80% nitrogen and 20% oxygen kill you?", then no. But if the question is "If you only breathe 80% nitrogen and 20% oxygen, will you eventually die?", then of course, yes you will. But the cause of your death wont be the air you breathe. 90.235.4.253 (talk) 01:56, 26 August 2008 (UTC)[reply]
Actually, the air could be what kills you - oxidation is one of the major contributors to ageing (just take a look at how many adverts stress the anti-oxidants in their products). --Tango (talk) 02:19, 26 August 2008 (UTC)[reply]


Heh, wasn't there a strange craze for huffing pure oxygen a few years back as a supposed cure-all for various ailments and medical conditions? --Kurt Shaped Box (talk) 02:25, 26 August 2008 (UTC)[reply]
Of course, we have an article. Algebraist 02:28, 26 August 2008 (UTC)[reply]
I wonder if those customers would like to buy a bridge? ;) --Kurt Shaped Box (talk) 02:41, 26 August 2008 (UTC)[reply]
Someone already did. Algebraist 02:45, 26 August 2008 (UTC)[reply]
Yes, it's true. A surprisingly large percentage of the population did this in the seventeenth century, and they all eventually died. (Note: they snorted the mixture because they called it "breathing", and they must have eventually died since they are not still alive.) -Arch dude (talk) 11:01, 26 August 2008 (UTC)[reply]
QUOTING FROM previous post: "on a long enough timescale, the mortality rate of everyone drops to 0" Or is it that the rate rises to 100%? Wanderer57 (talk) 16:20, 26 August 2008 (UTC)[reply]
Yeah, you're right, I got that wrong :) The real quote is "On a long enough timeline, the survival rate for everyone drops to zero." 90.235.4.253 (talk) 18:04, 26 August 2008 (UTC)[reply]

We can provide no medical advice on Ref Desk, but I am concerned about the absence of carbon dioxide in the posited breathing mixture. Wouldn't the absence of CO2 suppress respiration? Edison2 (talk) 04:23, 27 August 2008 (UTC)[reply]

No - you breathe in a couple of lungfuls of N2 + O2, some of the O2 is absorbed and some CO2 is excreted. It's THAT CO2 that triggers the next breath - not the trace amounts of CO2 brought into the lungs from outside. SteveBaker (talk) 04:39, 27 August 2008 (UTC)[reply]

Any way to discourage my parrot from washing her head in her water pot?

It's just a minor annoyance but it's worth asking. She gets a shower with me every morning if she wants one, but she still feels the need to stick her head in her drinking water, splah around and make a mess. I've tried replacing her water pot with a tube feeder but she didn't like that at all.

Is it even worth trying to discourage her? You know what parrots can be like when they've been doing the same thing every day for years. This is a hyacinth macaw by the way, I've posted here before about her. Thank you. —Preceding unsigned comment added by 84.70.186.194 (talk) 00:43, 26 August 2008 (UTC)[reply]

Could it be boredom? Maybe she's trying to cool off - how hot is it? SteveBaker (talk) 01:51, 26 August 2008 (UTC)[reply]
A suggestion, not to stop the head dunking but to try to reduce the mess. Get a large open-top plastic container, such as is sold for storing clothes etc. Perhaps 30" long, 18" wide, 12" deep, just as an example. Put the water pot in the container. (Never having lived with a parrot, this suggestion may be completely useless.) Wanderer57 (talk) 05:37, 26 August 2008 (UTC)[reply]
Beautiful animals the hyacinths. I would never discourage my parrot doing things. Maybe she wants to tell you that she wants another shower? Try immediately spraying her after she splashes around. If she complains splashing was for another reason. Maybe then she has an itch? They get ticks so that could be the problem. --Ayacop (talk) 08:48, 26 August 2008 (UTC)[reply]

Cheap, small, low-power technology for detecting a short-range distance?

I was thinking today about how one could make a cheap, small, low-power technology for detecting whether the detector in question is within, say, an inch of a wall or other object, without touching it.

One obvious idea I had was to use laser diode and a detector like an optical mouse. I thought about just using a light detector but that wouldn't work at night (or under various lighting conditions). I thought about a laser inferometer but those seem fairly large and probably take a lot of power.

I imagine there must also be a way to use just high frequency radio waves but I imagine the power requirements would be rather high.

Anyway... just thinking about this. Any ideas? I'm no engineer. This is not homework, just something I was trying to think over. --98.217.8.46 (talk) 01:25, 26 August 2008 (UTC)[reply]

In playing with LEGO robotics, I've used both sonar (with a regular speaker and microphone!) and an LED/light detector approach. The former works pretty good - sound is slow enough that you can reasonably measure the time-of-flight so long as you use a really high frequency sound 'blip' and have a small, responsive, speaker & microphone. The latter is highly dependent on the color and shininess of the surface. Since the speed of light is so high - you can't use the time-of-flight of the light at such short ranges (an inch out and an inch back is about 1/6th of a nanosecond - and it's incredibly hard to measure that short an interval effectively. So you're down to measuring the reflected intensity - for that, a laser is a sucky solution because the orientation of the surface to the light makes a DRASTIC difference to the intensity...compared to a more omni-directional LED. I got better results with an IR LED than a red one - I didn't try other colors. But such a solution really gives very variable results depending on the orientation, shininess and color of the surface. Sonar seems to work best. Best of all is a fine whisker - but that requires touching the surface (albeit a VERY gentle touch). I can't imagine radio working better than light - you still can't use time-of-flight - and the reflectivity issues are much more problematic because radio waves go right through most surfaces without much reflection. You could maybe get away with millimeter-band stuff - but that sounds difficult. None of these approaches are going to be very accurate though.
I wonder if you could do something like using the guts of a "stud finder" - those use a change in dialectric constant - they are certainly cheap and at the one I have produces a handy digital output that ought to be easy enough to read with a microcontroller. I don't know how good they are on all kinds of surfaces though...they clearly can detect wood through sheetrock - but that probably means they don't detect sheetrock so well.
I think an awful lot depends on what you know about the kinds of surfaces you are trying to detect.
SteveBaker (talk) 01:50, 26 August 2008 (UTC)[reply]
Sonar-based distance finders (as Steve mentioned) are very cheap. The parts are very simple to remove and use for whatever purpose you like. -- kainaw 02:21, 26 August 2008 (UTC)[reply]
Would it work to project a diffraction pattern and "read" the resulting projection to determine distance? One might use a square aperture to detect a tilted surface, and vary the size of the aperture to improve accuracy for a variety of distances. I've not seen anything like this, so maybe it's just a bad idea. Scray (talk) 02:40, 26 August 2008 (UTC)[reply]
Assuming we're thinking of the same method, it's only possible to calculate the angle between a certain interference band and the central antinode in an interference pattern. Without measuring the distance along the wall from the antinode and the band, it's not possible to find the wall-to-sensor distance. --Bowlhover (talk) 08:13, 26 August 2008 (UTC)[reply]
Plus it would fail on a sloping surface. SteveBaker (talk) 15:35, 26 August 2008 (UTC)[reply]
If you know the orientation of the wall (say, you have a compass and know the general layout), you could bounce a laser off the wall at an angle and see which of a row of detectors it struck; a lit source farther from the laser means the whole array is farther from the wall. If you don't know the orientation, but do know that the wall is flat (on the scale of your detector), you could use two lasers at different angles and see how far apart their reflections onto your detector array are. You could minimize the power consumption by only pulsing the device for a few microseconds several times per second. --Tardis (talk) 15:23, 26 August 2008 (UTC)[reply]
You could use a laser to project a spot onto the surface - then use a pair of cameras to image that spot - the resulting stereographic information would yield a distance pretty readily. You could use very crude cameras - such as the one the Wiimote uses. If lasers are cheaper than cameras - then you could use two lasers firing off parallel rays to project two laser dots onto the surface - then one camera could look at the distance between the two dots in "image space" and let perspective give you the range. With two InfraRed lasers, you might even be able to use an unomodified Wiimote (it uses bluetooth to talk to the Wii and the protocol it uses is well-documented). In a robotic application, you'll find the accelerometers in the Wiimote handy too!
More lateral thinking: What about using the mechanism in an optical mouse? Those are amazingly cheap. They contain a cheap, very low-res camera that measures the shift in 'texture' of your desktop over time to determine the speed and direction that your mouse is moving - but if you lift the mouse off the desk, the image becomes very blurry - no "texture" is detected and the mouse ceases to detect motion. That seems to happen at about one inch from the desktop. You could stick a mouse-mechanism onto the front of your machine and make it slowly vibrate somehow. So long as it reports no motion, there is nothing nearby - as soon as it "sees" motion, you're too close. It might fail if the surfaces you are measuring distance too are super-smooth and plain...but maybe you don't care about that?
SteveBaker (talk) 15:35, 26 August 2008 (UTC)[reply]
You have a better optical mouse than I do, Steve! It's hard to hold it steady enough to tell definitively, but mine seems to stop working at about 5mm off the (plastic) desk. (But that's still better than the ball mouse I also have, which doesn't seem to work at all when I pick it up…) --Tardis (talk) 15:48, 26 August 2008 (UTC)[reply]
I actually studied optical mice when I was looking for cheap navigation systems for LEGO robots. The chip that they all seem to use comes with a plastic lens that focusses the light at the plane of the desktop. Theoretically, every optical mouse on the planet ought to have the focal plane of the camera at the exact same height above the desk. However, I've found (as you just have) that there is some variability going on between one mouse and another. Perhaps they don't all use the plastic lens that comes with the chip - maybe to save money or allow some different internal arrangement for the mouse guts or...something...who knows?! Anyway - by replacing the super-short focal length lens that comes with the mouse for a longer len would allow the OP to choose the range at which it would accurately focus. You'd probably need a brighter light source too.
The distance at which it 'gets motion' may also depend on the nature of your desktop. I have an old beat-up leather-topped writing desk - so there is probably more "texture" for the mouse to latch onto than a relatively pristine plastic desktop. What do you get if you hold the mouse over the printed page of a book or something? I can get more "altitude" by aiming a desk lamp under the mouse - so light falloff might also be an issue if your mouse had a dimmer LED than mine.
Anyway - it's clear that it's possible. Whether it's good or easy in practice is a harder call.
SteveBaker (talk) 16:36, 26 August 2008 (UTC)[reply]
Cats and rats use their vibrissae (whiskers) as highly reliable short range proximity sensors. Cars of the 1950's had curb feelers which served the same function. Electro-optical curb feelers are discussed in that article. In the movie Dam Busters, bombing planes had to release the bombs at an exact height above lakes. A pair of spotlights and predetermined angles converged on the water surface when the plane was at the exact altitude. It would be a simple electronic process to use a pair of light beams to determine distance, or a light beam and a directional sensor or lens plus optical sensor. Bats and dolphins and some blind people use a form of sonar to echolocate and determine distance from objects. Edison2 (talk) 04:16, 27 August 2008 (UTC)[reply]
Whiskers would indeed be the simplest solution (Biology usually gets it right!) - but the OP specifically rules out touching the surface. SteveBaker (talk) 15:06, 27 August 2008 (UTC)[reply]
Using printed pages doesn't seem to change anything; one problem is that the mouse's light is directed at a shallow angle and thus misses the area under the sensor entirely with only a little vertical displacement. Running the mouse over the monitor (which is an odd concept) doesn't seem to give it any increased range, so perhaps it's the focusing issue. --Tardis (talk) 16:28, 27 August 2008 (UTC)[reply]

General relativity analogy thoughts.

I have only a rudimentary understanding of general relativity, but I've always had a problem when someone tries to use the bowling ball / trampoline analogy. You need gravity to make the bowling ball / trampoline analogy work. I realise this is used as a simple analogy, but using gravity in an analogy to explain gravity is, well, not much of an analogy if you ask me! It's a bit like saying: The night was so dark, it was as black as night! So I was thinking about this and came up with what I think is a better analogy, which admittedly is not quite as simple, but it goes something like this: imagine space-time has a uniform "pressure" or "density", of course these terms are usually applied to things which have mass already, but imagine space-time has a quality which is analogous to "density". Any mass existing in space-time creates an area of low space time density, less dense the closer you get to the mass. So just like a balloon in air, any mass is drawn towards areas of least density, or pushed away from areas of higher density, which ever way you want to look at it. So that when two objects interact, the are drawn towards their "low density zones", the bigger will obviously have a larger zone. However unlike a balloon, the pressure areas are not defined by a membrane, but actually exist as a gradual pressure gradient, dropping off the further away you get, just like how we currently observe gravity. So inverse of the high pressure atmosphere created by our gravity, the atmosphere it self is stuck to our planet by the low pressure time-space near our planet. If you take the analogy one step further, could matter it self be condensed space-time? The area of low "space-time pressure" is in fact the "gap" left in space time by the mass it self coalesced in the middle? I'd be happy to hear what anyone more familiar with general relativity would have to say. Vespine (talk) 02:32, 26 August 2008 (UTC)[reply]

I'd just like to say I like the fact that you pointed out that problem. Clever. I didn't read the rest of it, sorry =P --mboverload@ 03:06, 26 August 2008 (UTC)[reply]
I don't think your analogy really works. Things move towards areas of low pressure in order to equalise the pressure and remove the gradient, but when things move towards a massive object they actually make the object more massive and increase the gradient. I'd stick with the trampoline analogy, while the reasons behind why it works don't make much sense, the outcome is a very good approximation of the real thing - you can even get marbles to orbit the bowling ball in realistic orbits (although they decay rapidly due to friction). I'm not sure your analogy would be as good at something like that, and it's also much more difficult to understand - people know exactly what happens with bowling balls on trampolines. To be honest, I think you just have to accept that GR is difficult to understand and there's not a lot you can do to make it easier! --Tango (talk) 03:07, 26 August 2008 (UTC)[reply]
It doesn't matter whether I'm familiar with GR or not because GR is not about analogies or allegorical heuristics - these are just little things to understand some general concepts that don't even matter that much. Gravity is, as far as we know, a field of interaction just like electromagnetism (using gravitons instead of photons). The "rubber sheet" analogy is useful because gravity, unlike EM, is a one-way force in most applications - it can only attract. But the mathematics is general enough that similar tensor equations can be used to solve both GR and EM problems. Mass occurs by the Higgs mechanism, as far as we know, which is really not anything like you described. Mass is also concentrated space-time pretty much by definition of gravity until we get quantum gravity worked out. The pressure analogy sort of works, because that's sort of how an interaction field works, but not really. SamuelRiv (talk) 13:05, 26 August 2008 (UTC)[reply]
The bowling ball / trampoline analogy is actually a good model of Newtonian gravity, even though, as you say, it looks like it's begging the question. The gravity well article explains the math behind it. It's not a model of general relativity. A lot of people confuse the curved trampoline with curved spacetime, but they're very different. So don't try to understand general relativity that way, you never will.
It might be possible to model general relativity as a local variation in the "density" of spacetime, but the simplest version of that idea won't work. Newtonian gravity is a scalar field theory, meaning that the field can be described by a single real number at each point in space(time). In the trampoline analogy, that number is the height of the trampoline at each point. Density is also a scalar. But general relativity is a tensor field theory: the field at each point is described by a symmetric 4×4 matrix. So you need more than just a density. You could do it if you allowed the "density" to be different in different directions (making it more like strain), and included time as one of those directions. But I think that model would be harder to visualize than the usual one (curved spacetime).
Could matter be condensed spacetime? Sure. A black hole of mass m and charge q interacts gravitationally and electromagnetically just like any other object with the same mass and charge, even though there's no "real" mass or charge anywhere to be seen. People often say that the mass and charge exist inside the event horizon (where they can't be seen), but we don't know that. Nothing behind the event horizon has any effect on the outside; spacetime could just end unceremoniously right inside the event horizon, and the part outside would still behave like it has mass m and charge q. So the universe doesn't need real charge; you can fake it with black holes. Applying Occam's razor, it's tempting to suppose that it doesn't have real charge. I'm a big fan of this everything's-a-black-hole viewpoint, but whether it has any merit remains to be seen. Nobody's yet concocted a working quantum theory along those lines, and not for lack of trying. -- BenRG (talk) 14:56, 26 August 2008 (UTC)[reply]

Given the same amount of sleep

Would you choose more or less REM sleep? I'm asking because a news article says people sleeping east-west may have fewer dreams than those north-south. Thanks. 67.243.6.204 (talk) 02:28, 26 August 2008 (UTC)[reply]

Wait, humans have their own compass? --mboverload@ 02:35, 26 August 2008 (UTC)[reply]
Maybe, according to magnetoception. I'd like to see this article, though – it sounds fairly likely to be nonsense. Algebraist 02:36, 26 August 2008 (UTC)[reply]
Quite a few people seem to believe in the 'sleep alignment' thing. I have no idea what it's actually called - but I think it's one of those new age-type beliefs... --Kurt Shaped Box (talk) 02:40, 26 August 2008 (UTC)[reply]
They believe it because <personal opinion> When you change position on your bed there are different pressure points/air flows/light intensities. I constantly switch my sleeping alignment and sleep better in different directions depending on the season/weather/my crazyness.</personal opinion>--mboverload@ 02:45, 26 August 2008 (UTC)[reply]
I think just about everyone does *that*. I think what the OP was getting at is the practice of intentionally aligning your bed (i.e. physically moving it) in the direction of the compass/chi flow/ley lines/earth's magnetic field/whatever. --Kurt Shaped Box (talk) 02:52, 26 August 2008 (UTC)[reply]
Yes, and it "works" for them due to the placebo effect in addition to the points I laid out above =). --mboverload@ 03:05, 26 August 2008 (UTC)[reply]
I'm not sure the placebo effect can do much when you're asleep - it requires concious thought. It's more likely to be confirmation bias. --Tango (talk) 03:08, 26 August 2008 (UTC)[reply]
I'm not convinced of that. Based on purely anecdotal evidence, it seems to work in sleep (I'd look for studies, but I'm behind a corporate content filter). ie. after reading about lucid dreams and out-of-body experiences and all that nonsense, I had a couple "lucid" dreams. Of course again, that's anecdotal, but I'm sure there's at least one study that has employed placebo controls and sleep. -- Consumed Crustacean (talk) 03:14, 26 August 2008 (UTC)[reply]
As far as I know, lucid dreams are a purely psychological thing anyway, so I'm not sure there's a meaningful distinction between a "real" effect and a placebo, any effect is going to psychological in nature. --Tango (talk) 03:43, 26 August 2008 (UTC)[reply]
[13] says "some studies suggest humans who sleep in an East-West position have far shorter" REM sleep. No sources cited. The article is about another study involving deer and cattle. This is an August 25 article, probably where the OP got the idea from. -- Consumed Crustacean (talk)
"Some studies suggest" Ah, the greatest cop out phrase in the history of fake and misleading news. Thanks for the detective work Consumed! --mboverload@ 03:18, 26 August 2008 (UTC)[reply]

TO ORIGINAL POSTER: According to Rapid eye movement (sleep) lack of REM sleep is bad. I would go for more, but not too much. You should qualify your question with a base amount of REM sleep you are considering.--mboverload@ 02:47, 26 August 2008 (UTC)[reply]

I don't know about east-west versus north-south, but I sleep much better if I am facing the door. I always assumed that it is a basic instinct that I need to face the entrance so as to be more easily alerted to a predator. My girlfriend teases me but if we are set upon by wolves I'll be the first to know. Plasticup T/C 13:23, 26 August 2008 (UTC)[reply]
There was a recent study that showed cows and deer on Google Earth tended to sleep facing magnetic north-south, but they didn't go so far as to say why (or whether it would apply to humans too, although they had originally planned to look at campers). Confusing Manifestation(Say hi!) 01:56, 27 August 2008 (UTC)[reply]
Yes, I read that news article. No, I don't believe in new-age voodoo. Yes, I understand that "some studies" is a copout. No, I don't have a baseline for my question. I know so little about the subject anyway. Thanks a lot for the interesting discussion though. 67.243.6.204 (talk) 04:26, 28 August 2008 (UTC)[reply]

This food recall is a little scary because of the 30% death rate and reconfirms for me how much better it is to avoid processed foods when I can just make it myself. My question here is, the news agencies are saying that the people at risk are the elderly, immune-compromised individuals, and pregnant women. Why pregnant women? Is it the woman who is at risk, or is there a likelikood that the bacteria can migrate to the fetus? Franamax (talk) 03:10, 26 August 2008 (UTC)[reply]

Please note that the vast majority of food poisonings occur because of UNprocessed foods, such as fresh fruit. I would much prefer my food made in a controlled factory environment with all ingredients irradiated and put through large amounts of sterilization first =) --mboverload@ 03:13, 26 August 2008 (UTC)[reply]
Nothing against you by the way, just an interesting tidbit. --mboverload@ 03:16, 26 August 2008 (UTC)[reply]
Yeah, I know where you're coming from there. The point though is that poisoning from unprocessed food generally comes from surface contamination - and I'm willing to take responsibility for washing my food. With processed food though, the bacteria/virus is more often right there in the inside of the food, especially with meat. Franamax (talk) 03:48, 26 August 2008 (UTC)[reply]
As for Listeriosis, it's both that the woman is at risk, and that an infection can complicate pregnancy. Specifically, most infections occur in the third trimester of pregnancy, because that's when Th1-mediated immunity is most severely suppressed. Pregnant women are 20 times more likely than non-pregnant persons to catch the infection. Listeriosis can cause miscarriages and premature delivery. About 22% of cases of perinatal listeriosis result in the death of the fetus. Listeriosis can be caused, as noted, by uncooked foods, but also by cooked or pasteurized food that have been contaminated between the pasteurization/cooking and the consumption. Pregnant women are usually counselled to avoid unpasterurized milk, soft cheeses, and cold luncheon or deli meats. - Nunh-huh 03:22, 26 August 2008 (UTC)[reply]
Hmmm - thanx. TH1 suppression during pregnancy, I wasn't aware of that. I'd be guessing (off-topic) that it is actually the fetus generating the immune-supression? Franamax (talk) 03:48, 26 August 2008 (UTC)[reply]
I think it's actually an estrogen effect - but possibly a literature search would turn up other factors as well. - Nunh-huh 04:27, 26 August 2008 (UTC)[reply]
Fetuses have no working immune system, see Immune system#Passive memory, they borrow antibodies from the mother. --Ayacop (talk) 08:36, 26 August 2008 (UTC)[reply]
Even in the third trimester? Plasticup T/C 16:50, 26 August 2008 (UTC)[reply]
Well yes, babies have passive immunity using their mother's antibodies (from the pregnancy and milk) for a while after birth. Third trimester is nowt to that :) 79.66.44.182 (talk) 00:10, 27 August 2008 (UTC)[reply]
Could be due to one or both of progesterone[14] and/or estrogen[15], probably both. Scray (talk) 04:55, 27 August 2008 (UTC)[reply]
My understanding is that the fetus itself produces hormones which affect the mother. For instance (though I'd he hard pressed to find the reference), the fetus produces substances which modulate maternal weight gain in early pregnancy. It seems not unreasonable to consider that the fetus also modulates maternal immune response. After all, the fetus is 50% foreign material, so to speak. Franamax (talk) 08:44, 27 August 2008 (UTC)[reply]

Curling a ribbon by scraping it?

When you run one side of scissors along a ribbon, it tends to curl up. I've seen this technique used when wrapping Xmas presents.

Is there an English word or Wikipedia article that describes this simple physics process, or this physical phenomenon? I've noticed it happen to hairs too when they are brushed with knots in them. And I'm sure it may be used in other aspects of engineering or fabrication. --206.248.172.247 (talk) 03:35, 26 August 2008 (UTC)[reply]

There does not appear to be a single word, though this article describes the mechanism behind the process. Dostioffski (talk) 05:30, 26 August 2008 (UTC)[reply]

Maglev

(Note: I moved this question down from the original date because I added a third question four days after the original posting that seems to be getting overlooked.)Lowellian (reply) 11:45, 26 August 2008 (UTC)[reply]

Are maglev trains generally built along the lines of the locomotive model or the multiple unit model? —Lowellian (reply) 02:44, 21 August 2008 (UTC)[reply]

The lift motor also generates thrust - since every car requires lift, every car produces thrust. So I believe they pretty much have to be multiple unit trains. But in another sense, the "motor" is the track - so in a weird way, none of the train is the power unit. SteveBaker (talk) 02:56, 21 August 2008 (UTC)[reply]
The locomotive article has a section on maglev (that heading entitled "Magnetic levitation") that gives some information about maglev but does not in the least relate that information to the locomotive or explain what any of it has to do with a locomotive. Perhaps that section should be removed? —Lowellian (reply) 18:45, 21 August 2008 (UTC)[reply]

How easily do maglev trains turn (go in reverse) compared to traditional wheeled trains? —Lowellian (reply) 04:07, 23 August 2008 (UTC)[reply]

For a maglev MU, what's the difference? It's just a question of the rotation order with which you actuate the drive electromagnets.
Atlant (talk) 14:19, 23 August 2008 (UTC)[reply]
So, suppose a maglev train was running along at, say, 300 mph. Could it just almost instantaneously reverse direction and go in the other way, still at 300 mph? —Lowellian (reply) 01:28, 24 August 2008 (UTC)[reply]
Could you instantaneously reverse? Of course not! But neither can a traditional wheeled train. What you could do, of course, is reverse the direction of the driving force and begin decelerating at (approximately) the same G-force with which the maglev originally accelerated. In fact, for a maglev (as compared to a wheeled train) that's really your only means of braking unless you dump the maglev system and let the train slide along the guideway in what's probably an amazing shower of sparks. (Wheeled trains have friction brakes on the wheels, axles, or directly engaging the steel track.)
Atlant (talk) 14:27, 24 August 2008 (UTC)[reply]

Are maglev trains the trains of the future, or is there some other competing advanced technology with the same potential? Fifty or one hundred years from now, is it likely that most trains will be maglev? (Yes, I know Wikipedia is not a crystal ball, but we're not in the main article space.) —Lowellian (reply) 08:19, 25 August 2008 (UTC)[reply]

From reading our Maglev (transport) article and this survey article, it seems that the answer is "probably not". The lower operating costs, greater people-carrying efficiencies and environmental benefits of maglev trains are only realised over long distances. However, the high costs of constructing a new permanent way (from $100 million/km to $500 million/km) mean that a long-distance maglev network will require a huge initial investment, and will take a long time to break even. Short-distance point-to-point systems may be cheaper, but they don't seem to be economical. Just because something is technologically feasible doesn't necessarily mean it will become commercially viable - to coin a phrase, we could call this the Concorde syndrome. Gandalf61 (talk) 12:41, 26 August 2008 (UTC)[reply]
This is still the reference desk though and not the place for idle speculation. Gandald has some good points. Also, the high speeds maglev trains are capable of achieving is likely to be pointless for urban trains, the short distance between stations means you don't have enough time to accelerate (presuming you don't want to kill all your passengers). But more generally, we simply don't know. It will depend on what sort of new technologies we achieve and a whole lot of o ther stuff (e.g. how much energy ends up costing). E.g. room temperature superconductors will help a lot but I'm not convinced they are possible. IIRC last time I read the article, there is also some techs using more permanent magnets but they're still in initial development. Nil Einne (talk) 14:27, 26 August 2008 (UTC)[reply]
There is a general trade-off between Speed and Cost:
  • Maglev's produce an almost frictionless surface - speeds are high - but costs are astronomical.
  • Conventional steel wheels on steel rail is still fairly low friction - speeds can still be reasonable (150mph to 200mph is no problem) - and it's cheaper. Monorails are pretty much a similar deal.
  • Rubber on concrete is higher friction - speeds over 70mph start to become problematic - but it's very cheap.
  • Dirt tracks and draft animals...?!
Getting rid of friction costs money but it gains you speed. But over a short run, you don't need the speed (and you can't get it without high accelerations - which are uncomfortable for the passengers). So you end up with this uncomfortable problem that high speed is only useful over long runs when the track costs kill you. Over short runs, you can maybe afford the high cost of the track - but you don't need (and can't use) the speed - so why pay extra?
So you find that steel rail works best over most of the speed/distance envelope - with rubber-on-concrete being used for really short runs like the trains that run around between airport terminals and such. Monorails tend to be used mostly for reasons of novelty (eg at Disneyland). Under-slung monorails may offer advantages in tight urban areas where they have to be added as an afterthought into existing infrastructure.
Hence, no maglev trains. It's a solution in search of a problem.
SteveBaker (talk) 15:13, 26 August 2008 (UTC)[reply]
SteveBaker, are you saying that the lower-friction of steel-on-steel results in higher speed capability? I think this cannot be correct. See traction - if you want high speeds, you want high static friction between the wheel and the "road", and low friction between the wheel and the axle. Some other factor must be the limiter here. Nimur (talk) 21:27, 27 August 2008 (UTC)[reply]
"Friction" might not be the correct term, but it's a reasonably accurate description of the effect. The difference between rubber-on-concrete and steel-on-steel is the amount of deformation: a rubber wheel flattens on the bottom as it rotates, converting forward motion into heat in a very friction-like way. A steel wheel doesn't deform anywhere near as much, so there's very little energy loss. --Carnildo (talk) 23:12, 27 August 2008 (UTC)[reply]

Okay, thanks for the answers, though let me ask for a little bit more clarification: if maglev trains won't be predominant, what type of trains will be dominant then in the near (50-100 yr) future? Still the same steel-wheels-on-steel-rail trains as today? There's no other competing new technology besides maglev? —Lowellian (reply) 18:38, 27 August 2008 (UTC)[reply]

No, there isn't. There is no reason to expect anything but steel-on-steel to become dominant for the foreseeable future. (I would say that the foreseeable future in this context is more like 20-30 years than 50-100.) Trains using conventional steel-on-steel now run at up to almost 200 mph in regular service, with good energy-efficiency, and have been tested at over 350 mph. The same cost and political considerations for obtaining a route suitable to build a high-speed line apply no matter what the technology, but conventional trains can run onto conventional tracks, so the high-speed routes can stop at the city limits and trains can run through onto other parts of the network. It seems to me that the most likely thing to change this picture is a global war or energy shortage, and that would simply lead to the discontinuation of all high-speed trains, not the introduction of new technology. --Anonymous, 19:43 UTC, August 27, 2008.
— Preceding unsigned comment added by 208.76.104.133 (talkcontribs) 14:43, 27 August 2008 (UTC)[reply]

Why is 3d10 configuration so stable??

Why is 3d10 configuration so stable?? According to Hund's rule, pairing decrease stability ..So why is 3d1 where every electron is paired so stable?? —Preceding unsigned comment added by Tetsuya26 (talkcontribs) 11:54, 26 August 2008 (UTC)[reply]

"Stable" compared to what? DMacks (talk) 14:17, 26 August 2008 (UTC)[reply]
Hund's rule is saying that, once you have decided to place electrons in the same set of orbitals (like 3d taken as a whole), placing them in different orbitals gives a lower energy (better) state than placing them in the same orbital. However, you don't even see this in the spectroscopic notation: we just write 3d5 without bothering to say that this should be one electron in each orbital rather than, say, 2 full orbitals and a half-full. 3d10 is just fine because the only alternatives after you get to the obviously desirable 3d5 are to go to 4p or higher, which is worse. There are some near-degeneracies, though, where Hund's rule trumps the Aufbau principle; for example, copper is 3d104s1 even though 4s is "lower" than 3d. --Tardis (talk) 15:36, 26 August 2008 (UTC)[reply]

Geomagnetic reversal and the End of the World

I was watching a program about the End of the World (predicted to occur in 2012). In it, there was reference to Geomagnetic reversal and catastrophic (environmental, geologic, etc.)results when this occurs. I read the wiki artical but it is unclear if such a reversal would be "catastrophic" or if it would really make much difference in the grand scheme of things (other than wreaking havoc on our electronics!). Can anyone shed any light on this? —Preceding unsigned comment added by 216.154.16.106 (talk) 21:12, 26 August 2008 (UTC)[reply]

We talked about this just a few days ago. As I understand it, the problem is not that the field is reversed (ok - so your compass points the wrong way - I think we can deal with that!) It's that the reversal doesn't happen overnight. It might take years to decades for it to happen. During that interval, there might be either no magnetic field at all - or a wildly oscillating field. If the field were to "go away" for any amount of time, there is a risk that the mechanisms like the Van Allen belt might collapse and cease to protect us from hard solar radiation. A rapidly moving or oscillating field would induce electrical currents into large metal objects - and might do very bad things to electronics and such like. Since this kind of field reversal has happened MANY times in the past - certainly within the period since humans have evolved, it's not an "end of the world" event. Our species - and most others on the earth would survive. However, the number of deaths and associated problems with electronics and such might make it seem like a really major catastrophy to individual humans. But it's really hard to tell what all the ramifications of such an event would be on a technologically advanced civilisation. SteveBaker (talk) 21:30, 26 August 2008 (UTC)[reply]
The transition period is ~800 years, not decades. A terawatt geomagnetic engine does not stop (or reverse) on a dime. Dragons flight (talk) 02:11, 27 August 2008 (UTC)[reply]
Not really, we don't know exactly what would happen. The world has undergone plenty of reversals in the past and it's still here, so won't result in the end of the world, it might possibly result in the end of civilisation, but I doubt it (proto-humans survived the last reversal, so why can't we survive the next? We can protect ourselves with technology if we need to). Also, it's not going to happen in 2012 - it's a pretty quick thing by geological standards, but I don't think it's that quick, we would have detected changes by now if anything major was going to happen in 2012 (the changes we have measured suggest we've got another thousand years or two to go, at least). --Tango (talk) 21:32, 26 August 2008 (UTC)[reply]
Linear extrapolation from current trends suggest that a reversal could occur one or two thousand years in the future—there merest eyeblink on geological timescales, but a long, long time after 2012. (See geomagnetic reversal). Any effect on human society is purely speculative; penetration of the solar wind into the upper atmosphere may result in damage to electronic and electrical systems, and there may be an elevation in ambient radiation levels as more cosmic rays reach the Earth's surface. TenOfAllTrades(talk) 22:36, 26 August 2008 (UTC)[reply]
The year 2012 has become a popular choice by doomsayers based on interpretations of, among other things, the Mayan calendar. These doomsayers then tend to latch onto any possible explanation they can find for their predicted apocalypse, particularly things like planetary alignments and magnetic pole reversal, even if scientists know that these events are either not going to have the effect the doomsayers think they will (the gravitational effects of a perfect planetary alignment could be counteracted with a jumbo jet) or are almost certainly not going to happen as soon as 2012 (as TOAT points out, we suspect there will be a pole reversal in the next few thousand years, but thats three orders of magnitude bigger than the 4 years until 2012). Confusing Manifestation(Say hi!) 01:35, 27 August 2008 (UTC)[reply]

Keep a "hard(permanent) magnet" floating

I have searched the internet and read the article on magnets and magnetic fields, but there are still questions I feel unanswered.

Would it be possible for a sphere shaped "hard magnet" to float above another "hard magnet" or flawlessly being repelled whenever the magnet would touch or get close to the other magnets surface? I do know that monopole magnets are just hypothetic, so I wonder rather if there is a possibility to "wrap" a pole inside another or possibly balance the poles, so it's 100% controllable?

To help illustrate my point, you could imagine a series of slides standing in a row on a flat ground. I would like to drop a ball on the first slide, make it "bounce" on to the next, slide down, and bounce up to the next one again - all by just using regular magnets. Is this plausible?

Thanks in advance!

Kim, Norway —Preceding unsigned comment added by 89.10.23.121 (talk) 22:14, 26 August 2008 (UTC)[reply]

Not with ferrimagnetic materials alone. But you could some diamagnetic material (like bismuth orpyrolytic carbon) to stabilise the system. See Earnshaw's theorem. —Preceding unsigned comment added by LCMk2 (talkcontribs) 22:19, 26 August 2008 (UTC)[reply]
You mentioned a "spherical magnet." It would still have a north and a south pole, and if suspended over a north pole would flip so he south pole was down and crash into the magnet below. We know of no way to have a spherical magnet with the horth pole, say on the outside and the south pole on the inside, and no one has ever found a magnetic monopole, even though I don't think anyone has proved such to be impossible. Edison2 (talk) 04:02, 27 August 2008 (UTC)[reply]
He's referring to having north in the middle of the magnet and south on the outside, or vice versa. If this is done, the north and south will cancel each other out, and there will be no magnetism outside of the magnet. Besides using a diamagnet, you can spin the magnet, and use the gyroscopic effect to keep it balanced. These will both allow you to levitate the magnets, but the example you mentioned would be impossible because if the force is strong enough to push the magnet up to the next slide, it will be strong enough to keep the magnet from going down the slide. Get it? In general, if you think you found a perpetual motion machine, and you're not using general relativity, you messed up somewhere. — DanielLC 15:48, 27 August 2008 (UTC)[reply]

Lifespan of an American Spadefoot toad

I have had two male and one female American Spadefoot Toads for over seven years now. Seven years ago, my father told me they'd only live about three years. Is this generally true? --Ye Olde Luke (talk) 03:01, 26 August 2008 (UTC)[reply]

I don't know about this specific case, but it's not unusual for pets to live longer than the same animal would in the wild due to having a reliable source of food, protection from predators, etc. Perhaps your father was basing it on the life span of these toads in the wild. --Tango (talk) 03:12, 26 August 2008 (UTC)[reply]
A Google search suggests different wild populations of American Spadefoot Toads live from between 5 and 12 years. Dostioffski (talk) 05:40, 26 August 2008 (UTC)[reply]

An entirely new problem: they've escaped.

Somehow, my asking of this question seems to have correlated with my first real problem with the frogs.

Yesterday, I found that the frog case had been swarmed by ants. I rescued the three frogs, and relocated them temporarily to a large tupperware container filled almost to the top with sand. I placed a lid on the container, but not tight, since I wanted air to get in. Last night, the frogs were able to pop the top and escape into the house. This morning, I found the female and one male in the basement. I'm still searching for the other one. The family has made sure to keep all doors shut, so there's no way the frog can escape the house. My questions:

1) Is the final frog probably in the basement? Should I concentrate my search there?
2) Is the fact that two were in the basement relevant? Do spadefoot toads, who are burrowers, have some sort of inner sense helping them find the lowest place in the house?
3) After seven years of captivity, is the frog still capable of finding itelf food and staying alive until I locate it?
4) Will the frog move around? Or will it probably sit in the same place until nighttime? All they ever seemed to do is sit around while in the case.
5) Is there any way to set a trap for the frog? Maybe some type of cage with a cricket that the frog can enter but not exit (remember they can jump as well as burrow; I've seen it).
6) Anything else I should be aware of or take into consideration?

Thanks for any help you can give me. Depending on the answer for #3, time may be of the essence. --Ye Olde Luke (talk) 00:23, 27 August 2008 (UTC)[reply]

It will probably try and find somewhere damp, the basement may well qualify. You could try putting large tubs of damp sand (with food in, if possible) around the house that it can choose to burrow into (include one or two in the basement) and check on them every few hours until you find it. I doubt food will be an issue for a while, water will be more of a problem, but you've probably got some time to find it, especially if is succeeds in finding somewhere damp to stay. --Tango (talk) 01:42, 27 August 2008 (UTC)[reply]
I talked it over with my dad, and he's considering the idea, but he hasn't okayed it yet. We've decided towels would work better then tubs, to ensure that the frog can successfully get into the sand. Only problem is the frog eats live crickets. There's no concievable way to keep live crickets to stay on a pile of sand, is there? Oh, one last thing: any idea how frogs find their food/shelter? Visual? Olfactory? I just want to know if the only way the frog's gonna find our sand pile is by running into it. --Ye Olde Luke (talk) 01:59, 27 August 2008 (UTC)[reply]
I don't know, sorry! Hopefully someone else has some experience of catching frogs, I'm just guessing based on what little I know about them. --Tango (talk) 02:23, 27 August 2008 (UTC)[reply]

I once kept frogs. I put them in a terrarium with gravel on the botton and a terracotta dish filled with water. They burrowed under the dish and could not be seen. So a runaway frog might burrow under something damp and cool. Edison2 (talk) 04:00, 27 August 2008 (UTC)[reply]

August 27

Springs and simple harmonic motion

Okay, here's the thing. Physicists model the motion of a block attached to a spring using a sinusiodal function, or simple harmonic motion. I'm fine with this, but why can't the motion of the block be worked out from Newtonian mechanics? I've tried to do it, and haven't gotten very far, but it should be feasible. Normally I wouldn't have a problem, but then my teacher showed that, if our guess of the motion of a spring was correct, the angular frequency ω must equal sqrt(k/m), and used this to show that the amplitude of the initial compression doesn't affect the the period. But isn't this bad physics? After all, it could be that our guess of the motion of the block on the spring was wrong. If I'm not explaining myself well, just let me know. Thanks! —Preceding unsigned comment added by 65.92.231.82 (talk) 00:55, 27 August 2008 (UTC)[reply]

It is done through Newtonian mechanics - you set up the differential equation, which comes from Newton's Second Law of Motion. You can then make the substitution and substitute that in to get an equation of simple harmonic motion. Confusing Manifestation(Say hi!) 01:28, 27 August 2008 (UTC)[reply]
Can you give more details. I'm trying it now, but the d^2 never disappears, and the cosines never apprears. —Preceding unsigned comment added by 65.92.231.82 (talk) 02:27, 27 August 2008 (UTC)[reply]
The key equation of SHM is (which you'll have after the substitution suggested by ConMan - the a will be replaced by an appropriate constant of proportionality involving ks and ms), that is acceleration is proportional to displacement and in the opposite direction. You then simply observe that the sine (or cosine - which one you use depends on boundary conditions) function fits that equation, so you can use it as a solution. Just assume that (this is called an "ansatz"), substitute it in and solve for the constants, then you have your answer. --Tango (talk) 02:40, 27 August 2008 (UTC)[reply]
Is this (using an ansatz) the only way of getting the equation? —Preceding unsigned comment added by 65.92.231.82 (talk) 02:55, 27 August 2008 (UTC)[reply]
It's the only way I know of. It's certainly the easiest. --Tango (talk) 03:40, 27 August 2008 (UTC)[reply]
SEPARATION....OF....VARIABLES. GAHHH. This is the VERY FIRST TECHNIQUE THEY TEACH YOU FOR SOLVING DIFFERENTIAL EQUATIONS. YOU LEARN IT DURING YOUR FIRST SEMESTER OF INTRODUCTORY CALCULUS. AND THEN YOU GO OFF AND GET ADVANCED DEGREES IN ENGINEERING OR APPLIED MATHEMATICS AND LEARN FOURIER TRANSFORMS AND LAPLACE TRANSFORMS AND NUMERIC SOLUTIONS AND FIFTY NINE VARIATIONS ON HOW TO SOLVE DIFFERENTIAL EQUATIONS ... AND YOU FORGET SEPARATION OF VARIABLES. Nimur (talk) 21:36, 27 August 2008 (UTC)[reply]
Firstly, calm down! There is no need to shout. Secondly, it's been a while since I learned how to separate variables and I'm probably just being blind, but how do you separate a 2nd derivative? What am I missing (unless you mean to use ConMan's substitution first, in which there really wasn't a need to shout since he'd already said it)? --Tango (talk) 21:56, 27 August 2008 (UTC)[reply]
It's possible to do using the substitution and a bunch of rearranging (and fiddling around with a plus-or-minus sign), but I agree the ansatz is much neater. More generally, you could use an exponential ansatz, as long as you're prepared to work with complex numbers (and know the relationship between the exponential and trigonometric functions). Confusing Manifestation(Say hi!) 05:10, 27 August 2008 (UTC)[reply]
There is a very good underlying question here. We know that or, more generally, gives a family of solutions to the equation of motion - but how do we know that this covers the whole solution space ? How do we know that there isn't some set of initial conditions that will result in an entirely different solution with very different behaviour ? Well, in this particular case we know that the differential equation is linear, which allows us to use the superposition principle to create a two dimensional family of solutions which we are sure covers the whole solution space. But if the equation of motion is non-linear (if, for example, we exceeded the spring's proportional limit) then it becomes much more difficult to know that we have a complete family of solutions (or even to find any exact solutions in the first place). This is why physicists and engineers will always attempt to linearise a problem if they possibly can.Gandalf61 (talk) 10:19, 27 August 2008 (UTC)[reply]
Is it obvious (I know it's true) that linearity immediately implies a 2d solution space? I might be being unusually stupid, but I don't see it. Algebraist 22:04, 27 August 2008 (UTC)[reply]
It's not obvious to me, but that's not saying a lot - I avoid differential equations when I can help it! --Tango (talk) 22:17, 27 August 2008 (UTC)[reply]
So do I, and (since I also like a big hefty result) I would prove this with the Picard-Lindelöf theorem, but that isn't obvious and doesn't require linearity. Algebraist 22:20, 27 August 2008 (UTC)[reply]
I think the sinusoidal representation is the simplest solution to understand. All motions can be broken down into sine and cosine waves by Fourier analysis any way. Its just that in a linear spring mass system, there are no harmonics above the fundamental. —Preceding unsigned comment added by LargoRhythm (talkcontribs) 21:44, 27 August 2008 (UTC)[reply]

Yawning

I was yawning earlier today, twice, while other people were yawning. It made me wonder, why is yawning contagious? —Preceding unsigned comment added by Earthan Philosopher (talkcontribs) 03:18, 27 August 2008 (UTC)[reply]

See Yawn#Contagiousness, in short - no-one knows. Nanonic (talk) 03:24, 27 August 2008 (UTC)[reply]
The cited article actually agrees with original research and common experience: yawning is highly contagious. Edison2 (talk) 03:57, 27 August 2008 (UTC)[reply]
Yep - I'm pretty sure we know it's contageous. I believe there was a Science Friday piece on NPR that talked about this. What that show said was that yawning is nothing to directly do with tiredness - it's body-language that means something like "We all need to stop what we're doing and do something else now"...which could include going to bed. The reason it's contagious is because other people are subconsciously agreeing that there needs to be a change of group activity. SteveBaker (talk) 04:31, 27 August 2008 (UTC)[reply]
Okay I wrote out a response but lost it since it was blocked. Anyway I'm rather surprised about the article, it mentions yawning in non-human animals but mostly seems to treat these as seperate issues. To me that doesn't make much sense, I see no reason to presume humans are special here, whatever reasons for yawning hold for us likely hold for at least some other animals particularly those close related to us [16]. There are quite a lot of theories of why animals yawn [17], one of them is to show off their teeth. This would also explain why yawning is contagious, if another animal is showing off it's teeth you'd want to as well. Incidentally I came across this [18] paper of a Biology 103 student "We cannot be certain that humans have evolved from monkeys" do people really right that sort of thing in Biology papers? Nil Einne (talk) 08:03, 27 August 2008 (UTC)[reply]
I was under the impression that we can be certain that humans aren't descended from monkeys, but humans and monkeys are both descended from some proto-simian. Algebraist 10:45, 27 August 2008 (UTC)[reply]
I don't know much about the details of yawning, but the picture in that article could make a great lolcat. the wub "?!" 12:09, 27 August 2008 (UTC)[reply]
(Humans are a member of the Ape family - apes are not monkeys) Well, we aren't descended from any living species of ape. All apes are descended from a common antecedent - but that antecedent must in turn have been an animal that you would describe as an ape. That in turn must have descended from some kind of a simian (a "monkey") - but, again, probably not any living species. So the correct statement is: "Humans are descended from an animal that would be classified as an ape. Humans are not descended from any living species of ape." - I don't quite know why people fixate on "monkey" - the closest classification is "ape" and there are plenty of earlier species we descended via - so we're also descended from small furry mammals, fish-like things...bacteria. SteveBaker (talk) 13:38, 27 August 2008 (UTC)[reply]
I think the problem is most people don't understand that apes are usually not considered monkeys, particularly historically when the monkey thing first arose nor do they understand that monkeys (and apes) nowadays are not in fact the same as the ancesteral 'monkeys' (and apes) we all evolved from we so from their POV we are descended from historic modern day monkeys and we get silly questions like why did monkeys stop evolving (what?). (Of course it gets worse when they think humans are the end all of evolution and so ask why monkeys don't evolve into humans but that's another issue.) I'm still rather surprised the student thought it wise to demonstrate that ignorance in a biology paper but anyway... Nil Einne (talk) 20:04, 27 August 2008 (UTC)[reply]

I was told that we yawn because our brain lacksz oxygen. Philosophia X Known(Philosophia X Known) 15:06, 27 August 2008 (UTC)--Earthan Philosopher

No - that would simply cause a slightly increased rate of respiration. Lack of oxygen in the brain is, however the cause of ridiculously large fonts in signatures. SteveBaker (talk) 03:54, 28 August 2008 (UTC)[reply]

man without a face

hi, on channel 7 last night there was a story about a man (or kid?) with a strange growth on his face. Does anyone know if we have an article on this, and what is the condition called? 203.35.135.133 (talk) 07:38, 27 August 2008 (UTC)[reply]

I didn't see that report - but some possibilities are Proteus syndrome, von Recklinghausen's disease, Elephantiasis, Neurofibromatosis. SteveBaker (talk) 13:29, 27 August 2008 (UTC)[reply]
The program might have been aboutJoseph Merrick.
Wanderer57 (talk) 13:33, 27 August 2008 (UTC)[reply]
(That's what I was thinking - and that's where I got the list of diseases from!) SteveBaker (talk) 14:58, 27 August 2008 (UTC)[reply]
It might also be McCune-Albright syndrome or polyostotic fibrous dysplasia as shown on television recently in some countries with the case of Marlie Casseus, a young Haitian girl who had an 18 pound "growth" removed from her face. Nanonic (talk) 15:06, 27 August 2008 (UTC)[reply]
And strangely enough, being Wikipedia, we do have a fair few articles on people known for little more than having an odd tumour. Jose Mestre was featured on a Discovery Channel program called The Man with No Face in which he underwent surgery to have an enormous abnormal growth removed from his face.[19] Nanonic (talk) 15:12, 27 August 2008 (UTC)[reply]
Which was on the TV series My Shocking Story which is being shown on the Seven Network in Australia, so that's probably the person the original poster was talking about. Also see Mark Tatum who had to have his face effectively removed because of mucormycosis. Graham87 16:00, 27 August 2008 (UTC)[reply]

Cat Eyes

Do cats have a second set of eyelids under their regular ones? A little while ago I was petting my cat, and his eyes opened a bit, and there was skin covering them, which opened when I woke him up. Black Carrot (talk) 08:05, 27 August 2008 (UTC)[reply]

Sounds like the nictitating membrane. Algebraist 08:20, 27 August 2008 (UTC)[reply]
It's worth noting that if you can still see the membrane on your cat, you should take it to see a veterinarian. It's usually a sign of illness in cats. —Cyclonenim (talk · contribs) 11:29, 27 August 2008 (UTC)[reply]
Agree, this sounds like an unusual appearance and you should consult a veterinarian. Otolemur crassicaudatus (talk) 15:58, 27 August 2008 (UTC)[reply]
I think that's a bit hasty. It only happened once, and it fits the conditions mentioned in the article - he was asleep, and his eyelids slid open a bit. Thanks for the reference. Black Carrot (talk) 03:27, 28 August 2008 (UTC)[reply]

Can tapirs pick things up with their nose?

I asked this on the miscellaneous reference desk a while ago but no-one really knew - basically, can they use their nose anything like how an elephant uses his trunk? Can they pick up food and pop it into their mouths? Can they hold objects with their noses? Bradley10 (talk) 09:24, 27 August 2008 (UTC)[reply]

This National Geographic video shows them using their noses at least to assist in getting food into their mouths. Fribbler (talk) 11:35, 27 August 2008 (UTC)[reply]

Jurassic Park question

Is what they did in Jurassic Park possible if we did have any dinosaur blood? I know that the mosquito/amber technique wouldn't actually work, but could you, say, clone a dodo from the taxidermy dodos they have in Tring? Or could they make some new Yahzee River Dolphins? Bradley10 (talk) 09:27, 27 August 2008 (UTC)[reply]

DNA doesn't last forever - I don't think the mosquito/amber trick does work. While the shape and tissues of the mosquito are still there - the actual chemical composition of the DNA in the blood in it's stomach is completely wrecked. This is acknowledged in Jurassic Park too - they claim that they had to carefully reassemble the DNA - and fill in the missing bits with Frog DNA (this actually turns out to be the reason for the downfall of the park - most people miss that point in all of the action!)
A Dodo is much more do-able - the last viable DNA is only 350 or so years old, so perhaps it could be made to work. The Yangtze River dolphin is even easier because the DNA may still actually exist in the last few animals.
But using this technique to bring a species back is problematic. To get a viable breeding colony requires a sufficiently diverse gene pool. It's not enough to have one male and one female because all of their offspring would share genes from the same set of two archetypes - and the resulting in-breeding would soon produce all sorts of major health problems. Opinions vary on the minumum number of individuals needed - I heard the number 50 at one time - but a lot depends on how genetically diverse those 50 individuals are - and on how closely scientists could control their subsequent breeding to maximize diversity over the succeeding generations. We probably don't have DNA from 50 Dodo's - and we might not have it from 50 River Dolphins either.
With Jurassic Park, there are even bigger problems - the plants that the herbivores ate are probably all extinct - or perhaps have evolved chemical defenses that the dinosaurs would not be able to counteract (this too comes up in the movie - but very briefly). The idea that the T-Rex can survive on a diet of goat and lawyer is ridiculous - they'd need contemporary animals, which in a real Jurassic Park would be worth vastly too much money to feed them to carnivores. There might also be issues with them surviving modern diseases - but it's also possible that their great antiquity would mean that modern diseases were not able to infect them. It's hard to say. Worst of all - the amount of oxygen in our atmosphere is much less than it was in Jurassic times - the odds are good that these gigantic creatures wouldn't have sufficient lung capacity to survive at all in the modern world.
Another issue (which is carefully skirted in Jurassic Park - but which would affect the Dodo and River Dolphin too) is that having the DNA isn't enough. You also need to turn that DNA into a fertilised egg cell and implant it into the womb of a suitable donor species. In the case of a Yangtze River dolphin - you might be able to implant the egg into a Ganges River dolphin (or one of the other handful of river dolphin species) - but those are also under pressure. You probably can't use one of the more common oceanic dolphins as a surrogate because they live in salt water and river dolphins don't. Same problem with the Dodo - can you find a sufficiently close surrogate mother bird who can lay an egg containing the right nutrients, of sufficient size to contain the chick of a 50 pound bird? The nearest living relatives of the Dodo are pigeons...so no hope there. Perhaps some other large, flightless bird would be able to form a large enough egg - but there is no guarantee that it would have the right shell thickness with the right amount of oxygen transport and the right nutritional stuff.
With dinosaurs, you're in deep problems there. There are no relatives of the dinosaurs left to act as surrogates. This is quietly swept under the carpet in Jurassic Park.
There are some strong candidates for "bringing back" - I think the Mammoth is the most likely. Modern elephants are probably "close enough" to birth a mammoth calf - and we have freeze-dried meat with (perhaps) viable DNA from a significant number of individuals to make a breeding herd. Mammoths lived in sufficiently recent history that diet and such like wouldn't be a major issue.
It's an interesting matter - I think we'll see some of this kind of thing happening...but almost certainly not with Dinoasaurs.
SteveBaker (talk) 13:16, 27 August 2008 (UTC)[reply]
This is intriguing. Thank you.
I'm curious about your point that a T. rex could not survive on a delicious diet of modern goats and lawyers. Based on no knowledge of the matter at all, my impression was that an ancient carnivore such as a dinosaur could live on modern reptiles or mammals. What dietary requirements might be missing? Presumably the fat and protein in modern animals would provide the necessary energy supply and amino acids for growth (or am I wrong on that point as well?) Wanderer57 (talk) 13:59, 27 August 2008 (UTC)[reply]
No doubt the proteins and amino acids are there - but who knows what else too? Perhaps there are missing 'trace elements'? Look at modern animals: A lion can't live if fed only dog food (believe it or not - I know someone who tried that!)...and lions and dogs those are both modern carnivores. Chocolate is poisonous to dogs. Who knows what wierd stuff a dinosaur needs in it's diet? Who knows what stuff in modern foods would be poisonous to it?
Let's do a thought experiment. Consider even a human - cloned into an alien society a few million years in the future when humans and all of the apes and monkeys are long-extinct and giant super-intelligent cockroaches are now running the place. They find a mosquito trapped in amber with human blood in it's gut...they do the Jurassic Park thing and make a baby...so what happens?
The baby is brought into the world from an artificial placenta in an utterly sterile environment to avoid infection. They roach scientists have carefully researched all of the foods humans ate and they've even cloned tomatoes, corn, cows, etc. They somehow reproduce all of the nutrients present in human milk and make sure the baby gets all of that good stuff. But the scientists rapidly discover that the kid's blood won't clot. They have no clue why. They've double-checked the diet - cloned all the right foods - but eventually it dies. Well, maybe there was a cloning problem - maybe the DNA they got was from a haemophiliac. But no. They try this a dozen times with a dozen different clones from different DNA sources - always the exact same problem. They are totally at a loss.
What happened? Well, they cloned the human - but they didn't clone any "gut flora" - the 300 different species of bacteria that are specialised to live in the intestines of humans. A baby naturally gets those from it's mother - through all sorts of mechanisms. But a clone grown in a sterile lab a million years from now wouldn't pick them up. Our gut flora provide us with vitamin K2 - which is needed for blood clotting. No gut flora (or the wrong kind of gut flora or something that's poisoning the gut flora) - no K2 - the clone dies.
How would those future alien scientists figure that out with no living humans or human-descendants to study? Even if they do figure it out - perhaps no suitable bacteria exist in their future world that could substitute for the ones we get naturally. If there are 300 species of them living in our gut - then they would need an ungodly amount of research to figure out which ones they need to clone (and where the heck they'd get the DNA from is anyone's guess) in order to "fix" the poor clone's vitamin K2 deficiency.
We have no clue what subtle things like that we'd have to provide to a baby dinosaur...it's very likely we'd miss a whole bunch of things - even if we DID clone all of the right food plants and such. The problems of raising a creature from the Jurassic would be overwhelming...not just that some species of berry in the park happens to be poisonous - as chocolate is to a dog.
SteveBaker (talk) 14:53, 27 August 2008 (UTC)[reply]
Also, what would happen if the aliens used DNA from one of the uneaten lawyers? They'd get their butts sued off for negligent cloning over the blood clotting issue. Litigious Park! Franamax (talk) 18:21, 27 August 2008 (UTC)[reply]
The only (partly) serious attempt I know of to clone an extinct animal is that of the Thylacine which survived until the 1930s. You may want to read the article for some of the challenges faced, and this is with an animal relatively recently extinct Nil Einne (talk) 19:45, 27 August 2008 (UTC)[reply]
P.S. Just remembered after reading SB's post there has ben some talking regarding the Woolly mammoth too and this is mentioned in the article although I don't think there is a specific effort yet Nil Einne (talk) 19:51, 27 August 2008 (UTC)[reply]

Is there any animal which lacks self-preservation instinct?

So that's the question: does any animal which lacks survival instinct, and survives just "by chance"? Maybe sea sponges? Leptictidium (mt) 11:31, 27 August 2008 (UTC)[reply]

Sea sponges don't have a nervous system - they certainly don't have a brain - so they lack pretty much ALL instinct. So you've certainly answered your own question there. In general, there are lots of animals that don't have brains - or whose brains are far to primitive - so we could probably form a long list that would meet your criterion. But you're probably thinking of higher animals - so let's look in that direction a bit. I'm not sure what you mean by "survival instinct". Survival isn't a single ability - it's a HUGE range of abilities - and "instinct" implies something we're born with - not learned behavior. Humans lack the instinctive ability to eat the right things and exercise properly - that certainly affects our ability to survive in SOME circumstances - yet even newborn babies have the instinct to hold on tightly to your fingers if you try to lift them up by their hands. So to find an animal that lacks some survival instincts is easy - probably all animals have some gaps - but to find one that has no instincts for surviving any set of circumstances probably does require finding one with almost no brain at all. SteveBaker (talk) 12:49, 27 August 2008 (UTC)[reply]
Thank you. Leptictidium (mt) 12:52, 27 August 2008 (UTC)[reply]
Your body doesn't crave certain food when it is missing essential nutrients? I most certainly do have instincts for nutrition and exercise. It is only through years of practice that we manage to suppress those instincts. Plasticup T/C 13:52, 27 August 2008 (UTC)[reply]
The problem is that we evolved in a time when certain high-energy foods (animal fats for example) were in exceedingly short supply. We have the "instincts" to eat large quantities of those foods whenever they are available - but they were so rare and hard to obtain without taking a lot of exercise - that it was impossible to overeat on a routine basis. Therefore we never evolved a mechanism to stop us from eating them when we start to get fat. In a world where you needed to put on a few pounds in the summer in order to survive a long, hard winter - having an instinct to NOT stuff our faces with egg, bacon and cheese McMuffins would have been pretty fatal - so it would be evolved away in no time flat. Worse still - it's pretty clear that we've evolved to crave foods that have high nutritional content - so we not only lack an instinct NOT to over eat - but we also HAVE an instinct to shovel down as much fatty food as we can find! Now we know (intellectually) how bad this instinct is - we can use our big-brain intellects to fight against it...but (for some people at least) it's tough.
It's a similar deal with exercise - we didn't need to evolve a mechanism for craving "pointless" exercise because we got plenty of exercise doing our daily food hunting and the issue of "Death by under-exercise" simply never needed a "survival instinct". Now we no longer do that by necessity - we pretty much lack any instinctive desire to leap on a treadmill a couple of times a week. SteveBaker (talk) 14:13, 27 August 2008 (UTC)[reply]

Glaucoma / abd pain

While reading Cope's "Early Diagnosis of the Acute Abdomen", a trusted reference on the surgical abdomen, I came across the following statement: ...the observer must remember that certain extra-abdominal conditions may cause deceptive pain within the abdomen; thus...even acute glaucoma may temporarily mislead the practitioner. Can anyone explain how glaucoma could possible be misinterpreted as an acute abdomen? Tuckerekcut (talk) 16:38, 27 August 2008 (UTC)[reply]

The short answer is referred pain—conditions which cause pain in one of the body to present as pain elsewhere. The mechanism of pain referral is not well-characterized, but our article addresses some possibilities. This article notes that referred pain in the abdomen can be caused by acute angle-closre glaucoma, and is symptomatic of a loss of blood supply to the eyeball. TenOfAllTrades(talk) 17:48, 27 August 2008 (UTC)[reply]
For anyone else interested in a more complete answer (though your short answer is appreciated Ten), this [20] has a well thought out mechanism. In short, Noxious stimulation of trigeminal nerve afferents activates the paratrigeminal nuclei in the medulla with secondary stimulation of the vagus nerve. Tuckerekcut (talk) 18:23, 27 August 2008 (UTC)[reply]

What happens when blood goes into tissue?

 Done

I know, I know, I know - no medical advice!!! So use some discretion here - but I can't seem to find anything on google..

A moment after I had some blood taken for tests, I noticed a large bulge near the site. Apparently the blood had (at some point) flown into tissue.

Any information I can have on this? Does blood typically get absorbed? How does it get absorbed?

Thanks Rfwoolf (talk) 18:18, 27 August 2008 (UTC)[reply]

Why ask if you know we can't answer? It could be a bruise, or it could be something entirely different. If you are seriously concerned about it, please seek the advice of a physician. --Russoc4 (talk) 18:25, 27 August 2008 (UTC)[reply]
On the contrary you can answer general medical questions so long as it isn't tantamount to medical advice. My question is: What happens when blood is injected (leaked) into tissue? Does it get absorbed? In what way would it get absorbed? Rfwoolf (talk) 18:29, 27 August 2008 (UTC)[reply]
The lesion you describe is known as a hematoma. The entrapped blood will slowly degrade by several mechanisms. Some blood cells will be broken down by natural and mechanically accelerated hemolysis. Macrophages and similar cells may consume and degrade some constituents. Some white blood cells are capable of migrating back to the intravascular space. The plasma will likely diffuse through the tissue and migrate in and out of spaces based on osmotic forces. Some cellular debris may stay in the hematoma site for a long time. Tuckerekcut (talk) 18:35, 27 August 2008 (UTC)[reply]
Thank-you! Hopefully this question will be indexed by google and your answer will be helpful to others will similar questions. Rfwoolf (talk) 18:48, 27 August 2008 (UTC)[reply]

Evolution and the "no new information" argument

I really just don't understand this one. What do creationists even mean when they say "no new information is ever created through mutations/evolution"? Do they mean more genetic code isn't being added? (gene duplication) Or are they ignoring the fact that ACC and CAC, while the same base amino acids, code for completely different proteins? -- MacAddct  1984 (talk &#149; contribs) 19:09, 27 August 2008 (UTC)[reply]

I've never heard this one before (I don't spend much time listening to creationists), so can you give us a link to somewhere the argument is presented? Algebraist 19:13, 27 August 2008 (UTC)[reply]
I'm currently at work and sites such as Answers in Genesis [21] and Dr. Dino [22] are blocked because of its content "religion" (har har). Talk.origins has a brief description, but it's not very informative. I'll provide more links later, if no one expands upon it. -- MacAddct  1984 (talk &#149; contribs) 19:26, 27 August 2008 (UTC)[reply]
My understanding is most creationists claim mutations are always deleterious and/or there is no evidence/it is impossible to form new, more complicated, structures from mutation. Nil Einne (talk) 19:32, 27 August 2008 (UTC)[reply]
See [23] for example (does [24]) work? A mutation, being a random change in highly specified information contained in the nucleic acid base sequence, could almost never do anything but scramble the information; that is, reduce the information. Now sometimes such a loss of information results in a new trait—for example, purple or red flowers where there were only blue ones before (yes I know how dumb that is and how they're missing duplications etc but that's not the point) Nil Einne (talk) 19:36, 27 August 2008 (UTC)[reply]
Thanks Nil and Macaddct. Algebraist 19:39, 27 August 2008 (UTC)[reply]
Yeah, that's exactly the kind of argument that I often seen getting used. (Thanks for the cached link). -- MacAddct  1984 (talk &#149; contribs) 19:59, 27 August 2008 (UTC)[reply]
(edit conflict x5) Sounds like a misunderstanding/misapplication of the Second law of thermodynamics. Although entropy increases overall in a universe sense, biological/living systems can organize/reorganize. --mboverload@ 19:38, 27 August 2008 (UTC)[reply]
Indeed you may also want to see Specified complexity for further understanding of their flawed thinking Nil Einne (talk) 19:40, 27 August 2008 (UTC)[reply]
[EC]The attempted argument is a bastardization of information theory. Objections to evolution#Evolution cannot create information discusses it, but here's the gist: in order to justify a necessisary "intelligent designer", some folks have decided to make the nonscientific (and clearly false claim) that only a intelligent agent can add information into a system because natural systems can, allegedly, only lose information. This natural-systems-can-only-lose-information paradigm stems from a mischaracterization of thermodynamics and incorrect assumptions about the nature of genetic mutation. The Specified complexity arguments of William Dembski are commonly cited (outlined in his "Law of conservation of information"). — Scientizzle 19:47, 27 August 2008 (UTC)[reply]
Oh okay, that's pretty much what I suspected. Thanks for the links. (I was trying to find it listed in criticisms of evolution article maybe I'll redirect it) As always, just blatant and willful ignorance of facts. -- MacAddct  1984 (talk &#149; contribs) 19:59, 27 August 2008 (UTC)[reply]
Various experiments have shown to everyone's satisfaction that species do change over time, so the Creationists have been backed into saying those changes are going "downhill" by losing genetic information:

The rare ‘beneficial’ mutations to which evolutionists cling all appear to be like wingless animals, blind cave animals, and many examples of antibiotic resistance. They are downhill changes, losses of information which, though they may give a survival advantage, are headed in precisely the wrong direction for evolution. (source)

This rebuttal to the recent successes of the critters in the E. coli long-term evolution experiment concludes with "it is only possible to obtain truth about the past if we start with the only source of absolute truth in the present—the inerrant Word of God". So there you are. --Sean 19:55, 27 August 2008 (UTC)[reply]
So they aren't trying to convince him to hand over his "data" then unlike some other creationists? Nil Einne (talk) 20:07, 27 August 2008 (UTC)[reply]

A side note (and hopefully not to stray too far off-topic), Arguments creationists should not use is a beautiful example of back-peddling and apologetics. -- MacAddct  1984 (talk &#149; contribs) 19:59, 27 August 2008 (UTC)[reply]

Oh, I don't know. It's actually much more honest than most Creationist groups in the respect that they recognize that some arguments which are incredibly common that Creationists have been using forever are wrong to the point of making the Creationists look stupid or dishonest (e.g. the Darwin eye misquote, which is always a useful way to spot totally ignorant Creationists). --98.217.8.46 (talk) 21:26, 27 August 2008 (UTC)[reply]

Creationists either don't understand - or refuse to acknowledge - the most basic science on this. The things they say are (without exception) ridiculously easy to demolish - but the people they are aiming these bogus arguments at are desperate to believe whatever crap is hurled in order that they can continue to cling to horribly outmoded beliefs. So there is guaranteed to be a breakdown in communication.

Here is the truth:

Evolutionary change comes about through at least THREE mechanisms:

  1. Sexual reproduction - where two sets of similar (but not identical) genes are shuffled together.
  2. Viral gene insertion where some virus comes along and inserts its DNA into that of the host species.
  3. Mutation - where either a transcription error in the DNA or a zap from radiation or some nasty chemical agent in the environment (a carcinogen, for example) causes the insertion, deletion or corruption of a part of the genetic code - resulting in a brand new gene popping up.

Mutation is a genuine source of completely new genes - they come from absolutely nowhere in the environment - it's a truly random change.

The other two mechanisms are indeed nothing more than a reshuffling of existing genes. But even that can result in totally new "features" in the resulting organism because the consequences of two old, well-established proteins meeting in the same animal for the first time in history can easily result in some bizarre - but useful - side-effect.

So - sure, new 'features' can arise that have never been seen before - there is no problem with some limited pool of information being the only source of genetic material...that's obviously not true. Creationists who claim this are simply sticking their fingers in their ears and singing "La, la-la, la-laaaa. I can't hear you!" because they've been told this plenty of times and clearly enough.

The reason the entropy argument is bogus is because entropy only increases in closed systems - and it's a statistical increase - not an absolute cast-iron increase. So within a group of animals stuck in a "closed system" (on an island or something), the arrival of new mutations results in a very few, rare improvements that survive into the future and a much larger number of disasterously 'damaged' creatures that don't survive long enough to pass on their genes to the next generation. The total entropy of the island does exactly what the laws of physics predict - it increases.

Life on earth is evolving and improving - but at the cost of increasing the chaos on the planet at a much higher rate than would occur if there was no life (or life without evolution).

We can simulate this in a computer relatively easily. When we do this, we can actually measure the entropy of the system and watch it increase as predicted by physics while the synthetic "animals" befome more and more well-suited to their artificial environment.

SteveBaker (talk) 03:40, 28 August 2008 (UTC)[reply]

The evolution of intelligence

It seems odd to me that at least three, probably four, groups of animals have developed highly complex and intelligent brains independently (corvids, parrots, cetaceans, and primates), during the Cenozoic era, while no known fossil from the Mesozoic has an encephalization quotient much above that of an ostrich (the highest are troodontids). (I say "possibly four" because there were parrots in the Cretaceous, but I'm not sure about their intelligence.)

Is there any speculation as to why this is? (Is the Mesozoic fossil record poor enough that we could have missed an entire taxon as significant as, say, parrots or corvids?) —Preceding unsigned comment added by Vultur (talkcontribs) 23:33, 27 August 2008 (UTC)[reply]

Objects rotating in space

I contend that it is impossible for an object to be rotating in more than one axis simultaneously without a continual force acting upon it, but there are others that disagree with me. Picture a pencil spinning along its axis, nudged so that it also spins end over end. Doesn't the gyroscope effect prevent an object that is rotating on one axis from also rotating on another axis such that after it has rotated 180° it is then rotating in the opposite direction on the original axis of rotation? Surely this periodic reversal of rotation (in an absolute, external observer sense) requires continuous application of force? How can I more clearly elucidate my point, or am I mistaken? — PhilHibbs | talk 23:38, 27 August 2008 (UTC)[reply]

Any two rotations in 3D space around any two axes (that intersect - presumably at the object's centre of mass) combine to make a single rotation around a single axis, see Euler's rotation theorem. This means that if you try to make something rotate simultaneously around two axes you'll actually find that it's rotating about one, different, axis. --Tango (talk) 23:56, 27 August 2008 (UTC)[reply]
And that rotation can't change without external force, by the conservation of angular momentum. Algebraist 23:57, 27 August 2008 (UTC)[reply]
Certainly you can combine any number of rotations in to a single rotation about some other axis. Whether you can change the rotation without an external force is a little tricky. The classic example of the spinning ice-skater who - with arms outstretched - can change the SPEED of her rotation by pulling her arms inwards. That's a change in speed - but not of the axis of rotation. However if she pulls in just one arm, she'll start spinning around an axis that DOESN'T run through her head and her feet. But she's still spinning about the same axis from a point of view of conservation of rotational momentum because her head/body moved one way as the arm moved the other way - the axis of rotation for the entire system is still vertical with respect to the ice - but from the perspective of the skater, it changed.
This principle is important for spacecraft. Using thrusters to change your axis of rotation consumes reaction mass - rocket fuel - which is valuable stuff...but instead you can have a big flywheel inside and use solar power to spin it up and to keep it spinning against friction. When the spacecraft needs to rotate, it applies forces to the flywheel - the flywheel spins one way - the remainder of the spacecraft rotates the other way in order to conserve rotational momentum. The spacecraft APPEARS to rotate without any external force being applied - where in physical terms, it's total rotational momentum didn't change. This mechanism is how the Hubble Space telescope turns to point where it needs to point without ever running out of fuel.
Gyroscopes are not magical - they use the same principles. When you see a kid's toy gyro spinning on a table - you do indeed see it "precessing" (it's axis of rotation changes) - but that's because there IS an external force - friction with the table (or whatever mounting points it uses) and gravity.
SteveBaker (talk) 03:06, 28 August 2008 (UTC)[reply]
I don't know, I still find gyroscopes kind of freaky. Confusing Manifestation(Say hi!) 03:59, 28 August 2008 (UTC)[reply]

Spider silk

After watching a spider lower itself from the ceiling, I got to wondering. What happens to such a strand after the critter is done with it? Does it hang there until it disintegrates or somebody comes by? Or does the spider have some way to reel it in and recycle it? Spider silk answereth not. Clarityfiend (talk) 02:10, 28 August 2008 (UTC)[reply]

See spider web —Preceding unsigned comment added by 79.76.196.178 (talk) 02:38, 28 August 2008 (UTC)[reply]
It says "It is not uncommon for spiders to eat their own web daily...", but how would a spider do that with a single strand when there's no other strands to move about on, unlike a web? Clarityfiend (talk) 03:41, 28 August 2008 (UTC)[reply]

What eats starfish?

What kinds of animals are predators of starfish? The starfish article mentions predators in at least two places, but does not elaborate. -- Dominus (talk) 03:58, 28 August 2008 (UTC)[reply]

Do today's dyes weaken fabrics still?

My mom just told me that back in the days, yellow (I think!) clothing ripped most easily because yellow dye weakened the fabric. Is that true? Is it still true? What about other colors? Thanks. 67.243.6.204 (talk) 04:31, 28 August 2008 (UTC)[reply]